Dit 2014 Step 2 Ck-1

December 10, 2016 | Author: Anonymous anyiNExxMQ | Category: N/A
Share Embed Donate


Short Description

Download Dit 2014 Step 2 Ck-1...

Description

USMLE STEP 2 CK REVIEW

STUDY GUIDE 2014 EDITION

DOCTORS IN TRAINING* STRUCTURED. FOCUSED. AWESOME.

B rian J enkins , MD

Doctors In Training.com: U SM LE Step 2 CK Review, 2014 edition

Author: Brian Jenkins, M D

Copyright © 2014 D octors In Training.com , LLC w w w .doctorsintrainingxom

Doctors In Training.com , LLC 1701 River Run, Suite #750 Fort W orth, T X 76107 All Rights Resei ved. 1his test is protected by copyright No pari olthis I oh may be reproduced in any Ibi ni or by any means, including photocopying, or util bed by any storage and retrieval system, without written permission from the i pyt ight owner.

DISCLAIMER: THE AUTHO R DISCLAIMS ANY I.IAHIL ITY, LOSS, INJURY, O R DAMAGE INCURRED AS A CO NSEQ UENTS, DIRECTLY O R INDIRECTLY. O r THE USE A N D APPI iCAT ION O F AMY O F THE CO N TEN T A N D MATE RIAL CO NTAIN ED IN THIS TEXT. ALTHOUGH THE INFORMATION IN THIS TEX I HAS BEEN CAREFULLY REVIEWED I OR CORRECTNESS. Wlf AUTHO R CA N N O T ACCEPT ANY RESPONSIBILITY FOR ANY ERRORS O R OMISSIONS THAT MAY BE MADE. THE A U 1IIO R MAKES N O W ARRANT Y, EXPRESS ORTMPLIED. AS TO THE COMPLETENESS. CURRENCY Oft ACCURACY o r IHE CONTENT'S OF THIS TEXT. THE INFORMATION CO NTAIN ED IN Ti-fTS TEXT SHO U LD N O T BE CONSTRUED AS SPECIF­ IC INSTRUCT IONS TOR INDIVIDUAL PATIENTS, MANUFACTURER'S PRODUCT INFORMATION A N D PACKAGE INSERTS SHOULD BE. REVIEWED TOR CURRENT INFORMATION, INCLUDING CO N TRA IN DICA I IONS. DOSAGES. AN D PRECAUTIONS. I or p. oblems. questions, or r.onujriiy you may contact die authiv at supExirtifialor tor-sr.traintng.toni

N EU R O LO G Y 1. 2. 3. 4. S. 6. 7. 8. 9. 10. 11. 12. 13. 14. 15. 16. 17.

Normal Neuro Function Meningitis O ther CNS Infections Headache T IA Stroke Hemorrhage Seizures Degenerative Disorders part 1 Degenerative Disorders part 2 Peripheral Disorders Neoplasms and Sleep Loss of Consciousness Pedi Neuro Ophthalmology part 1 Ophthalmology part 2 Audiovestibular Disorders

3 4 6 8 11 13 15 17 19 21 23 26 29 32 34 37 40

P S Y C H IA T R Y 1. 2. 3. 4. 5. 6. 7. 8. 9. 10. 11.

Depression Antidepressants O ther M ood Disorders Anxiety Disorders Psychotic Disorders Personality Disorders Substance Abuse part 1 Substance Abuse part 2 O ther Psych Disorders Delirium and Dementia Pedi Psych

45 47 51 53 55 59 61 62 64 66 68

E N D O C R IN O L O G Y 1. 2. 3. 4. 5. 6. 7. 8. 9. 10. 11.

Type 1 Diabetes Type 2 Diabetes Insulin Therapy Acute Complications of Diabetes Chronic Complications of Diabetes Normal Thyroid Function and Hyperthyroidism Hypothyroidism and Thyroid Cancer Parathyroid Disorders Pituitary Disorders Cushing Syndrome and Hyperaldosteronism O ther Adrenal Disorders

73 74 76 77 79 81 83 86 88 91 92

TABLE OF C O N T E N T S

TABLE OF CONTENTS

TABLE OF CO N TEN TS

TABLE OF CONTENTS IV

E R , IC U , S U R G E R Y 1. 2. 3. 4. 5. 6. 7. 8. 9. 10. 11. 12.

V

95 98 100 102 105 108 111 113 116 121 122 123

C A R D IO V A S C U L A R 1. 2. 3. 4. 5. 6. 7. 8. 9. 10. 11. 12. 13. 14. 15. 16. 17. 18.

VI

Accidents and Injuries part 1 Accidents and Injuries part 2 Toxicology part 1 Toxicology part 2 Toxicology part 3 Cardiovascular Emergencies Critical Care Trauma part 1 Trauma part 2 Trauma part 3 Pre-Op and Post-Op Issues Surgical Emergencies and Transplantation Cardiology Basics Atherosclerosis Hypercholesterolemia Stable Angina Unstable Angina Myocardial Infarction Arrhythmias part 1 Arrhythmias part 2 H eart Failure Valvular Diseases Cardiomyopathies and Pericardial Disease Myocardial Infections Hypertension Antihypertensives Shock Vascular Conditions Vasculitis Pedi Cardiology

127 128 129 130 132 134 135 137 138 140 142 144 145 147 148 150 152 154

PU LM O N O LO G Y 1. 2. 3. 4. 5. 6. 7. 8. 9. 10. 11.

URI Lower Respiratory Infections part 1 Lower Respiratory Infections part 2 ARDS and Asthma C O PD Neoplasms and Interstitial Lung Disease Pulmonary Vascular Diseases Pleural Diseases Sleep Apnea and Pulmonary Surgical Concerns Pedi Pulmonology part 1 Pedi Pulmonology part 2

159 161 163 165 167 169 171 173 174 176 179

TABLE OF CONTENTS V II

G A STR O EN TER O LO G Y 1. 2. 3. 4. 5. 6. 7. 8. 9. 10. 11. 12. 13. 14. 15.

V I II

Biostatistics Research Studies Abstracts and Advertisements Ethics

211 214 216 220

G E N IT O U R IN A R Y 1. 2. 3. 4. 5. 6. 7. 8. 9. 10. 11. 12. 13.

X

183 184 185 187 189 190 193 195 197 199 200 201 204 206 207

E P ID E M IO L O G Y A N D E T H IC S 1. 2. 3. 4.

IX

GI Infections Viral Hepatitis Oral and Esophageal Conditions Gastric Conditions Small Intestine part 1 Small Intestine part 2 Large Intestine part 1 Large Intestine part 2 Colorectal Cancer and G I Bleeding Pancreatic Diseases Biliary Diseases part 1 Biliary Diseases part 2 Alcoholic Liver Disease and Cirrhosis O ther Hepatic Diseases Pedi G I Disorders

Diuretics Disorders of the Kidney part 1 Disorders of the Kidney part 2 Nephritic Syndromes Nephrotic Syndromes Renal Failure Acid-Base Disorders Hyponatremia O ther Electrolyte Disorders Bladder and Ureteral Disorders M ale Reproduction part 1 M ale Reproduction part 2 Pedi Urology

225 226 229 231 232 234 236 239 242 244 245 248 251

H EM E/O N C 1. 2. 3. 4. 5. 6. 7. 8. 9. 10. 11. 12. 13.

Anemia part 1 Anemia part 2 Anemia part 3 Genetic Disorders of Hemoglobin Leukocyte Disorders and Hypersensitivity Thrombocytopenia Coagulopathies and Hypercoagulable States Hematologic Infections H IV H IV Treatment Myeloma and Lymphoma Leukemia Pedi H em e/O nc

255 257 258 259 260 262 264 266 268 269 272 274 276

TABLE OF C O N T E N T S

TABLE OF CONTENTS XI

M U SCU LO SK ELETA L 1. 2. 3. 4. 5. 6. 7.

Orthopedics part 1 Orthopedics part 2 Metabolic Bone Diseases Infections, OA and Neoplasms RA and Lupus Other Rheumatologic Diseases Pedi Ortho

281 284 286 288 290 292 295

D ER M A TO LO G Y 1. 2. 3. 4. 5. 6.

Infections part 1 Infections part 2 Infections part 3 Inflammatory Skin Conditions Bullous Diseases and Neoplasms Plastics, Pigmentation and H air Loss

301 304 308 311 316 318

G YN ECO LO G Y 1. 2. 3. 4. 5. 6. 7. 8. 9. 10. 11. 12.

Menstrual Physiology Menopause Contraception Amenorrhea Menstrual Disorders part 1 Menstrual Disorders part 2 PCOS and Pelvic Prolapse Gynecological Infections and STDs Uterine and Cervical Neoplasms Vaginal and Ovarian Neoplasms Benign Breast Disorders Breast Cancer

325 327 330 332 335 337 339 341 343 345 347 349

O B S T E T R IC S 1. 2. 3. 4. 5. 6. 7. 8. 9. 10. 11. 12. 13. 14.

Normal Pregnancy Physiology Prenatal Care Medical Complications part 1 Medical Complications part 2 M aternal Drug Use Congenital Infections Obstetric Complications part 1 Obstetric Complications part 2 Obstetric Complications part 3 Obstetric Complications part 4 L& D: Assessment of Fetus L& D: Labor L&.D: Malpresentation and Cesarean Section Postpartum Care

353 355 358 360 362 364 367 369 371 373 375 377 379 381

P E D IA T R IC S 1. 2. 3. 4. 5.

Development Infancy to Adolescence Infections and Immune Disorders Genetic Disorders Preventive Medicine

387 389 393 399 402

NOTES COURSEVIEWING OPTIONS

The course consists of 169 instructional videos. Examples of 13, 15, 17, 21, 28, and 34 day plans are provided below for maximum flexibility to meet your personal study needs.

No. of Videos/Day

Video Runtime/Day*

13-day plan

13

Approx. 4.5 hrs

15-day plan

11

Approx. 4 hrs

17-day plan

10

Approx. 3.5 hrs

21-day plan

8

Approx. 3 hrs

28-day plan

6

Approx. 2 hrs

34-day plan

5

Approx. 1.5 hrs

*Does not include study breaks or time spent annotating and answering questions.

A

l t h o u g h y o u h a v e t h e f l e x ib il it y t o v ie w t h e v id e o s in a n y o r d e r , w e s t r o n g l y r e c o m m e n d

HOW MANY VIDEOS YOU VIEW IN A DAY.

Introduction

1A IB

Neurology 1 - Normal Neuro Function

1C

Neurology 2 - M eningitis

ID

Neurology 3 - O ther CNS Infections

IE

Neurology 4 - Headache

IF 1G

Neurology 5 - T IA

1H

Neurology 7 - Hemorrhage

Neurology 6 - Stroke Neurology 8 - Seizures

ij IK

■8

Psychiatry 1 - Depression Psychiatry 2 - Antidepressants Psychiatry 3 - O ther M ood Disorders

2A

Psychiatry 4 - Anxiety Disorders

2B

Psychiatry 5 - Psychotic Disorders

2C

Neurology 9 - Degenerative Disorders part 1

2D

Neurology 10 - Degenerative Disorders part 2

2E

Neurology 11 - Peripheral Disorders

2F

Neurology 12 - Neoplasms and Sleep

2G

Neurology 13 - Loss o f Consciousness

2H

Neurology 14 - Pedi Neuro

21

Neurology 15 - Ophthalmology part 1

2J 2K

Neurology 16 - Ophthalmology part 2 Neurology 17 - Audiovestibular Disorders

3A

Psychiatry 6 - Personality Disorders

3B

Psychiatry 7 - Substance Abuse part 1

3C

Psychiatry 8 - Substance Abuse part 2

3D

Psychiatry 9 - O ther Psych Disorders

3E

Psychiatry 10 - Delirium and Dementia

3F

Psychiatry 11 - Pedi Psych

3G

Endocrinology 1 - Type 1 Diabetes

3H

Endocrinology 2 - Type 2 Diabetes

31

Endocrinology 3 - Insulin Therapy Endocrinology 4 - Acute Complications o f Diabetes

3J 3K

Endocrinology 5 - Chronic Complications of Diabetes

COURSE O R D E R

THAT YOU WATCH THE VIDEOS IN THE ORDER IN WHICH YOUR DASHBOARD PRESENTS THEM REGARDLESS OF

RECOMMENDED

RECOMMENDED COURSE ORDER

COURSE ORDER RECOMMENDED

RECOMMENDED COURSE ORDER

Hi

4A 4B

Endocrinology 6 - Normal Thyroid Function and Hyperthyroidism Endocrinology 7 - Hypothyroidism and Thyroid Cancer

4C 4D 4E 4F 4G 4H 41 4J 4K 5A

Endocrinology 8 - Parathyroid Disorders Endocrinology 9 - Pituitary Disorders Endocrinology 10 - Cushing Syndrome and Hyperaldosteronism Endocrinology 11 - Other Adrenal Disorders ER, ICU, Surgery 1 - Accidents and Injuries part 1 ER, ICU, Surgery 2 - Accidents and Injuries part 2 ER, ICU, Surgery 3 - Toxicology part 1 ER, ICU, Surgery 4 - Toxicology part 2 ER, ICU, Surgery 5 - Toxicology part 3 Cardiovascular 1 - Cardiology Basics

5B 5C 5D

Cardiovascular 2 - Atherosclerosis Cardiovascular 3 - Hypercholesterolemia Cardiovascular 4 - Stable Angina

5E SF 5G

Cardiovascular 5 - Unstable Angina Cardiovascular 6 - Myocardial Infarction Cardiovascular 7 - Arrhythmias part 1

5H

Cardiovascular 8 - Arrhythmias part 2

SI 5J

ER, ICU, Surgery 6 - Cardiovascular Emergencies Cardiovascular 9 - Heart Failure

5K 6A 6B

Cardiovascular 10 - Valvular Diseases Cardiovascular 11 - Cardiomyopathies and Pericardial Disease-

6C

Cardiovascular 12 - Myocardial Infections Cardiovascular 13 - Hypertension

6D 6E

Cardiovascular 14 - Antihypertensives Cardiovascular 15 - Shock

6F

ER, ICU, Surgery 7 - Critical Care

6G 6H

Cardiovascular 16 - Vascular Conditions Cardiovascular 17 - Vasculitis

61

Cardiovascular 18 - Pedi Cardiology ER, ICU, Surgery 8 - Trauma part 1

6J 6K 6L

ER, ICU, Surgery 9 - Trauma part 2 ER, ICU, Surgery 10 - Trauma part 3

ER, ICU, Surgery 11 - Pre-O p and Post-Op Issues

7B

ER, ICU, Surgery 12 - Surgical Emergencies and Transplantation Pulmonology 1 - U R I

7C 7D 7E

Pulmonology 2 - Lower Respiratory Infections part 1

7F

Pulmonology 4 - A R D S and Asthm a Pulmonology 5 - C O PD

' 7G

IBM

Pulmonology 3 - Lower Respiratory Infections part 2

Pulmonology 6 - Neoplasms and Interstitial Lung Disease

71

Pulmonology 7 - Pulmonary Vascular Diseases

7J 7K

Pulmonology 8 - Pleural Diseases Pulmonology 9 - Sleep Apnea & Pulmonary Surgical Concerns

7L

Pulmonology 10 - Pedi Pulmonology part 1

7M

Pulmonology 11 - Pedi Pulmonology part 2

8A

Gastroenterology 1 - GI Infections

8B

Gastroenterology 2 - Viral Hepatitis

8C

Gastroenterology 3 - O ral and Esophageal Conditions

8D

Gastroenterology 4 - Gastric Conditions

8E

Gastroenterology 5 - Small Intestine part 1

8F

Gastroenterology 6 - Small Intestine part 2

8G

Gastroenterology 7 - Large Intestine part 1

8H

Gastroenterology 8 - Large Intestine part 2

81

Gastroenterology 9 - Colorectal Cancer and G I Bleeding

8J 9A

Gastroenterology 10 - Pancreatic Diseases

9B 9C

Gastroenterology 12 - Biliary Diseases part 2 Gastroenterology 13 - Alcoholic Liver Disease and Cirrhosis

9D

Gastroenterology 14 - O ther Hepatic Diseases

9E

Gastroenterology 15 - Pedi G I Disorders

9F

Epidemiology and Ethics 1 - Biostatistics

9G

Epidemiology and Ethics 2

9H

Epidemiology and Ethics 3 - Abstracts and Advertisements

91

Epidemiology and Ethics 4 - Ethics

9J 9K

Genitourinary 1 - Diuretics

9L

Genitourinary 3 - Disorders of the Kidney part 2

Gastroenterology 11 - Biliary Diseases part 1

Research Studies

G enitourinary 2 - Disorders of the Kidney part 1

COURSE O RD ER

7A

RECOMMENDED

RECOMMENDED COURSE ORDER

RECOMMENDED COURSE ORDER

RECOMMENDED COURSE ORDER 10A 10B IOC 10D 10E 10F 10G 10H 101 10J llA 11B 11C 11D I IF 111 11G

Genitourinary 4 - Nephritic Syndromes Genitourinary 5 - Nephrotic Syndromes Genitourinary 6 - Renal Failure Genitourinary 7 - Acid-Base Disorders Genitourinary 8 - Hyponatremia Genitourinary 9 - Other Electrolyte Disorders Genitourinary 10 - Bladder and Ureteral Disorders Genitourinary 11 - Male Reproduction part 1 Genitourinary 12 - Male Reproduction part 2 Genitourinary 13 - Pedi Urology Heme/Onc Heme/Onc Heme/Onc Heme/Onc Heme/Onc Heme/Onc

1 - Anemia part 1 2 - Anemia part 2 3 - Anemia part 3 4 - Genetic Disorders of Hemoglobin 5 - Leukocyte Disorders and Hypersensitivity 6 - Thrombocytopenia

HI

Heme/Onc 7 - Coagulopathies and HyperCoagulable States Heme/Onc 8 - Hematologic Infections Heme/Onc 9 - H IV

in UK

Heme/Onc 10 - H IV Treatment Heme/Onc 11 - Myeloma and Lymphoma

n il

11L Heme/Onc 12 - Leukemia 11M Heme/Onc 13 - Pedi Heme/Onc 12A Musculoskeletal 1 - Orthopedics part 1 12B Musculoskeletal 2 - Orthopedics part 2 12C Musculoskeletal 3 - Metabolic Bone Diseases 12D Musculoskeletal 4 - Infections, OA and Neoplasms 12E

Musculoskeletal 5 - RA and Lupus 12F Musculoskeletal 6 - O ther Rheumatologic Diseases 12G Musculoskeletal 7 - Pedi Ortho 12H Dermatology 1 - Infections part 1 121 Dermatology 2 - Infections part 2 12J Dermatology 3 - Infections part 3 12K Dermatology 4 - Inflammatory Skin Conditions

RECOMMENDED COURSE ORDER 13A 13B

Dermatology 5 - Bullous Diseases and Neoplasms

:' r v ; v

:7



: i , v- : r

i:;V

.

Dermatology 6 - Plastics, Pigmentation and H air Loss Gynecology 1 M enstrual Physiology 13D Gynecology 2 - Menopause 13E Gynecology 3 - Contraception 13F Gynecology 4 y Amenorrhea 13C

13G Gynecology 5 - M enstrual Disorders part 1 13H Gynecology 6 - M enstrual Disorders part 2 131 Gynecology 7 - PCO S and Pelvic Prolapse 13J Gynecology 8 - Gynecological Infections and STDs 14A Gynecology 9 - Uterine and Cervical Neoplasms

1 § §



14B

Gynecology 10 - Vaginal and Ovarian Neoplasms

14C

Gynecology 11 - Benign Breast Disorders

14D

Gynecology 12 - Breast Cancer

14E

Obstetrics 1 - Normal Pregnancy Physiology

14F

Obstetrics 2 - Prenatal Care

14G

Obstetrics 3

14H

Obstetrics 4 - Medical Complications part 2

141

Obstetrics 5 - M aternal Drug Use

14J

Obstetrics 6 - Congenital Infections

-

Medical Complications part 1

15A Obstetrics 7 - Obstetric Complications part 1 15B

Obstetrics 8 - Obstetric Complications part 2

15C

Obstetrics 9 - Obstetric Complications part 3

15D Obstetrics 10 - Obstetric Complications part 4 15E Obstetrics 11 - L& D : Assessment of Fetus

1111 1SF Obstetrics 12 - L& D : Labor 15G

Obstetrics 13 - L& D : Malpresentation and Cesarean Section

1511 Obstetrics 14 - Postpartum Care

§§J§

mm

151 15J 15K

Pediatrics 1 - Development Pediatrics 2 - Infancy to Adolescence Pediatrics 3

-

Infections and Immune Disorders

15L

Pediatrics 4

-

Genetic Disorders

3 -5 M

Pediatrics 5

-

Preventive Medicine

vasai

N eurology 1

N orm al N euro F unction

2

M en in g itis

3

O th e r C N S Infections

4

H eadache

5

T IA

6

Stroke

7

H em orrhage

8

Seizures

9

D egenerative D isorders p a rt 1

10 D egenerative D isorders p a rt 2 11 P eripheral D isorders 12 N eoplasm s and Sleep 13 Loss o f Consciousness 14 Pedi N euro 15 O p h th alm o lo g y p a rt 1 16 O p h th alm o lo g y p a rt 2 17 A udiovestibular D isorders

NORMAL NEURO FUNCTION

End of Session Quiz 1.

W h a t are the tw o m ost common locations o f aneurysms in the circle o f Willis?

2.

W h ic h spinal cord lesion matches each o f the following descriptions? ° Fasciculations but also spastic paralysis ° Impaired proprioception + pupils do not react to light ° Bilateral loss of pain and temp below the lesion + hand weakness ° Bilateral loss of vibration sense + spastic paralysis of legs then arms ° Bilateral loss of pain/temp below lesion + bilateral spastic paralysis below lesion + bilateral flaccid paralysis at the level of the lesion

3.

A lesion to which area o f the brain is responsible for each o f the following clinical scenarios? o Contralateral hemiballismus ° Hemispatial neglect syndrome ° Poor comprehension • Poor vocal expression ° Personality changes ° Agraphia and acalculia (inability to write, inability to do mathematical calculations)

MENINGITIS

3 Question Warm-Up 1

W here does each o f the following spinal tracts decussate/cross over? » Dorsal columns ° Lateral corticospinal tract ° Spinothalamic tract

2.

W h a t cerebral artery infarct can cause aphasia?

3.

W hat are the characteristic features o f Brown-Sequard syndrome?

Bacterial Meningitis 4. W hat are the common organisms and empiric IV antibiotic choices for bacterial meningitis based on the age of the patient? Age Range

Organisms

Empiric Antibiotics

60 years, alcoholism, or debilitating comorbidities

1) ampicillin (Listeria coverage) 2) cefotaxime or ceftriaxone 3) vancomycin 4) dexamethasone IV q6hrs x4d

5. W hat is the rational for dosing dexamethasone prior to or along with the first dose of antibiotics for empiric treatment of bacterial meningitis? ° Dexamethasone, when given with or prior to the first dose of antibiotic, reduces the risk o f___________________________ inchildren with meningitis, especially in the cases o f___________________________ . ° In adults with bacterial meningitis, dexamethasone reduces both morbidity and mortality, especially in the case o f _________________________ .

Viral Meningitis 6. W hat is the treatm ent for viral meningitis?

End of Session Quiz 7.

W h a t organism is responsible for bacterial meningitis given each o f the following findings on C S F examination? ° Gram-positive diplococci ° Gram-negative diplococci ° Small pleomorphic Gram-negative coccobacilli ° Gram-positive rods and coccobacilli

8.

W h a t medication should be given to close contacts o f those w ith either meningococcal or H ib (Haemophilus influenzae type B) meningitis?

9.

W h en should a C T scan be performed as a next step instead o f an L P in a patient suspected o f having meningitis?

10.

You suspect an A ID S patient may have meningitis. W h a t fungal infection should you be m ost concerned about?

11.

W h a t is the treatm ent for fungal meningitis?

12.

W h a t medications are used in combination in the treatm ent o f T B meningitis?

z

£! —I CO

NEUROLOGY

° Acetaminophen for pain ° IV fluids as needed ° Empiric antibiotics until___________________________ If younger than 3 years, severely ill or immunocompromised, continue empiric antibiotics until bacterial culture results confirm nonbacterial etiology ° ___________________ if suspicion of HSV or signs of encephalitis such as focal neurologic findings Discontinue if HSV PCR and cultures are negative or alternative diagnosis is made

Z z

m

OTHER CNS INFECTIONS

3 Question Warm-Up 1.

W h a t is the most effective way to prevent bacterial meningitis in newborns?

2.

W h a t other drug should be given just before or along with the first dose of antibiotics in a patient suspected o f having bacterial meningitis?

3.

W h a t is the most common location of a berry aneurysm?

W est Nile Virus ° Birds are the reservoir, and mosquitoes are the vectors. Humans, horses and dogs are incidental hosts ° Sx: usually only headache, malaise, back pain, myalgia and anorexia for 3-6 days (“flu­ like”) ° Severe Sx in 1/150: meningitis +/- encephalitis including muscle weakness and flaccid paralysis (via anterior horn involvement), alterations in consciousness, possibly death ° D x:______________________________ ° Treatment:________________________

End of Session Quiz 5.

A patient is adm itted to the hospital w ith the presumptive diagnosis o f viral meningitis. A n M R I o f the head shows lesions within the right temporal lobe. W ith which pathogen is this pattern most consistent?

6.

W h a t are the features o f Reye syndrome?

7.

H o w should you treat a patient w ho has been bitten by an animal suspected o f having rabies or by an animal th at cannot be observed for 10 days?

8.

A patient is brought into the E R w ith progressive muscle weakness, retained sensation, headache, vomiting, neck pain and fever. C S F analysis shows increased lymphocytes and norm al glucose and protein. W h a t life-threatening complication can result i f this disease progresses?

HEAD ACH E

HEADACHE

3 Question Warm-Up 1

H ow do the symptoms o f encephalitis differ from those o f meningitis?

2.

Fill in the table o f C SF findings in cases o f meningitis caused by different types o f pathogens.

Pressure

W BCs

Glucose

Protein

Healthy Bacterial Viral TB/ fungal 3.

4.

W h a t other term should you remember when thinking about Reye syndrome?

W hat are the differences in the acute treatment of tension headaches, cluster headaches and migraine headaches?

Type of Headache

Treatment

Tension headache

NSAIDs

Cluster headache

100% 0 2 (6+ L/m in on non-rebreather for 20+ min) and sumatriptan or dihydroergotamine (D H E 45)

Migraine headache

Sumatriptan (or other triptan), dihydroergotamine (D H E 45), NSAIDs, and/or antiemetics (chlorpromazine, prochlorperazine, metoclopramide) in varying combinations based on severity, nature of symptoms and patient’s history

S. W hat agents can be used for prophylaxis of migraine headaches? : verapamil (often first-line b/c safe and well tolerated) propranolol, metoprolol (good choice if comorbid hypertension) amitriptyline, nortriptyline (good choice if comorbid depression, insomnia, pain syndrome) ------------------------------ : naproxen (good choice if menstrual migraine or comorbid osteoarthritis or other pain that could benefit from NSAIDs) ------------------------------ : valproic acid (good if history of bipolar disorder), topiramate, gabapentin

6. W hat headache symptoms would lead you to suspect a brain tum or as a cause of a headache?

m > O >

n x m

Pseudotumor Cerebri 7. W hat are the characteristic features of pseudotumor cerebri? ° Young, obese woman ° Headaches —daily (worse in the morning), pulsatile, possible nausea/vomiting, possible retroocular pain worsened by eye movement O ° Most worrisome sequela is vision loss ° C T scan:___________________________ 0 CSF pressure elevated ( ______________ in non-obese patient,______________ in obese patient)

8. W hat treatm ent options are available for managing pseudotumor cerebri? ° Confirm absence of other pathology with C T and M RI of the head (r/o central venous thrombosis) ° Discontinue any inciting agents (e.g.,_______________________________ ) ° ___________________________ in obese patients o ___________________________ - first line (start 250mg qid or 500mg bid —> increase to 500mg qid to lOOOmg qid) ° Invasive treatment options - Serial lumbar punctures - Optic nerve sheath decompression Lumboperitoneal shunting (CSF shunt)

[9 )

NEUROLOGY

° Mild headache which progressively worsens over days to weeks ° New onset after age 50 ° _________ ; worsened by bending, lifting, cough or Valsalva maneuver (increased intracranial pressure) ° Associated seizures, confusion, altered mental status ° Abnormal neurologic signs and symptoms (e.g., focal numbness or weakness) ° Disturbs sleep or presents immediately upon awakening ° Vomiting precedes headache ° Known systemic illness (e.g., cancer, HIV, or collagen-vascular disorder)

X

What is the most likely cause of headache based on each of the following descriptions? Made worse by foods containing tyramine Obese woman with papilledema Jaw muscle pain when chewing Periorbital pain with ptosis and miosis Photophobia and/or phonophobia Bilateral frontal/occipital pressure Lacrimation and/or rhinorrhea Elevated ESR “Worst headache of my life” Headache + extraocular muscle palsies Scintillating scotomas prior to headache Headache occurring either before or after orgasm Responsive to 100% oxygen supplementation Frontal headache made worse by bending over Trauma to the head —> headache begins days after the event, persists for over a week and does not go away

E nd o f Session Quiz 10. W h a t is the pattern o f pain in a migraine? In a tension headache?

11. A 27-year-old m an comes to the clinic because o f a progressively worsening headache. H e says that he never used to have headaches until this one. H e adds that this one was easy to ignore at first but over the last few weeks has never let up and is increasing in severity. W h a t should be next for this patient?

12. W h at would be the preferred antihypertensive in a patient w ith chronic hypertension and recurrent migraines?

TIA NEUROLOGY

3 Question Warm-Up 1.

A lesion to which area o f the brain is responsible for each o f the following clinical scenarios? ° Agraphia and acalculia ° Hemispatial neglect syndrome ° Personality changes ° Coma

2.

M eningitis is diagnosed in a neonate. W h a t are the m ost likely organisms, and w hat is the empiric treatment?

3.

W h a t should always be done prior to LP?

Transient ischemic attack (T IA ) 4. W h at is the anticoagulant of choice in a patient with a history of stroke or T IA given each of the following scenarios? ° First TIA ° TIA/stroke due to atrial fibrillation ° TIA/stroke + coronary artery disease ° Repeat TIA/stroke while on aspirin

Carotid artery disease 5. W h at are the classic signs and symptoms of carotid artery stenosis? o

° Transient ischemic attacks (TIAs) ° Reversible ischemic neurologic deficits lasting up to 3 days ° Amaurosis fugax (transient unilateral blindness) ° Cerebrovascular accidents (CVAs) o ______________________ are not caused by carotid artery stenosis

6. W h at are the surgical indications for carotid endarterectomy? ° Symptomatic patients with narrowing o f______________________ ° Symptomatic men with narrowing o f______________________ ° Asymptomatic patients with narrowing of______________________ provided the fife expectancy is > 5 years and the surgeon has a perioperative complication rate of < 3%

I [ 'I ]

W hat are the important nonsurgical treatments for carotid artery stenosis? ° H TN control to < 140/90 ° Dyslipidemia control to LDL < 100 mg/dL, HD L > 35 mg/dL, triglycerides < 200 mg/dL Lipid control with statins reduces stroke while other lipid-lowering drugs do not Niacin reduces carotid artery mtima thickness Al IA diet

° ° ° 6 ° °

DM control to fasting glucose < 126 mg/dL and H bA lC < 7% Smoking avoidance, consider varenicline (Chantix) Increased physical activity to at least 30-60 minutes 4 times weekly Red wine consumption up to 2 drinks daily is beneficial. Avoidance of heavy drinking Evaluation for CAD and PAD ________________ (if history of TIA/stroke while on aspirin —» use Aggrenox or clopidogrel instead)

End o f Session Quiz 8.

List some major signs and symptoms o f a T IA .

9.

W h at anticoagulant would you give a patient who has just had his/her first TIA ? I f the patient had another T IA while on ASA, what would you add?

10.

H ow long must a focal neurologic deficit last to qualify as a stroke?

STROKE

3 Question Warm-Up 1.

A child presents to the E R w ith mental status changes, hypoglycemia and lesions suggestive o f chickenpox. W h a t is the most likely diagnosis?

2.

W h a t type o f headache causes unilateral, severe periorbital headache w ith tearing?

3.

C S F analysis shows low glucose, elevated neutrophils and Gram-positive diplococci. W h a t is the diagnosis?

4. W hat are the five main lacunar syndromes that may arise from a lacunar infarct?

Lacunar Syndrome

Description Weakness of the face, arm and leg on one side of the body + absent sensory or cortical signs (aphasia, neglect, apraxia, hemianopsia) (M ost common, about 50% of lacunar strokes) Sensory defect (numbness) of the face, arm and leg on one side of the body + absent motor or cortical signs

Ipsilateral weakness and limb ataxia out o f proportion to the motor defect, possible gait deviation to the affected side + absent cortical signs Weakness and numbness of the face, arm and leg on one side of the body + absent cortical signs

Facial weakness, dysarthria, dysphagia and slight weakness and clumsiness o f one hand + absent sensory or cortical signs (Least common)

End o f Session Quiz 5.

In what timeframe must thrombolytic therapy be instituted in cases of ischemic stroke?

6.

W h at is the principal cause o f a lacunar infarct?

7.

A patient with a D V T develops a stroke. W h a t study would most likely identify the underlying etiology o f the stroke?

8.

W hat neurologic defects would be seen with an infarction o f the following arteries?

Anterior cerebral artery

Middle cerebral artery

Posterior cerebral artery

Lacunar arteries

Basilar artery

HEM ORRHAGE

HEMORRHAGE

3 Question Warm-Up 1.

W h at are the four most common sequelae of meningitis in children?

2.

W h a t does a ring-enhancing brain lesion on C T in a patient w ith seizures suggest?

3.

A 30-year-old w om an is in the office with a complaint o f facial pain. She describes th at whenever her face is lightly touched she experiences incredible electrical-like pain. W h a t is the first-line treatm ent for this condition?

4. W hat is the treatm ent for a subarachnoid hemorrhage? ° Discontinue all anticoagulants and reverse any anticoagulation ° Systolic blood pressure < only if cognitive function is intact (adequate cerebral perfusion pressure) until the aneurysm is clipped or coiled to prevent rebleeding. If the cerebral perfusion pressure is not adequate, then lowering the BP will increase the risk of infarction. - _____________________________ preferred A void_____________________________ which can increase intracranial pressure ° ___________________________(a CCB) to prevent vasospasm ° Prevent physiologic derangements that may worsen brain injury - Avoid hypoxia and hyperglycemia Maintain a normal pH, euvolemia and normothermia ° Phenytoin for seizure prophylaxis is controversial and generally avoided due to poorer outcomes. ° Ventriculostomy to monitor intracranial pressure in select patients ° Surgical__________________________ into aneurysm

[ 15]

End of Session Quiz 5.

In which scenario is seizure prophylaxis with anticonvulsants recommended: parenchymal hemorrhage or subarachnoid hemorrhage (SAH)?

6.

W h at are 3 feared complications o f parenchymal hemorrhage?

7.

W h at are the most common causes o f an epidural hematoma and a subdural hematoma?

8.

I f you suspect a patient has an epidural or subdural hematoma, should you perform a L P to confirm the diagnosis?

9.

W h a t is the treatment for subarachnoid hemorrhage?

SEIZURES

3 Question Warm-Up 1.

C om paring ischemic stroke, intracerebral hemorrhage and subarachnoid hemorrhage, w hat are the different BP goals and BP medications?

BP goal

BP medications

Ischemic stroke Intracerebral hemorrhage Subarachnoid hemorrhage

2.

A 40-year-old m an presents w ith daily, unilateral retroorbital headaches associated w ith rhinorrhea and lacrimation. W h a t is the diagnosis, and what treatm ent w ill rapidly abort his headache?

3.

You suspect an A ID S patient may have meningitis. W h a t specific C SF preparation should be ordered in addition to the usual C S F analysis, G ram stain and culture?

4. W hich medications or medication withdrawals are known for causing seizures?

5.

W h at seizure medications are used for prevention of each of the following types of recurrent seizures?

Seizure type

Initial treatment of choice

Grand mal (tonic-clonic)

valproate, carbamazepine, phenytoin, lamotrigine, topiramate

Partial > valproate, topiramate Absence Myoclonic

6.

W hich seizure medication matches each of the following descriptions? ° ° ° °

Gingival hyperplasia Drag of choice for absence seizures Second choice for absence seizures Drug of choice for trigeminal neuralgia

SEIZURES

7.

Which drugs are known for causing Stevens-Johnson syndrome?

8. Which drugs are known for inducing the cytochrome P450 system, thereby speeding up the metabolism of other drugs such as O C P s and warfarin?

End o f Session Quiz 9.

W h at type o f seizure fits each description? Focal sensory or motor deficit with NO loss of consciousness Focal sensory or motor deficit with impaired consciousness (commonly localized to temporal lobe on EEG) Involves both hemispheres of brain with a pattern of neuromuscular activation: tonic, clonic, tonicclonic, myoclonic, or atonic. Loss of consciousness present with postictal period Characterized by a brief (few seconds) impairment of consciousness. No postictal period. Spike-andwave pattern on EEG.

10.

W h at are the most common causes o f seizures in children aged 2-10 years?

11.

W hat are the most common causes of seizures in young adults (18-35 years)?

12.

13.

14.

Although benzodiazepines are used to end a seizure in status epilepticus, what is of more concern in the initial treatment?

W h a t is the drug o f choice for absence seizures?

A 45-year-old man is brought to the E R for new-onset status epilepticus. W h at are some of the components o f the work-up to determine the cause o f epilepsy?

DEGENERATIVE DISORDERS PART I

3 Question Warm-Up 1.

W h a t two side effects should a physician be aware o f when using atypical antipsychotics?

2.

W h a t are the signs and symptoms o f a T C A overdose? H ow is it managed?

3.

A patient is brought into the E R w ith headache, vomiting, neck pain and fever. There is progressive muscle weakness, but sensation is intact. C S F analysis shows normal glucose and protein, but the C SF lymphocyte count is high. W h a t is the diagnosis?

4. W hat medications are used in the management of Parkinsonian symptoms? ° Levodopa + carbidopa ° ______________________(MAO-B inhibitor) used in early disease and has neuroprotective effects. ° Dopamine agonists: - ________________________(ergot compound) Non-ergot D3 stimulators —pramipexole, ropinirole, rotigotine (transdermal) - Apomorphine (subQ) —rescue therapy for sudden akinetic episodes ° _____________________to potentiate levodopa: entacapone, tolcapone ° _____________________for tremor: trihexyphenidyl, benztropine ° ________________ to increase dopamine release. Used as short-term monotherapy in mild disease.

W hat are the characteristic features of amyotrophic lateral sclerosis (A LS, Lou Gehrig disease)? ° Weakness but with normal sensation ° Initial presenting symptoms: ________________________ (80%) in hands, fingers, shoulder girdle, lower extremity (foot drop) Oh pelvic girdle ( 20 %)

° Upper motor neuron (UMN) signs and symptoms: movement stiffness, slowness and incoordination; spasticity and hyperreflexia (spastic paralysis); slowed rapid alternating movements; gait disorder ° Bulbar UMN signs and symptoms: dysarthria; dysphagia; pseudobulbar affect with inappropriate laughing, crying or yawning ° Lower UMN signs and symptoms: weakness, gait disorder, reduced reflexes (flaccid paralysis), muscle atrophy and fasciculations ° Cognitive defects: frontotemporal executive dysfunction ° Neuromuscular respiratory failure after months to years (average survival from time of diagnosis is 3-5 years)

W hat are the C s of Huntington chorea? ° CAG repeat disorder on chromosome Cuatro (4) Caudate andputamen atrophy on MRI Acetylcholine decrease G A B A decrease

° Cognitive decline (dementia) ° Choreiform movements ° Cuarenta (40) = age of onset

End o f Session Quiz 7.

W h a t medication is most commonly used to treat Parkinson disease?

8.

W h a t brain lesion is seen in patients w ith Parkinson disease?

9. W h a t will an electromyogram reveal in ALS?

10. W h a t medication is used to treat ALS?

11. W h a t is the life expectancy once a patient is diagnosed w ith ALS?

12. W h at drugs may be used to treat H untington disease?

DEGENERATIVE DISORDERS PART 2

3 Question Warm-Up 1.

W h a t is the pattern o f pain in a migraine? In a tension headache?

2,

W h a t is the treatm ent for nephrogenic diabetes insipidus caused by lithium toxicity?

3.

A patient fell off a 20-foot-ladder and landed on his head. H is wife says although he seemed dazed initially, he recovered quickly and seemed “fine” for 2 or 3 hours, before becoming confused, disoriented and somnolent. W h a t are the diagnosis, the underlying injury and the treatment?

4. W h at are the usual components of a “dementia work-up” ?

5. W h at are the unique features of Lewy body dementia?

6. W hat are the unique features of frontotemporal dementia (Pick disease)?

DEGENERATIVE DISORDERS PART 2

End o f Session Q uiz 7.

A 66-year-old woman with forgetfulness and decreased bilateral parietal lobe activity on P E T scan has what form o f dementia?

8.

W h a t medications are used in the treatment o f Alzheimer disease?

9.

How does one differentiate between vascular dementia and Alzheimer disease?

10.

W h at are two symptoms that should clue you in to the diagnosis o f multiple sclerosis (MS)?

11.

W h at is the most sensitive test for multiple sclerosis?

12.

W h a t medication decreases the frequency o f relapses in patients w ith multiple sclerosis?

13.

W h a t im portant neuronal tract is the first to be compressed and compromised in the case o f syringomyelia?

PERIPHERAL DISORDERS

3 Question Warm-Up 1.

H ow do the features o f acute dystonia differ from tardive dyskinesia?

2.

W h ich medication is used more than any other in the treatm ent o f Parkinson patients?

3.

W h a t is the maxim um am ount o f tim e a TLA. may last?

4. W hat is the classic presentation of Guillain-Barre syndrome (GBS)? ° Symmetric muscle weakness that progresses over days to 4 weeks (usually 2 weeks) Usually beginning in the distal legs but may begin in the arms or facial muscles in 10% of cases __________________________ requiring mechanical ventilation in ___________of cases __________________________ and/or oropharyngeal weakness in __________ which may include bilateral facial muscle paralysis ° Autonomic dysfunction in 70% - usually________________________ ° Absent or depressed deep tendon reflexes ° Little if any change in sensation ° No fever at the onset of symptoms ° GBS may be preceded by: Campylobacterjejuni diarrheal illness (about 20% of cases) HIV infection CM V infection EBV infection - Mycoplasma infection - Other viral infections Immunization (extremely rare)

5.

How is the diagnosis of Guillain-Barre syndrome made in a patient with ascending muscle paralysis? ° Characteristic clinical presentation ° CSF analysis (elevated protein and normal WBCs) ° Electrodiagnostic studies: nerve conduction studies and electromyography (EMG) reveal

PERIPHERAL D ISO RDERS

6. W hat is the prognosis of a patient with Guillain-Barre syndrome? ° ° ° °

7.

Spontaneous regression and complete recovery by 1 year in 80-90% Relapse in 10% Prolonged disease with delayed or incomplete recovery in 5-10% Death despite ICU care in 5%

W hat is the treatment of Guillain-Barre syndrome? ° Hospitalization for respiratory monitoring including vital capacity, BP monitoring, cardiac monitoring (telemetry) and daily abdominal auscultation for ileus ° Mechanical ventilation required in 30% of patients ° ICU monitoring for autonomic dysfunction required in 20% of patients Equally effective at shortening time to independent walking by iSQfb Combining the two offers no additional benefit

° _______________________ are NO T recommended in the treatment of GBS. Previously the mainstay of therapy; new studies show absolutely no benefit.

8. W hat is required to make the diagnosis of Bell’s palsy? Clinical diagnosis: ° Diffuse involvement of the entire facial nerve —>facial muscle paralysis (upper and lower) Rule out Lyme disease b y _________________ : tick bite, heart block, arthritis, vertigo, hearing loss Rule out Otitis media b y__________________ Rule out stroke b y _______________________

° Acute onset (1-2 days) —>progressively worsening weakness for 3 weeks —» recovery within 6 months ° Anything other than the above presentation requires imaging (CT and/or MRI) and screening blood tests to rule out other pathology

| 2 4 ]

W hat is the treatm ent for Bell’s palsy? ° Eye care to prevent corneal trauma Artificial tears hourly while awake Lubricating ointment qHS Patch covering the eye at night ° Glucocorticoids (e.g., prednisone 60mg daily x 1 week) ° +/- Valacyclovir lOOOmg tid x 1 week (acyclovir provides no additional benefit over glucocorticoids)

End of Session Quiz 10.

11.

12.

13.

W h a t test can help make the diagnosis o f myasthenia gravis?

H o w does Lam bert-Eaton syndrome differ from myasthenia gravis (M G ) on history and physical exam?

W h a t are the treatm ent options for benign essential tremor?

A 35-year-old wom an presents w ith ptosis and diplopia that worsens throughout the day. W h a t is the underlying problem?

14.

W h a t is a classic presentation o f Guillain-Barre syndrome?

15.

H ow do you treat Guillain-Barre syndrome?

16.

H o w can Bell’s palsy be easily differentiated from a motor cortex stroke?

NEOPLASMS AND SLEEP s

'

. i■ •-

v ... '■

mw® ® " m

■ ■" $

3 Question Warm-Up 1

W h a t two medications could be used for prophylaxis against meningococcal meningitis?

2.

W h a t two classes o f medication could be used both to treat chronic hypertension and also to prevent recurrent migraines?

3.

H ow do edrophonium, neostigmine and pyridostigmine work in the treatment o f myasthenia gravis?

. W hat are the differences between nightmares and night terrors? ° Nightmares - during REM sleep, patients that appear to wake up are actually awake ° Night terrors - during non-REM sleep, patients that appear awake (and are frightened/ screaming, tachycardic and diaphoretic) are actually not fully awake, difficult to arouse and usually fall right back to sleep after the episode

W hat is required to make the diagnosis of narcolepsy? ° _______________________ (sudden loss of muscle tone) only occurs in narcolepsy and is virtually diagnostic when present ° Other causes of excessive daytime sleepiness are ruled out Overnight polysomnogram (to r/o O SA and periodic limb movement disorder) Rule out sedating medications as a cause ° Multiple Sleep Latency Test - when given 4-5 opportunities to nap every 2 hours, narcolepsy patients fall asleep in less than 8 minutes

W hat is the treatm ent for narcolepsy? ° ° ° °

Avoidance of drugs that cause sleepiness Scheduled naps (once or twice a day for 10-20 minutes) Stimulants - _______________________ isfirst-line Support group attendance

° If cataplexy —* venlafaxine, fluoxetine or atomoxetine

W hat medications are common in the treatm ent of insomnia? W hat makes each one unique? M elatonin

Non-addictive, O T C , vivid dreams, safe for < 3 months

Valerian

O T C herbal remedy, studies show no benefit

Antihistamines (diphenhydramine, doxylamine)

Commonly used by patients first-line, associated with poor sleep quality, not for long-term use, anticholinergic side effects (avoid in the elderly)

Trazodone

Antidepressant, decreases sleep latency, small risk of priapism

TCA s (amitriptyline, doxepin)

Antidepressant, small risk of arrhythmias (obtain EKG prior to use), anticholinergic side effects (avoid in the elderly)

Benzodiazepines: (temazepam, lorazepam, clonazepam, diazepam, chlordiazepoxide)

Addictive, short-term only (< 35 days)

Zolpidem Zaleplon

Act at the benzo receptor, short-term only (< 35 days), rebound insomnia when discontinued

Eszopiclone

May be used long-term

Ramelteon

Non-addictive because it works at melatonin receptors instead of GABA/benzo receptors, avoid if hepatic insufficiency, long-term studies are lacking

NEOPLASMS AND SLEEP

8. Restless Leg Syndrome ° The sensation of unpleasant paresthesias that compels the patient to have voluntary, spontaneous, continuous leg movements that temporarily relieve the sensations. The discomfort worsens at rest, in the evening and/or during sleep. Sensation of “spiders or ants” on/in Ket/calf muscles. ° Usually a primary, idiopathic disorder ° Secondary RLS can result from iron deficiency, end-stage renal disease, diabetic neuropathy, Parkinson disease, pregnancy, rheumatic diseases (RA), varicose veins, caffeine intake. ° Treatment: pramipexole or ropinirole qHS (or levodopa/carbidopa), iron replacement, avoidance of caffeine, clonazepam qHS, gabapentin, opioids

End o f Session Quiz 9.

W h a t is the next step once a brain tum or has been identified on C T or M R I of the head?

10. W h a t are the important characteristics o f neurofibromatosis type 1?

11. W h at is the mechanism o f action o f the preferred medication in the treatment of restless leg syndrome?

12. W h a t E E G waveforms correspond to the different stages o f sleep? ° Awake ° Awake, relaxed, drifting off to sleep ° Stage N1 : • Stage N2 ° Stage N3 • REM 13. Benzodiazepines increase which stage o f sleep at the expense o f what other stages o f sleep?

14. W h a t are the two most common primary brain tumors in adults? W h a t are the 3 most common primary brain tumors in children?

LOSS OF CONSCIOUSNESS

3 Question Warm-Up 1.

W h a t is the treatm ent o f acute dystonia and how does it differ from the treatm ent o f tardive dyskinesia?

2.

W h a t E E G pattern is seen in cases o f absence seizures?

3.

W h a t are the m ost com m on prim ary sources o f metastases to the brain?

Syncope basics Causes: ° Reflex syncope Vasovagal: associated with emotional stress, trauma, pain, sight o f blood, prolonged standing Situational: associated with micturition, defecation, coughing, G l stimulation

° Carotid-sinus hypersensitivity: associated with head turning, shaving, tight collar ° Cardiogenic: associated with exertion, palpitations, chest pain, SOB ° Orthostatic ° Cerebrovascular: associated with prolonged loss of consciousness, seizures, neurologic deficits ° No identifiable cause

Work-up basics: ° Rule out orthostatic hypotension via tilt test on multiple occasions ° Rule out seizure by history and physical More likely seizure: history of seizure, prodrome of deja-vu postictal confusion, tongue lacerations More likely syncope: prodrome of lightheadedness or sweating, history of prolonged standing Nonspecific: brief limb jerking, urine incontinence

° ° ° °

CBC, electrolytes, BUN/Cr, glucose Assess volume status Pulse oximetry and ECG Evaluation of medications

° In patients over 40 (without history of carotid disease or carotid bruits), rule out carotid sinus hypersensitivity with carotid sinus massage while on telemetry monitor

Also consider: ° Serial cardiac enzymes and ECGs x3 Especially if: > 45 years old, diabetes mellitus, smoker; prior myocardial infarction or > 2-3 risk factors

° Echocardiogram Especially if: murmur exertional syncope or history of heart disease

° Cardiac stress test ° Bilateral carotid duplex Especially if: > 65 years old, C A D , PVD or bruit

° 24-hour Holter monitor Especially if: abnormal ECG, palpitations, heart disease or family history of sudden death

° C T head without contrast and EEG Especially: if neurologic symptoms, new seizure, headache

W hat is the differential diagnosis for a patient presenting to the emergency room for loss of consciousness? A EIO U TIPS:

W hat should you think about for initial empiric therapy in a patient coming into the emergency room with loss of consciousness?

W hy is thiamine given in a glucose infusion to alcoholics with hypoglycemia? Glucose administration in the absence of thiamine can theoretically exacerbate damage to the mammillary bodies and w orsen______________________________.

End of Session Quiz 8.

W h ich cause o f syncope is consistent w ith each o f these historical items or physical exam findings? W hile shaving W hile singing in a choir concert W ith a positive tilt test after taking blood pressure medication W ith prolonged loss o f consciousness Preceded by palpitations Type 1 diabetic interrupted while eating

9.

10.

11.

12.

13.

W h a t test is used to confirm orthostatic hypotension?

W h a t measurements indicate a positive tilt test?

In an intact brainstem, the patient’s eye should move in which direction w ith ice water infusion into an ear canal?

W h a t are the elbows doing in decorticate posturing?

A patient is brought into the emergency room w ith loss o f consciousness. W h a t should be administered before empiric glucose infusion?

PEDI NEURO

i

3 Question Warm-Up 1.

W hat drugs when combined with SSRIs are known for causing serotonin syndrome?

2.

W hich commonly used antidepressant should be avoided in patients at risk for seizure?

3.

A 60-year-old patient presents w ith an acute onset o f broken speech. W h at type o f aphasia is this? W h a t lobe and vascular distribution have been affected? W h at is the first step in the workup?

Arnold-Chiari Malformation ° Downward displacement of the cerebellar tonsils and medulla through the foramen magnum ° Type I is the most common type and is often asymptomatic. Manifestations may include headaches and/or cerebellar symptoms. ° Type II (of IV severity) is usually accompanied by other neurologic anomalies.

W hat other neurologic anomalies are associated with an Arnold-Chiari malformation?

W hat are some of the possible presenting features of cerebral palsy? ° Spastic features - spastic paresis of any or all limbs, clonus present ° Athetosis features - slow, writhing movements in distal muscles ° Choreiform features - rapid, irregular, unpredictable contractions of muscles in face or extremities ° Dystonic features - uncontrollable jerking, writhing or posturing Infants have persistence of primitive reflexes, involuntary grimacing, tendency to drool and delayed psychomotor development. ° Ataxia - difficulty coordinating purposeful movements ° Atonic features - severe hypotonia present at birth with no future ability to stand or walk ° Neonates may show signs of encephalopathy including lethargy, decreased spontaneous movement, hypotonia and suppressed primitive reflexes. ° Associated disorders: mental retardation, epilepsy, sensory impairment (speech, hearing, vision)

TJ m

End of Session Quiz 7.

W h a t is the preferred treatm ent for febrile seizures?

8.

W hich element o f the quad/triple screen is abnormal in cases o f neural tube defect?

9.

W h a t neural tube defect matches each o f the following descriptions? Incomplete closure of the dorsal vertebral arches, often at the lumbosacral junction Condition where the above defect is severe enough for there to be herniation of the meninges A more severe defect in which the spinal cord and meninges have herniated through Failure o f closure o f the anterior portion of the neural tube resulting in lack o f forebrain, meninges and parts o f the skull

10.

W h a t is the definitive treatm ent for persistent hydrocephalus?

II.

Retinoblastoma can be detected from w hat part o f the physical exam?

12.

W h a t is the next step when a retinoblastoma is suspected on PE?

13.

W h a t does cerebral palsy look like in a neonate?

14.

W h a t medications are often used to treat spasticity in cerebral palsy?

g z m C

70 O

OPHTHALMOLOGY PART I

3 Question Warm-Up ]

W h a t vitamin supplement is recommended to all sexually active women of childbearing age? W h a t developmental defect is this preventing?

2.

W h at is the diagnosis o f a patient who has periods o f mood disturbances while psychotic as well as periods o f psychosis w ith normal affect?

3.

A n elderly patient presents to the E R w ith a headache and a dilated right pupil. D uring the history, she reports that she fell at home 5 days ago. W h at is the most likely diagnosis?

Describe what light reflexes will be seen in both eyes if the right optic nerve is damaged prior to the pretectal nucleus (A K A afferent defect). ° No constriction of either the left or right pupil when light is shone in the right eye ° Both pupils constrict if the light is shone in the left eye

Describe what light reflexes will be seen in both eyes if the right oculomotor nerve is damaged (A K A efferent defect). ° Right pupil will not respond to light shone in either the right or left eye ° Left pupil will constrict when a light is shone in either eye

W hat is amblyopia and what are the signs/symptoms? ° Decreased vision due to a disruption in the normal development of vision usually from strabismus, cataracts or refractive error prior to age 10 ° Possible presentations: esotropia (inward deviation), exotropia (outward deviation), diplopia and/or refractive error not correctable with lenses

W hat is the most common cause of blindness in the following populations of adults in the US? ° Over age 55 ° Under age 55 ° Blacks of any age

Etiology

Type of Discharge

Bacterial

Purulent, copious, 24hrs a day

Viral (adenovirus)

Watery, eyelid may be sealed in am

May also have fever, URI, LAD, pharyngitis

Allergic

Bilateral, watery eyelid may be sealed in am

Pruritus, other allergy symptoms

Other Features

OPHTHALM OLOGY PART I

8. W hat are the distinguishing features of bacterial, viral and allergic conjunctivitis?

9. W hat is the m ost likely cause of conjunctivitis appearing in the first 24 hours of life?

10. W hat causes of red eye most closely match each of the following statements? ° ° ° ° ° ° °

May indicate a collagen-vascular disorder Potential serious complication of corneal ulceration Colored halos Itching eye Preauricular lymph node enlargement “Dry eyes” Shallow anterior chamber

11. W hat are the classic features that distinguish orbital cellulitis from periorbital cellulitis?

12. W hat are the distinctions between a chalazion, a hordeolum and anterior blepharitis? W h at is the treatm ent for each?

Description

Treatment

Chalazion

Inflammation of internal Meibomian sebaceous glands (eyelid swelling)

Usually self-limiting but can be treated w ith surgical excision and/ or intralesional steroid injection

Hordeolum (stye)

Infection of external sebaceous glands of Zeiss or M ol (tender, red swelling at the lid margin)

° H ot compress 3-4 times a day for 10-15 minutes ° I f unresolved in 48 hours, then I8cD ° +/- Antibiotic ointment q3 hours

Anterior blepharitis

Infection o f eyelids and lashes secondary to seborrhea (red, swollen lid margins + dandruff on lashes)

o W ash lid margins daily with shampoo ° Remove scales daily with cotton ball ° Antibiotic ointment qd to lid margins

[ 35]

End of Session Quiz 13.

W h at would be the visual field defect for a lesion o f the optic tract?

14.

W h at are the symptoms o f H om er syndrome? W h a t is the classic cause?

15.

Does strabismus cause amblyopia or does amblyopia cause strabismus?

16.

O n morning O B /G Y N rounds, a very concerned new mother asks you about her 12-hour-old newborn’s red eyes. W h at is the likely reason for the baby’s red eyes?

17.

W h a t is the next step in the management o f a patient who has sustained a chemical burn injury to the eye?

18.

W h a t is the easiest way to distinguish a hordeolum from a chalazion?

19.

W h a t im portant diseases are associated with uveitis?

OPHTHALMOLOGY PART 2

0 "O 1 H X >

3 Question Warm-Up 1.

W h a t C S F findings would you see in a case o f subarachnoid hemorrhage?

2.

A young child has loss o f the red light reflex. W h a t is the diagnosis?

3.

W h a t is considered first-line pharmacotherapy for depression?

i3 O i— o o -< T3 > 73

4. W h at is the classic presentation of a patient who has a cataract? ° Painless progressive decrease in vision manifested by difficulty driving at night, reading road signs or reading fine print ° Usually bilateral, but often unilateral ° Nearsightedness is often an early manifestation ° Possible disabling glare in bright sunlight or from oncoming headlights (more likely with steroid-induced cataracts)

5. W h at is the treatm ent for acute angle-dosure glaucoma? Initial medications: ° Pressure-lowering eye drop regimen: 1 drop each one minute apart of 0.5% timolol (Timoptic), 1% apraclonidine (Iopidine) and 2% pilocarpine (Isopto Carpine) ° Acetazolamide 250mg two tabs once ° If refractory to above combination: given IV once diagnosis confirmed by ophthalmologist

Surgical intervention: ° Laser peripheral iridotomy - tiny hole is made in the periphery of the iris so that aqueous humor can flow into the anterior chamber

[ 37]

W hat is the treatment for a corneal abrasion? ° Thorough eye exam with removal of any foreign body by irrigation ° Topical antibiotics QID continued 3-5 days or until the eye is symptom free for 24 hours (ointment > drops) - erythromycin, sulfacetamide, ciprofloxacin or ofloxacin ° OTC lubricant (Refresh PM, Lacri-lube) as needed up to hourly ° Pressure patching is optional for___________ , andis____________ if a foreign body is present. There is no role for pirate patching. ° Pain control with systemic opioids (e.g., Tylenol #3) or ophthalmic NSAIDs (e.g., diclofenac, ketorolac) ° NEVER prescribe a topical anesthetic (OK to use during the initial exam) or topical steroids! ° 24hr follow-up for contact lens abrasion, abrasion >3mm or abrasion with diminished vision.

W hat disease would you suspect in a 35-year-old woman with new-onset rapid loss of vision and pain when moving the eye? How would you treat this?

W hat eye abnormalities might be seen in a patient with vitamin A deficiency? ° Night blindness or complete blindness ° Xerophthalmia ° Bitot spots (areas of abnormal squamous cell proliferation and keratinization of the conjunctiva)

End of Session Quiz 9.

W h a t are some major clinical features o f acute angle-closure glaucoma?

10.

W h a t is the major exam finding in open-angle glaucoma?

11.

W h a t is the treatm ent for closed-angle glaucoma?

12.

W ith which disorders m ight you see a cherry-red spot on the macula?

13.

14.

W h a t is the treatm ent for macular degeneration (M D)? W h a t is the treatment for retinal detachment (RD)?

W h a t is the differential diagnosis for dislocation o f the lens o f the eye?

AUDIOVESTIBULAR DISORDERS

3 Question Warm-Up 1.

W hat are the 5 main lacunar syndromes that may arise from a lacunar infarct?

2.

W h a t is the typical complaint o f a patient w ith retinal detachment?

3.

W h a t is the treatment o f choice o f mania w ith psychosis?

When is observation without antibiotics appropriate for a child with acute otitis media? According to 2013 AAP/AAFP guidelines, you may refrain from antibiotics and simply observe if: ° Age 6 months to 2 years + unilateral AOM without otorrhea + mild illness + appropriate follow-up available + antibiotics can be started promptly if symptoms worsen ° Age > 2 years + unilateral or bilateral AOM without otorrhea + mild illness + appropriate follow-up available + antibiotics can be started promptly if symptoms worsen ° In either case, the decision to observe without antibiotics should be made jointly between provider and parent/caregiver(s) ° Antibiotics should be started if improvement is not noted in 48-72 hours

W hat are the classic signs and symptoms of bullous myringitis? Bullous myringitis is a bullous/vesicular inflammation of the tympanic membrane that may occur in association with acute otitis media. It typically manifests as follows: » More painful than; usual acute otitis media °

Otoscopy: large, reddish vesicles on the TM

W hat is the treatment for bullous myringitis? ° ____________________ is a common organism —> treat with oral___________________ ° Topical analgesics

7.

W hat are the diagnostic features of mastoiditis? ° Symptoms occur days-weeks after developing acute otitis media ° Erythema, edema, tenderness behind the ear ° External ear displaced ° Diagnosis made from C T scan of the mastoid process

8. W h at are the distinguishing characteristics of acute labyrinthitis? ° Acute onset of vertigo, nausea, vomiting and nystagmus - Auditory function preserved = vestibular neuritis Hearing loss + above symptoms = labyrinthitis ° Single episode that lasts days to weeks (usually not longer than 2 weeks) ° Preceded by a viral URI ° Nystagmus: horizontal, suppressed with visual fixation and has a fast phase away from the affected side 0 Abnormal head thrust test: W hen examiner rapidly turns the patient’s head to the affected side, the patient is unable to maintain visual fixation ° Gait instability, but preserved ambulation ° Absence of focal neurologic defects

9. W h at is the treatm ent for acute labyrinthitis (vestibular neuritis)? ° Typically subsides spontaneously within weeks ° Corticosteroid taper shown to improve recovery ° Symptomatic treatment only indicated for the first 48 hours of illness E.g., scopolamine patch, meclizine, metodopramide or promethazine Long-term recovery is theoretically delayed if used long-term ° Vestibular rehabilitation exercises ° M R I if > 60 years of age, headache, focal neuro signs, vascular risk factors or sustained vertigo inconsistent with acute labyrinthitis (vestibular neuritis)

10. W h at is the m ost com m on cause of conductive hearing loss in adults? W hat is the m ost com m on cause of sensorineural hearing loss in adults? ° Conductive —otosclerosis ° Sensorineural —presbycusis

11. Cholesteatom a: ° Overgrowth of desquamated keratin debris within the middle ear space that may eventually erode the ossicular chain and external auditory canal ° Causes: negative middle ear pressure (chronic retraction pocket) from eustachian tube dysfunction or direct growth of epithelium through a T M perforation ° Commonly associated with chronic middle ear infection ° PE: grayish-white “pearly” lesion behind or involving the TM , conductive hearing loss, vertigo ° Treatment: surgical removal usually involving tympanomastoidectomy and reconstruction of the ossicular chain

AUDIOVESTIBULAR DISORDERS

12. W hat is the treatment for Ramsay Hunt syndrome? Ramsay Hunt syndrome is herpes zoster oticus and is treated as follows: ° Narcotic analgesia for pain relief ° Oral steroids to decrease inflammation ° Antiviral therapy with valacyclovir (highest efficacy), famciclovir or acyclovir

End of Session Quiz 13.

W hat are the important characteristics seen on otoscopic exam of a patient with otitis media?

14.

W h at is the underlying cause o f benign paroxysmal positional vertigo (BPPV)?

15.

16.

Explain how the W eber test can help distinguish conductive hearing loss from sensorineural hearing loss.

W hat are the major characteristics o f Meniere’s disease?

Psychiatry 1

D epression

2

A ntidepressants

3

O th e r M o o d D isorders

4

A n x iety D isorders

5

Psychotic D isorders

6

P ersonality D isorders

7

Substance A buse p a rt 1

8

Substance A buse p a rt 2

9

O th e r Psych D isorders

10 D eliriu m and D em entia 11 Pedi Psych

DEPRESSION

3 Question Warm-Up 1.

W h a t is the empiric treatm ent for a brain abscess?

2.

W h a t is the treatm ent for an epidural hematoma or subdural hematoma?

3.

W h a t is the typical pseudotumor cerebri patient?

4. A t what point does grief/bereavement become pathological? G rief becomes pathological when any of the following are found: ° Depression criteria met for at least 2 weeks after the first 2 months following the loss ° Generalized feelings o f______________________________________________ O ° Distressing feelings do not diminish in intensity by___________________ ° Inability to move on, trust others and reengage in life by___________________

5. W h at medical conditions can cause severe depression?

6,

W h at medications are known for causing symptoms of depression? ° Sedatives: alcohol, benzos, antihistamines ° Stimulant withdrawal o _______________________ (antihypertensive often used for hypertension in pregnancy) ° First-generation antipsychotics (such as haloperidol) ° Anti-nausea drugs including metoclopramide and prochlorperazine O ° Insufficient thyroid replacement -» hypothyroidism o ________________________(used in viral hepatitis treatment)

DEPRESSION

7.

W h at are the sym ptom s of m ajor depression with atypical features? W h at medications w ork well for this?

8. W hat is the first-line treatment for major depression with seasonal pattern?

End o f Session Quiz 9.

A 36-year-old male smoker says he has been thinking a lot about quitting, and wants to talk to you about ways to go about it. A t what stage o f change in overcoming his smoking habit is this man?

10.

W h a t neurotransmitter derangements are seen in patients with depression?

II.

List the symptoms o f depression using the mnemonic SIG E CAPS:

12. Diagnosis of major depressive disorder (MDD) requires five of the above symptoms, including depressed mood or anhedonia, that must last how long?

ANTIDEPRESSANTS

3 Question Warm-Up

4.

1.

A patient presents w ith hyperphagia, hyperorality and hypersexuality. W h at is the diagnosis, and w hat brain lesion causes this?

2.

W h a t are the m ost com m on causes o f seizures in young adults (18-35 years)?

3.

W h a t is the treatm ent for cluster headaches?

Categorize each of the following antidepressants: nortriptyline, bupropion, mirtazapine,fluvoxamine, doxepin,phenelzine, fluoxetine, imipramine, amitriptyline, milnacipran, sertraline, venlafaxine, paroxetine, tranylcypromine, duloxetine, citalopram, trazodone, escitalopram, desvenlafaxine, nefazodone

SSRI -

TC A -

MAOI -

NDRI -

SNR.I -

Tetracyclic -

W h at are the sym ptom s of m ajor depression with atypical features? W h at medications w ork well for this?

W hat is the first-line treatment for major depression with seasonal pattern?

End o f Session Q uiz 9.

A 36-year-old male smoker says he has been thinking a lot about quitting, and wants to talk to you about ways to go about it. A t what stage o f change in overcoming his smoking habit is this man?

10.

W h a t neurotransmitter derangements are seen in patients with depression?

11.

List the symptoms o f depression using the mnemonic SIG E CAPS:

12. Diagnosis of major depressive disorder (MDD) requires five of the above symptoms, including depressed mood or anhedonia, that must last how long?

ANTIDEPRESSANTS

S > I z I —

a m

I

TJ 73 m on on >

z

3 Question Warm-Up

4.

1.

A patient presents w ith hyperphagia, hyperorality and hypersexuality. W h a t is the diagnosis, and what brain lesion causes this?

2.

W h a t are the m ost common causes o f seizures in young adults (18-35 years)?

3.

W h a t is the treatm ent for cluster headaches?

Categorize each of the following antidepressants: nortriptyline, bupropion, mirtazapine,fluvoxamine, doxepin, phenelzine, fluoxetine, imipramine, amitriptyline, milnacipran, sertraline, venlafaxine, paroxetine, tranylcypromine, duloxetine, citalopram, trazodone, escitalopram, desvenlafaxine, nefazodone SSRI -

TCA -

M AOI -

N D RI -

SN RI -

Tetracyclic -

Atypical -

[ 47 ]

SSRIs 5. Which drugs should not be taken with SSR Is because of the risk of serotonin syndrome?

6. W hat are the characteristic features of serotonin syndrome? ° (anxiety, agitation, delirium, restlessness, disorientation) o ' ___________________________(diaphoresis, tachycardia, hyperthermia, hypertension, vomiting, diarrhea) ° _____________________________ __ (tremor, muscle rigidity, myoclonus, hyperreflexia) ________________________ - slow, continuous, horizontal eye movements Spontaneous or inducible clonus Sabinski signs bilaterally

7. W hat is the treatment for serotonin syndrome? ° Discontinue all serotonergic agents —* symptoms usually resolve in 24 hours ° Supportive care to normalize vital signs Oxygen, IV fluids,icardiac monitoring If medical treatment for tachycardia or hypertension is needed, use short-acting agents (e.g., esmolol or nitroprusside)

° Sedation with - _________________________ ° If temp > 41.1° C —>sedation, paralysis and ET tube —> mechanical cooling (e.g., ice, cooling blankets, misting fans) Paralysis should relieve the hyperthermia which is caused by muscle activity There is no benefit in using antipyretics in this scenario

If agitation despite benzodiazepine —>__________________________ ° After resolution of symptoms, assess need to resume serotonergic agent

8. W hat are the symptoms of serotonin withdrawal syndrome? W hich SSR Is are well known for causing this when stopped abruptly? ° SSRI discontinuation syndrome is characterized by dizziness, nausea, fatigue, muscle aches, chills, anxiety and irritability that begins within days of abrupt discontinuation and dissipates over 1-2 weeks. ° Worst offenders:_____________ ____________________

TCAs 9.

W hat evaluation should take place prior to the initiation of T C A s in children? Because TCA s can cause arrhythmias, the following should be performed: ° Screen patient’s history for______________________________________________ 0 Screen family history for sudden death prior to age 40, long Q T syndrome, arrhythmias and hypertrophic cardiomyopathy ° __________________________ prior to initiation and again when medication is optimized

10. W hat are the symptoms of overdose with tricyclic antidepressants (TCA s)? ° : tachycardia, hypotension, conduction abnormalities ° __________________________ : sedation, obtundation, coma, seizures ° ________________ : mydriasis, xerostomia, ileus, urinary retention

° ABCs - Airway, Breathing, Circulation ° Activated charcoal lg/kg up to 50kg (unless ileus is present) ° Continuous cardiac monitoring for at least 6 hours —* if no problems, then clear for psych eval ° Frequent neuro checks o Lab/Studies: TC A level, Chem 7, EKG

AN TIDEPRESSAN TS

II. How is T C A overdose managed?

° If ingestion < 2hrs ago —* gastric lavage ° If hypotension —* IVF (LR or NS) —* if ineffective, then norepinephrine ° If QRS > 100 msec -►trial__________________________ then infusion if effective ° If seizures —>_______ , barbiturates and/or propofol (but n o t____________ . which is ineffective against toxin-induced seizures)

O ther Antidepressants 12. W hat food substances should be avoided when taking M AOIs in order to avoid a tyram ine-induced hypertensive crisis? Foods that are spoiled, pickled, aged, smoked, fermented or marinated contain tyramine. The following foods should be avoided because of sufficient quantities of tyramine to be problematic while on MAOIs: ° Fermented cheeses (cream cheese and cottage cheese are OK) ° Smoked or aged meats (sausage, bologna, pepperoni, salami, smoked or pickled fish) ° Chianti, most beers and wines (especially over 120mL) ° Soy sauce, shrimp paste, miso soup ° Sauerkraut, avocados, ripe bananas, Fava beans ° Brewer’s yeast and yeast extracts (yeast used in baking is OK)

13. In what group of patients is bupropion (W ellbutrin) contraindicated? Because of an increased risk of lowering the seizure threshold, bupropion should be avoided in patients with:

14. W h at are the indications for electroconvulsive therapy (E C T )? ° Severe debilitating depression refractory to antidepressants ° ° ° ° °

Psychotic depression Severe suicidality Depression with catatonic stupor Depression with food refusal leading to nutritional compromise Situations where a rapid antidepressant response is required (e.g., pregnancy)

° Previous good response to EC T ° Medical condition preventing the use of antidepressants (e.g., elderly patients) ° Bipolar disorder/mania ° Schizophrenia/psychosis (esp. catatonic)

[ 49]

End o f Session Quiz 15.

H ow long must a patient expect to take an SSRI before they see improvement in their depression?

16.

In T C A overdose, what can be used to correct the prolonged QRS interval and possible seizures?

17. W hich antidepressant matches the following statements? ° SE: priapism ° Lowers the seizure threshold and can be used for smoking cessation ° Appetite stimulant that is likely to result in weight gain ° Works well with SSRIs and increases REM sleep " Can be used for bedwetting in children 18. W h at are the symptoms of serotonin syndrome?

OTHER MOOD DISORDERS

3 Question Warm-Up 1.

C T scan o f the head shows a crescent-shaped lesion. W h a t event most likely caused this lesion?

2.

W h a t is the initial radiologic study in a patient w ith T IA /stroke symptoms? W h a t radiologic studies need to be performed later to evaluate the underlying cause o f the TIA /stroke?

3.

A 2-year-old develops lethargy, hypoglycemia and abnormal LFTs approximately one week after being seen for a febrile U R I. W h a t is the most likely diagnosis?

Bipolar Disorder 4.

How is depression managed in patients with bipolar disorder? ° Mild depression —>___________________________________ ° Moderate depression —> add a second mood stabilizer (lamotrigine) or add an atypical antipsychotic (olanzapine, quetiapine, risperidone) ° Severe depression —* consider________________

5. W h at are the potential side effects of lithium use in the treatm ent of bipolar disorder? ° CNS effects (depression, tremor, cognitive dulling) o ________________________ _ (hyperthyroidism, hypothyroidism, euthyroid goiter) o __________________________ (nausea, vomiting, diarrhea, metallic taste changes, weight gain) ° Nephrogenic diabetes insipidus (thirst, polydipsia, polyuria)

OTHER HOOD DISORDERS

6. W hat is the treatment for nephrogenic diabetes insipidus caused by lithium toxicity?

Adjustment Disorder 7. W hat are the diagnostic criteria for adjustment disorder? ° Clinically significant emotional or behavioral reaction causing marked distress or impairment in social or occupational functioning ° Symptoms develop in response to an identifiable psychosocial stressor (e.g., divorce, failure at school, peer problems) other than bereavement ° Symptoms begin within of the stressor ° Symptoms disappear within____________ of the disappearance of the stressor If the stressor is chronic (e.g., ongoing parental conflict) then the disorder may last longer than 6 months and is termed chronic

8. W hat is the difference between major depressive disorder and adjustment disorder with depressed mood?

End o f Session Quiz 9.

W hat is the drug o f choice in the treatment o f bipolar disorder in a patient with renal failure?

10. W h at is the most problematic congenital malformation associated with maternal lithium use?

11. How long must hypomania symptoms or depressive symptoms be present to diagnose cyclothymia?

ANXIETY DISORDERS

3 Question Warm-Up 1.

H ow does one differentiate between subarachnoid hemorrhage and a traumatic lumbar puncture as a cause o f bloody cerebrospinal fluid?

2.

W h ich spinal tracts convey the following information? ° Touch, vibration and pressure sensation ° Voluntary motor command from motor cortex to body ° Voluntary motor command from motor cortex to head/neck ° Pain and temperature sensation ° Important for postural adjustments and head movements

3.

A patient is recovering in the IC U after suffering a subdural hematoma that occurred because o f a motor vehicle collision. The neurosurgery team performed a craniotomy and drain placement to evaluate the clot. For the past few days the drainage in the collection bulb was serous. Now, however, the drainage is thick and yellow. A long w ith this, the patient’s neurological exam has deteriorated. W h a t is the likely cause o f this clinical picture?

4. How is acute stress disorder different than post-traumatic stress disorder (P TSD )?

5.

W hat are the treatm ent options for PTSD ? ° ° ° °

Psychotherapy including behavioral (exposure) therapy and cognitive therapy _______________ - first line Other antidepressants - TCAs (imipramine, amitriptyline), MAOIs Benzodiazepines should be avoided in PTSD due to lack of efficacy and potential for abuse a _______________ (carbamazepine or valproate) improve impulsive behavior, arousal and flashbacks o _______________ (prazosin) improves nightmares and sleep disturbance ° Atypical antipsychotics if refractory to other therapies

End of Session Quiz 6.

How long must anxiety and worry symptoms be present to diagnose a patient with generalized anxiety disorder?

7.

W h at are the treatment options for generalized anxiety disorder?

8.

How long must the symptoms o f P T S D be present to make the diagnosis?

9.

W h at are the treatment options for PTSD ?

PSYCHOTIC DISORDERS

3 Question Warm-Up 1.

W h a t complication may arise from perform ing L P in a patient w ith elevated intracranial pressure?

2.

W h a t are the symptoms o f a basilar artery stroke?

3.

Bereavement and adjustment disorder share similar tim e tables in a patient’s symptomatology. W h a t are the key differences between them?

n x O H n g on

o73 o

Schizophrenia 4. W h at are the diagnostic criteria for schizophrenia? ° At least 2 o f the following during a 1-month period (including at least 1 of the first 3): Delusions (irrational belief that cannot be changed by rational argument) Hallucinations (most common type is auditory) Disorganized speech (e.g., frequent derailment or incoherence) Grossly disorganized or catatonic behavior - Negative symptoms (e.g., flat affect, poverty of speech, lack of emotional reactivity) ° Social/occupational dysfunction ° Duration of at least 6 months

5. W h at is the difference between each of the following disorders? Schizotypal personality disorder Schizophrenia Schizoaffective Schizoid personality disorder Schizophreniform Brief psychotic disorder

[ 55]

D ISO RDERS PSYCHOTIC

6.

W h at m ight you see on neuroim aging of a patient with schizophrenia?

7. W hat drugs are known to cause psychosis in patients?

Antipsychotic Medications 8. Categorize the following antipsychotics in the appropriate category as neuroleptics (low- or high-potency) or atypical antipsychotics: olanzapine, thioridazine, quetiapine, chlorpromaxine, haloperidol, fluphenazine, loxapine, risperidone, thiothixene, trifluoperazine, clozapine, aripiprazole Low-potency neuroleptics -

High-potency neuroleptics -

Atypical antipsychotics -

9. A patient previously diagnosed with schizophrenia arrives at the psych ER with a severe neck spasm that forces his head to be maintained in an unusual position. W hat is the treatment? Acute dystonia (torticollis in this case) due to antipsychotics

R x :______________

10. W hat features characterize tardive dyskinesia that may develop from the use of high-potency typical neuroleptics?

DISO RDERS

12. In what tim e frame would you expect to see Parkinsonian symptom side effects in a patient taking antipsychotics?

PSYCHOTIC

11. W h a t is the treatm ent for tardive dyskinesia?

13. W hat is the treatm ent for Parkinsonian symptoms that have developed from neuroleptic administration?

14. W h at are the signs and symptoms of neuroleptic malignant syndrome? ° Mental status change —initial symptom in most patients (agitated delirium with confusion rather than psychosis) ° Muscular rigidity +/- tremor ° Hyperthermia greater than 38-40° C ° Rhabdomyolysis appearing over 1-3 days ° Autonomic instability - tachycardia, high/labile blood pressure, tachypnea, diaphoresis

15. W h at is the treatm ent for neuroleptic malignant syndrome? ° Stop the offending medication ° Supportive care in the ICU - IVF - Lower fever with cooling blankets, ice packs in the axilla, Tylenol - Reduce hypertension with clonidine and/or nitroprusside (—» cutaneous vasodilation can facilitate cooling) - D V T prevention with heparin or enoxaparin - For agitation, use benzos (clonazepam, lorazepam) o ______—» prevents rigidity and hyperpyrexia by inhibiting calcium release ° Other possible therapies:________________________

[57]

End of Session Quiz 16.

Give examples o f negative symptoms of schizophrenia.

17. W hich neuroleptics are known for their extrapyramidal side effects?

18.

19.

20.

W h a t are the different treatments for acute dystonia, tardive dyskinesia and neuroleptic malignant syndrome? ° Acute dystonia ° Tardive dyskinesia ° Neuroleptic malignant syndrome

W h a t is the most common side effect o f olanzapine?

W h at is the drug category o f choice for the treatment o f the negative symptoms o f schizophrenia?

PERSONALITY DISORDERS

3 Question Warm-Up 1.

A wom an who witnessed a murder is able to calmly describe every gory detail. W h ich defense mechanism is she displaying?

2.

W h ich defense m echanism is a person displaying when she subconsciously pushes memories o f past abuse out o f her conscious mind?

3.

A m other w ho is frustrated w ith her child yells at her husband. W h a t defense mechanism is she displaying?

PERSONALITY DISORDERS

End of Session Quiz 4.

A 40-year-old woman comes to your office after becoming dissatisfied with her former physician. She explains that her former physician was the most evil creature with absolutely no redeeming qualities. She finishes her story and remarks that she “can tell you are so smart and will cure her of everything.” W hat personality disorder might this patient have?

5.

A 32-year-old woman tells you during one of her office visits that you are the best doctor she’s ever had but that your nurse is very disrespectful. O n a subsequent visit, she threatens to change doctors because you do not feel a particular lab test is necessary. Additionally, you notice several symmetrical cuts on her left forearm which she attributes to cat scratches. W h at personality disorder does this person have?

6.

A 55-year-old woman comes to your office wearing all black including a black miniskirt and black feather boa. She is also wearing an excessive amount of lipstick, and you notice her having conversations w ith many o f the other patients in the waiting room. W h at personality disorder do you suspect in this patient?

7.

A person demands only the best and most educated doctor in town. W hat personality disorder does this person have?

SUBSTANCE ABUSE PART I

3 Question Warm-Up 1.

W h a t is the m ost serious side effect o f clozapine?

2.

W h a t is T odd’s paralysis?

3.

W h a t is the first-line treatm ent for acute otitis media?

4. W hat treatm ents are effective in helping prevent relapse in recovering alcoholics? ° _________________________ is the tried-and-true best relapse prevention O O o

° Acamprosate (Campral) tid

End of Session Quiz 5.

W h a t are the components o f the C A G E questionnaire?

6.

In w hich vitamins are alcoholics typically deficient?

7.

W h a t is the m ost successful treatm ent for alcoholism?

8.

List the symptoms associated w ith the life-threatening condition delirium tremens (D T).

9.

W h a t is the drug o f choice for alcohol withdrawal?

SUBSTANCE ABUSE PART 2

3 Question Warm-Up 1.

W hat medical conditions can cause severe depression?

2.

A 25-year-old man is seen in the E R w ith severe headache. H e tells you that the headaches always occur at around the same time, and that the pain is localized around the right eye. O n exam his pupils are unequal and his right eye is tearing. W hile he waits for the C T scan o f the head, w hat treatment should he receive?

3.

W h a t are the major symptoms o f H untington disease?

End of Session Quiz 4.

A 19-year-old slender woman presenting w ith recent weight loss is found to have erythem a o f her turbinates and nasal septum. W h at is the cause o f her weight loss?

5.

A patient is brought to the E R by police, restrained and exhibiting violent behavior. W h a t O D is likely? W h a t is the Rx?

6.

W h a t features are unique to P C P intoxication that allow you to distinguish it from LSD intoxication?

7.

W h a t drugs can be used in the case o f hypertension in a patient with cocaine or amphetamine intoxication?

OTHER PSYCH DISORDERS

3 Question Warm-Up 1.

W h at is the treatment for benign paroxysmal positional vertigo (BPPV)?

2.

W hat disorder classically presents w ith the triad o f cognitive impairment, urinary incontinence and abnormal gait?

3.

A patient with a history o f lithium use presents with copious amounts o f dilute urine. W h at condition is this?

. Conversion Disorder ° Sensory or motor neurological symptoms with no recognized neurological or medical cause Motor symptoms: ° Weakness/paralysis 0 Tremor ° Dystonia/myoclonus ° Gait disorder ° Dysphagia ° Dysphonia Sensory symptoms: ° Numbness/paresthesias ° Blindness ° Deafness ° May or may not be related to specific psychological stressor

Som atic Symptom Disorder ° One or more somatic symptoms that are distressing or significantly disruptive, e.g.: Pain symptoms Sexual symptoms (erectile dysfunction, decreased libido) Neurological symptoms (motor or sensory deficits, gait disorder, tremor) - Gastrointestinal symptoms (vomiting, diarrhea) ° Symptoms may or may not be due to a recognized medical condition ° Individual experiences disproportionate and persistent thoughts about the seriousness of his/her symptoms, and/or a persistently high level of anxiety about his/her health or symptoms.

Illness Anxiety Disorder ° Preoccupation with having or acquiring a serious illness ° High level of anxiety about health Individual performs excessive health-related behaviors, such as repeatedly checking his/ her body for signs of illness ° Somatic symptoms are not present

End of Session Quiz 7.

W h a t serum lab abnormalities may be seen in a patient with prolonged excessive vomiting/purging?

8.

W h a t somatoform disorder matches each o f the following descriptions? Patient w ith normal anatomy is convinced a part of his/her anatomy is abnormal. Unexplained loss o f sensory or motor function. Normal exam/tests Overwhelming worry about having a particular disease, w ithout any signs/symptoms Unexplained complaints in multiple organ systems

9.

10.

W h a t is a major difference between factitious disorder and malingering?

W h ich eating disorder can be treated effectively w ith SSRIs?

\ND DE

DELIRIUM AND DEMENTIA

3 Question Warm-Up 1.

Albuminocytologic dissociation (increased protein in the CSF, with only modest increase in cell count) indicates what condition?

2.

A violent patient w ith vertical and/or horizontal nystagmus has been exposed to what substance?

3.

Categorize each o f the following antidepressants: duloxetine, nefazodone, bupropion, mirtazapine, desvenlafaxine,fluvoxamine, doxepin,fluoxetine, imipramine, escitalopram, amitriptyline, phenelzine, milnacipran, sertraline, venlafaxine, paroxetine, tranylcypromine, citalopram, trazodone, nortriptyline SSR1

TCA -

MAOI -

N D RI -

SN RI -

Tetracyclic -

Atypical -

End of Session Quiz 4.

W h a t are key features o f delirium that differentiates it from dementia?

5.

W h a t are the two most common causes o f dementia, and how do you differentiate between them when m aking a diagnosis?

6.

A nurse pages you, saying that one o f your elderly patients has been sleeping most o f the two days he’s been on the unit, but is very agitated and aggressive w ith the nurses while awake. She asks you to write for a benzodiazepine to sedate the patient. W h a t do you propose instead and why?

7.

W h a t disease th a t causes dem entia is also associated w ith visual hallucinations and frequent falls?

8.

W h a t disease that causes dementia is also associated w ith unpleasant behavioral and personality changes?

PEDI PSYCH

PEDI PSYCH

3 Question Warm-Up 1.

After a minor car accident, a woman wears a neck brace and requests permanent disability. W h a t’s the most likely diagnosis?

2.

In which disease would you find atrophy o f the mammillary bodies?

3.

W h a t is the most common cause o f sensorineural hearing loss? W h at is the most common cause o f conductive hearing loss?

Attention-Deficit Hyperactivity Disorder (A D H D ) 4. W hat other medications can be used in the treatment of A D H D in children who fail to respond to stimulants or atomoxetine?

5.

W hat are the common complications of A D H D stimulant medications, and how are these complications managed? Stimulants ° Insomnia ~> address sleep hygiene, take meds earlier in the day, change to shorter duration formulation, clonidine at night 0 Appetite suppression and weight loss administer meds after meals rather than before ° Tics —> usually transient, choose low-moderate dose methylphenidate which does not worsen tics ° Psychosis or mania —>discontinue (no need to taper) ° Decreased growth velocity -> reassure parents that adult height is not affected, drugholidays may help with “catch-up” growth

Atomoxetine ° Increased risk of suicidality —* close observation and usually discontinuation ° Liver injury —» discontinue

[ 68 ] I

Tourette Syndrome 6. W hat therapeutic options are available for Tourette syndrome? ° Counseling/psychotherapy for social adjustment and coping ° If interfering with necessary functions of life —> anti-dopamine agents: fluphenazine, pimozide or tetrabenazine (which are all tolerated better than haloperidol in these patients) ° If only focal motor or vocal tics —» Botox injections into affected muscles ° If impulse control problems —> clonidine or SSRIs ° If refractory to medical management —* consider deep brain stimulation of globus pallidus, thalamus or other subcortical target (undergoing clinical trials)

Autism Spectrum Disorder 7.

W hat are some of the characteristic features of autism spectrum disorder? ° “Living in his own world” ° Symptoms evident prior to age 3 ° Lack of responsiveness to others, poor eye contact, absence of social smile ° Impairments in communication, language delay, repetitive phrases ° Peculiar repetitive, ritualistic habits (e.g., spinning around, hand flapping) ° Fascination with specific, seemingly mundane objects (vacuum cleaners, sprinklers) ° Usually below-normal intelligence

PEDI PSYCH

End of Session Quiz 8.

How is A D H D diagnosed?

9.

W hich childhood psychiatric disorder matches each of the following statements? Females only. Loss of previously acquired purposeful hand skills between 6-30 months Impairments in social interactions, communications, play. Repetitive behaviors Characterized by hostility, annoyance, vindictiveness, disobedience and resentfulness Multiple motor and vocal tics Impulsive and inattentive

10.

W hen treating a child suspected of having a learning disorder, what must first be investigated?

E ndocrinology 1

Type 1 D iabetes

2

Type 2 D iabetes

3

Insulin Therapy

4

A cute C om plications o f D iabetes

5

C hronic C om plications o f D iabetes

D IA B E T E S EV A LU A T IO N O F H Y P E R H Y P O T H Y R O ID IS M ) A N D LESS CO M M O N PRO BLEM LES, A E V A R IO U S SYN D R O M A D R E N A L IN SU FFIC IE N C Y A N D A D R E N A L E X C E S S ),

N orm al Thyroid F unction and H yperthyroidism

7

H ypothyroidism and Thyroid C ancer

8

Parathyroid D isorders

9

P itu itary D isorders

10 C ush in g Syndrom e and H yperaldosteronism 11 O th e r A d ren al D isorders

ENDOCRINOLOGY

6

TYPE I DIABETES

3 Question Warm-Up 1.

W h a t is the treatm ent for neuroleptic m alignant syndrome?

2.

W h a t is the m ost common complication o f recurrent otitis media?

3.

W h a t is the treatm ent for delirium tremens (DTs)?

4. W hat anti-islet antibodies can be seen in patients with type I diabetes mellitus? ° ° ° °

Anti-insulin (IAA) Anti-islet cell cytoplasm (ICA) Anti-glutamic acid decarboxylase (GAD) Anti-tyrosine phosphatase (IA-2)

End of Session Quiz 5.

W h a t lab test is used in diabetic patients to assess the adequacy o f glycemic control over the last three months?

6.

W h a t is the leading cause o f death in diabetics?

7.

W h y m ust 3-blockers be used w ith caution in diabetics?

8.

W h a t can cause hypoglycemia in a non-diabetic patient?

TYPE 2 DIABETES

TYPE 2 DIABETES

3 Question Warm-Up 1.

W h at medications other than stimulants are used in the treatment o f A D H D ?

2.

W h a t are the three reasons for involuntary psychiatric hospitalization?

3.

W h a t is the psychiatric condition in which a person travels a long distance, takes a new name and has no memory o f his prior life?

4. W hich of the oral agents used in the control of type 2 diabetes has the following characteristics: Lactic acidosis is a rare but worrisome side effect Most common side effect is hypoglycemia Oldest and cheapest of the oral agents Often used in combination with any of the other oral agents Also helps lower triglycerides and LDL cholesterol levels N ot safe in settings of C H F Should not be used in patients with elevated serum creatinine Should not be used in patients with inflammatory bowel disease Hepatic serum transaminase levels should be carefully monitored when using these agents Not associated with weight gain, often used in overweight diabetics Metabolized by liver, excellent choice in patients with renal disease

[ 74]

Primarily affects postprandial hyperglycemia, taken with meals

5. W hat are the criteria for the diagnosis of metabolic syndrome? Diagnosis based on any three of the following: ° ____________________ : Waist circumference > 40 in (102 cm) in men, or > 35 in (88 cm) in women (IDF criteria is > 94 cm in men and > 80 cm in women) ° ___________________ > 150 mg/dl ° ____________________ < 40 mg/dl in men, or 130/85 mmHg ° ____________________ > 100 mg/dl (or 2-hour post oral glucose > 140 mg/dl)

H -< ~U m ro O > CD m H m on

End of Session Quiz W h a t skin finding can be a sign o f having insulin resistance?

7.

W hich diabetic medications should be avoided in patients w ith heart failure?

8.

A type 2 diabetic has been well-controlled on a regimen o f metformin, pioglitazone and glyburide, but now he is developing some episodes o f sweating, tachycardia and confusion. These episodes resolve quickly if he eats something. W hich o f his medications would be most likely to cause hypoglycemia?

9.

M atch the diabetes drug with its mechanism o f action:

ENDOCRINOLOGY

6.

Decreases G I absorption o f starch and disaccharides Stimulates insulin release Decreases hepatic gluconeogenesis Increases tissue glucose uptake and improves insulin sensitivity Mimics the action o f GLP-1: i glucagon, T insulin, delays gastric emptying Inhibits DPP-IV, leading to 1 glucagon, T insulin, delays gastric emptying

I [75]

INSULIN THERAPY

INSULIN THERAPY

3 Question Warm-Up 1.

A n aphasic patient has difficulty producing words but understands everything you say. W h a t type o f aphasia does he most likely have?

2.

W h a t would you see on physical exam o f a patient that has a cholesteatoma?

3.

W h a t antidepressant is preferred in the treatment of depression w ith comorbid neuropathic pain?

End of Session Quiz 4.

W hich type o f insulin is used in continuous infusion insulin pumps and in treatment o f DKA?

5.

W h at must be kept in m ind for a type 1 diabetic patient that plans to begin a strenuous exercise program?

ACUTE COMPLICATIONS OF DIABETES

3 Question Warm-Up 1.

W h a t is the definitive treatm ent for an epidural or subdural hematoma?

2.

W h a t medication is used in the diagnosis o f symptomatic myasthenia gravis?

3.

W h a t are the m ost common causes o f seizures in children ages 2-10?

4. W hat are the common causes of DKA? Usually due to excess glucagon, catecholamines or corticosteroids ° Infection (pneumonia, gastroenteritis, UTI) ° Medication reduction or omission ° Severe medical illness (MI, CVA, trauma) ° Undiagnosed DM ° Dehydration ° Alcohol or drug abuse ° Corticosteroids

5. W hat are the necessary steps in the treatm ent of diabetic ketoacidosis?

ACUTE COM PLICATIONS OF DIABETES

End of Session Quiz 6.

W hat are the signs and symptoms of diabetic ketoacidosis?

7.

A 68-year-old type 2 diabetic ran out o f his metformin a week ago, and now presents to the E R with 3 days o f severe polyuria, dry mouth and vomiting. H e is lethargic, and seems to be an unreliable historian. His labs show Na+=144 m Eq/L, K+=3.7 m Eq/L, Cl"=101 m Eq/L, H C 0 3 =18 m E q/L , and serum glucose=413 mg/dL. Based on this information, what is the most likely diagnosis, and what two tests will help you confirm that diagnosis?

8.

W h a t is the work-up for the underlying cause o f DKA?

CHRONIC COMPLICATIONS OF DIABETES

3 Question Warm-Up 1.

W h a t medications are used in the treatm ent o f Tourette syndrome?

2.

W h a t is the cause o f muscle rigidity, fever and rhabdomyolysis in a schizophrenic patient?

3.

W h ich antidiabetic agent is associated w ith lactic acidosis?

4. How do we diagnose diabetic gastroparesis?

5.

W hat are the treatm ent options for diabetic gastroparesis?

W hat are the routine health maintenance recommendations for diabetics? ° ° ° ° °

°

°

° ° ° ° °

Exercise: walking at least 2 hours a week reduces mortality by about 40% Healthy diet Daily finger stick blood glucose (FSBG) documented and brought to clinic visit Physical exam every 3-6 months with attention to blood pressure (goal < 130/80), weight loss, feet, waist circumference HbAlC every 3 months if > 7.0, every 6 months if < 7.0 American Diabetic Association goal < 7.0 American Association of Clinical Endocrinologists goal < 6.5 Urine microalbumin every 3 months-1 year 24-hour urine for protein, C r and CrCI if UA protein > 100 or high serum C r Consider annual 24-hour urine for protein, C r and CrCI Lipid panel every year Goal total cholesterol < 150 Goal LD L < 100 (< 70 if evidence of vessel disease): Use statins Goal H D L > 40 (men), > 50 (women) —(niacin may worsen insulin resistance) Chem 8 and UA every year Dilated eye exam (rule out retinopathy, glaucoma, cataracts) every year Influenza vaccine every year Pneumococcal vaccine Consider daily ASA 81 mg, ACE inhibitor and statin

End of Session Quiz 7.

W hich class o f antihypertensive drugs will reduce proteinuria and slow/prevent the progression o f diabetic nephropathy?

8.

W h a t is the treatment for proliferative diabetic retinopathy? W h a t is the treatment for peripheral neuropathy?

9.

Besides retinopathy, what other common eye diseases are diabetics at an increased risk o f developing?

10. H ow is diabetic gastroparesis diagnosed and treated?

NORMALTHYROID FUNCTION &HYPERTHYROIDISM

3 Question Warm-Up 1.

W h a t are the symptoms o f T C A overdose?

2.

D uring a yearly physical, a middle-aged m an jokes that he needs to buy some reading glasses because he is having difficulty reading fine print. H e also notes th at he has trouble driving at night and reading road signs. Though the patient may have some presbyopia, for w hat else should you examine him?

3.

W h ich cranial nerve is responsible for each o f the following actions? ° Eyelid opening ° Taste from anterior 2/3 of tongue ° Head turning ° Tongue movement ° Muscles of mastication ° Balance ° Monitoring carotid body and carotid sinus chemoreceptors and baroreceptors

AND HYPERTHYROIDISM FUNCTION NORMAL THYROID

End of Session Quiz 4.

W hat thyroid abnormalities would you expect to find during pregnancy?

5.

A patient has exophthalmos. W h at is the most likely cause?

6.

W hat is the treatment o f the most common cause o f hyperthyroidism?

7.

In which o f the following hyper thyroid diseases is radioactive iodine most likely to result in hypothyroidism: Graves disease, toxic multinodular goiter, toxic adenoma?

HYPOTHYROIDISM AND THYROID CANCER

3 Question Warm-Up 1.

W h a t serum lab abnormalities m ight you see in a patient with bacterial meningitis?

2.

W h a t is the general treatm ent for DKA?

3.

W h a t disorder is present in a teenager w ith a history o f theft, vandalism and violence toward family pets?

4. Thyroid Nodules ° ° ° °

Incidence of palpable thyroid nodules (> 2cm) is about 4-5% of adults About 1 out of 10 nodules is cancer Death rate from thyroid cancer is about 1 out of 1 million Higher chance that a nodule is cancer if: - Child Elderly History of external radiation Male Family history of thyroid cancer ° More likely malignant if sono/Doppler shows: Irregular margins Intranodal vascular pattern - Microcalcifications

How do you manage a thyroid nodule? ° Check TSH, free T4, thyroid ultrasound to measure size and assess for other nodules ° If hyperthyroid -►radionucleotide uptake scan No role for radionudeotide uptake scan if euthyroid or hypothyroid! Hot nodule —»treat as hyperthyroid Cold nodule —* FNA

AND THYROID

CA N CER

5.

HYPOTHYROIDISM

° If hypothyroid -> replace thyroid replacement and monitor for decrease in nodule size If nodule persists after thyroid replacement —» FNA ° If euthyroid —> FNA

6.

How should you follow up a FN A of a thyroid nodule? ° Malignant —> needs surgery ° Benign -> repeat thyroid ultrasound every 6 months-1 year to make sure no increase in size If size increases —* repeat FNA 0 Nondiagnostic —> repeat FNA ° Intermediate —» repeat thyroid ultrasound in 6 months o Even if FNA is negative, you cannot completely rule out thyroid cancer. Explain to patient that there is still about a 1% chance that this could be cancer and offer to let them choose between ever 6 months ultrasound surveillance or removal of the nodule (partial vs. total thyroidectomy).

[84] |

x

End of Session Quiz 7.

W h a t are the major differences between de Quervain thyroiditis and H ashim oto thyroiditis?

-< TJ 0 H 1 -<

73

O o >

z

8.

W h a t is the m ost common type o f thyroid cancer?

o H X -< 73 q o

n > z n

9.

W h a t is the next step in the management o f a newly found thyroid nodule in a patient w ith hyperthyroidism?

[ 85]

DISORDERS PARATHYROID

3 Question Warm-Up 1.

A patient has exophthalmos, pretibial myxedema and decreased T S H . W h a t is the diagnosis?

2.

W h a t are the most worrisome side effects o f the A D H D drug atomoxetine?

3.

W h a t important side effects are common to many o f the atypical antipsychotics?

4. W hat are the 2 most common causes of primary hyperparathyroidism? W hat is the treatm ent for each? The 2 most common causes are parathyroid adenoma and parathyroid hyperplasia. Treat as follows: ° Surgical parathyroidectomy if one of the following:

-

____________________________ > 1.0 mg/dL above the upper limit of normal ____________________________ reduced by 30% ____________________________ T-score < -2.5 at any site Age < ----------------------------------------------

0 If adenoma and surgery indicated —> removal of only the gland containing the adenoma and biopsy of 1-3 other glands ° If hyperplasia and surgery indicated —> removal of 3 % glands and marking the remaining half with a surgical clip (or forearm autotransplantation of the gland to remain in cases where recurrence is likely such as M EN type I and Ila) 0 If surgery is not recommended or is refused: Prevent worsening hypercalcemia by avoiding certain medications (________________ and____________________), volume depletion, prolonged bed rest or calcium ingestion >IOOOmg/day Adequate hydration (6-8 glasses of water daily) to avoid renal stones Minimize bone resorption via _________________ , exercise, lOOOmg calcium daily (except if calcitriol is elevated) and adequate vitamin D intake (400-600 lU/day) Routine monitoring of serum calcium (q6m), serum creatinine (ql2m) and bone density at the hip, L-spine and forearm (ql2m)

[86] |

W hat is the treatm ent for hyperparathyroidism due to chronic renal disease? Hyperphosphatemia ° Dietary restriction of phosphate (protein) ° Oral phosphate binders taken with meals - most commonly used (not calcium citrate which increases aluminum absorption) Sevelamer - nonabsorbable (does not cause hypercalcemia), can be taken with calcium Lanthanum —long-term side effects currently unknown Aluminum hydroxide - now avoided because of gradual induction of aluminum toxicity

Renal Osteodystrophy ° Lessen hyperphosphatemia via the above measures in order to lessen bone resorption ° Calcitriol, other vitamin D analog or cinacalcet (a calcimimetic) to suppress P T H secretion

End of Session Quiz 6.

H ow will vitamin D deficiency affect the levels o f Ca2+, P T H and phosphate?

7.

W h y m ight P T H be elevated in renal disease?

8.

W h a t happens to phosphate in patients with hyperparathyroidism caused by renal disease?

9.

Com pare P T H , alkaline phosphatase, serum calcium and serum phosphate levels in patients w ith the following diseases:

C a2+ Paget Disease Osteomalacia/Rickets Chronic renal failure Osteoporosis Osteopetrosis Prim ary hyperparathyroidism Hypoparathyroidism Pseudohypoparathyroidism

Phos

Aik Phos

PTH

PITUITARY DISORDERS

3 Question Warm-Up 1.

List the symptoms o f depression using the mnemonic SIG E CAPS:

2.

A patient comes to clinic w ith the complaint o f hearing loss and vertigo. O n examination o f the tympanic membrane you note a grayish-white “pearly” lesion involving the T M . W h a t is the diagnosis?

3.

W h a t is the most likely cause o f galactorrhea, impotence (or menstrual dysfunction) and decreased libido in a patient w ith a history o f schizophrenia?

Hyperprolactinemia 4. W hat are the symptoms of hyperprolactinemia? ° Premenopausal female - hypogonadism —* infertility, oligo/amenorrhea; rarely galactorrhea ° Postmenopausal female —none since already hypogonadal; rarely galactorrhea ° Male symptoms - hypogonadism (low testosterone) —> decreased libido, impotence, infertility (low sperm counts), gynecomastia, rarely galactorrhea

5. W hat is the treatm ent for a prolactinoma? ° First step: Dopamine (DA) agonist ( _ _______________________ > bromocriptine or pergolide) ° If DA agonist ineffective —* switch to a 2nd DA agonist ° If DA agonists are ineffective -» ____________________ ° If female with adenoma > 3cm and desire to become pregnant (during which time the DA agonist is withheld) transphenoidal surgery even if the DA agonist is effective ° If large adenoma is surgically removed —> radiation therapy after surgical debulking

Acrom egaly 6. W hat are the symptoms and signs of acromegaly? W hat tests can be used to confirm the diagnosis? The average tim e frame from onset to diagnosis is 12 years, and it presents with the following symptoms: ° Enlarged jaw (teeth spread apart); nose and frontal bones (coarse facial features); hands and feet (increase in ring, glove, shoe size) ° Soft tissue growth: voice deepens, macroglossia (teeth indentations in tongue), carpal tunnel syndrome and other entrapment syndromes, hypertrophy of synovial tissue and cartilage —> arthropathy ° Cardiovascular disease: H TN , LVH, diastolic dysfunction ° Glucose intolerance in 50%, DM in 10%

Diagnostic testing: ° Screen: measure serum levels of insulin-like growth factor 1 (IGF-1) ° Confirm diagnosis with oral glucose suppression test (75g glucose —>measure G H at lh r and 2hr —» if G H concentration is >lng/mL = acromegaly) ° If testing positive for acromegaly —>pituitary M RI to eval for mass or empty sella

7.

W hat is the treatm ent for acromegaly? ° Transsphenoidal resection of pituitary adenoma or external beam radiation ° If unable to resect adenoma —>somatostatin analog (octreotide or lanreotide) —inhibits G H secretion ° If somatostatin analog ineffective —> cabergoline (dopamine agonist that inhibits GH secretion) Bromocriptine is less effective than cabergoline ° If cabergoline ineffective —> pegvisomant (GH receptor antagonist)

End of Session Quiz 8.

W hat is the most common presentation o f hyperprolactinemia?

9.

W hat drugs are known for causing elevated prolactin levels?

10.

W hat is the next step in the management o f a patient with hyperprolactinemia not due to an obvious drug cause?

11.

W h at is the next step in the management o f a patient found to have an absent pituitary on M R I (empty sella)?

12.

W h a t is the visual field deficit classically associated w ith prolactinoma?

13.

W h a t complications can result from acromegaly?

CUSHING SYNDROME AND HYPERALDOSTERONISM

n

c GO X

z

o GO -<

z

o 73

3 Question Warm-Up 1.

W h a t is the m ost common pituitary tumor, and what is the treatment?

oz m

Z

a X -<

-o 2.

A 16-year-old woman has a left arm paralysis after her boyfriend dies in a car crash. No medical cause is found. W h a t is the diagnosis?

m

73 >

i— D

O 7> 3.

A t w hat age do physicians begin to dose dexamethasone w ith (or prior to) the first dose o f antibiotics in cases o f suspected bacterial meningitis?

Z GO

z

End of Session Quiz 4.

O f D H E A , D H E A -S and testosterone, which is made only by the adrenals and is a more specific marker for an androgen-producing adrenal tum or in a woman?

5.

W h a t are the electrolyte abnormalities found in hyperaldosteronism?

6.

W h a t is the most specific lab finding in making the diagnosis o f primary hyperaldosteronism?

7.

A patient s work-up reveals a high plasma aldosterone concentration and low plasma renin activity. W h a t is the diagnosis, and what medication could be used to treat this condition until definitive treatment can be undertaken?

OTHER ADRENAL DISORDERS

3 Question Warm-Up 1.

W h at is the first-line treatment for moderate hypercalcemia?

2.

A 22-year-old man has 4 months o f social withdrawal, worsening grades, decreased emotional expression, and concrete thinking. His speech is frequently tangential and incoherent, and he sometimes says that the T V weatherman is giving him instructions. W h at is the diagnosis?

3.

W h a t is the pattern o f vision loss in glaucoma vs. macular degeneration?

End of Session Quiz 4.

A patient w ith elevated blood pressure, palpitations, headache and excessive perspiration is found to have elevated urine vanillylmandelic acid levels. W hat effect would giving a 3-blocker have on this patient?

5.

W hat is the likely condition o f a female infant with virilization o f the genitalia and hypotension?

6.

W h a t serum lab abnormality would you see in 17a-hydroxylase deficiency and in 21a-hydroxylase deficiency?

7.

A patient w ith acromegaly is found to have elevated C a2+ on a blood draw during a work-up o f his peptic ulcer. W h a t is the diagnosis o f this patient?

ER, ICU, Surgery 1

A ccidents and Injuries p art 1

2

A ccidents and Injuries p a rt 2

3

Toxicology p art 1

4

Toxicology p a rt 2

5

Toxicology p a rt 3

6

C ardiovascular Em ergencies

7

C ritical C are

8

T raum a p a rt 1

9

T raum a p a rt 2

10 T raum a p a rt 3 11 P re -O p and P o st-O p Issues 12 Surgical Em ergencies and T ransplantation

ACCIDENTS AND INJURIES PART I

3 Question Warm-Up 1.

W h a t lab changes will be seen in a patient w ith hyperaldosteronemia?

2.

W h a t lab findings are diagnostic o f H ashim oto thyroiditis?

3.

W h a t are the 5 basic criteria for metabolic syndrome?

Burns 4. W hat are the differences between 1st, 2nd, 3rd and 4th degree burns?

Traditional Classification

Depth Classification

Involvement

1st degree

Superficial

Only the epidermis

Signs and Symptoms Painful, erythema Capillary refill intact

2nd degree

3rd degree

4th degree

Superficial partial­ thickness

Epidermis and partial thickness of the dermis

Deep partial­ thickness

Epidermis and partial thickness of the dermis

Full thickness

Epidermis, entire dermis and possibly deeper tissues Additional involvement of muscle and bone

Painful, erythema Capillary refill intact Painful, blisters

W hite and/or charred

W hat complications can arise from electrical burns? ° In electrical burns, internal damage may be worse than external damage ° Complications may include:__________________ , ___________________ , bony injuries, myoglobinuria, acidosis,___________________ and/or various neurologic disturbances

W hat is unique to the management of electrical burn patients as compared to heat burn patients? °

____________________________ to prevent myoglobinuria, renal failure and acidosis in the face of muscle necrosis ° High index of suspicion for compartment syndrome ° Obtain an EKG and monitor for dysrhythmias

W hat are the common life-threatening complications in a patient with substantial burns?

End of Session Quiz 8. W h a t is the Parkland burn formula?

9.

A patient presents w ith 2nd degree burns covering the anterior surfaces o f both arms. W h a t is the surface area o f the burn?

10. W h a t are the complications to watch for in a patient that has suffered an electrical bum ?

11. A m ine worker is brought into the E R after an explosion occurred; examination o f the burns, which is difficult secondary to pain at the site, reveals extension into the dermis and blisters. To w hat degree is this burn classified?

12. W h a t pulm onary pathology should you be -watching for in a patient who had a saltw ater near-drowning episode?

13. W here is an aspirated object such as a penny m ost likely to end up?

14. W h a t is the next step in the managem ent o f a patient th at has aspirated an object th at cannot be dislodged, and is now having difficulty moving air and is becoming hypoxic?

ACCIDENTS AND INJURIES PART 2

ACCIDENTS AND INJURIES PART 2

3 Question Warm-Up 1.

A 60-year-old obese patient presents with dirty, velvety patches on the back of the neck. W h at is the diagnosis, and what is the initial workup?

2.

W hat is the appropriate treatment of hypertension in cases of pheochromocytoma?

3.

W h at is the classic presentation o f a patient w ith hyperprolactinemia?

Bites and Stings 4. W hat is the treatment for a black widow spider bite? ° Mild skin reactions resolve in less than 12 hours without complications Initially wash wound with soap and water Ice to reduce inflammation -

Analgesia as needed Pressure and immobilization to slow the systemic spread of venom 24 hours observation for signs of systemic involvement ° If systemic symptoms (AKA latrodectism manifesting with muscle spasms, abdominal stiffness, altered mental status, autonomic stimulation): -

formusclespasm Analgesia with acetaminophen +/- opioids Antivenin ideally within________________ of the bite

5. W hat is the treatment for a brown recluse spider bite? ° If ulceration —* wound care with dressing changes and debridement ° If signs of infection, cellulitis, abscess —> antibiotics (___________________________ ) ° Consider______________________________to possibly reduce the extent of local necrosis due to leukocyte inhibitory properties (r/o G6PD deficiency prior to dapsone use due to risk of hemolytic anemia)

W hat is the treatm ent for a dog or cat bite? ° Clean surface with iodine, then copious pressure irrigation with normal saline. Use a soft IV catheter to get deep into the wound ° Should you suture the wound closed? Puncture bites and dog bites to the_______________ should not be closed with sutures should be sutured due to low rate of infection Higher likelihood of infection favors leaving wound open. Infection more likely in _____ bites than in dog bites ° ___________________ prophylaxis if the animal cannot be observed for 10 days or if the animal is suspected to be rabid. ° ____________________ immunization if not received in the previous 5 years 0 If the victim is a child, then follow-up psychological assessment for__________________ is indicated as it occurs in more than 50% of these children.

W hat are the indications for a tetanus booster in an adult patient? ° Td should be given to every adult every___________________ A Tdap booster is recommended once in place of the Td between ages 19-64 years ° Td should also be given to patients with wounds as recommended below:

immunizations

Uncertain or < 3 prior tetanus immunizations

Non tetanus-prone wound, LE, clean 8c minor

Td if > 10 years since last dose

Td (D T if < 7 years old) -» complete series (3 total)

Tetanus-prone wound (dirt, contamination, puncture, crush injury)

Td if > 5 years since last dose

Td (D T if < 7 years old) + tetanus immune globulin 250 units IM at site other than Td —> complete series

> 3 prior tetanus

End of Session Quiz 8.

W h a t E K G abnormality is classic for patients w ith hypothermia?

9.

W h a t is the treatment for a black widow spider bite?

10.

A fter appropriate irrigation and cleansing, should a dog bite wound to the arm be left open or sutured closed?

11.

W h a t is the treatm ent for a skin laceration on the dorsum o f the hand that resulted from a closed fist hitting a victim s mouth?

TOXICOLOGY PART I

3 Question Warm-Up 1.

W h a t is the treatment for hyperparathyroidism due to parathyroid hyperplasia?

2.

W h at is the L D L goal in a patient with diabetes?

3.

W h a t method is used to calculate fluid repletion in burn patients?

W hat are the potential consequences of acetaminophen overdose?

Stage 1

30 min - 24 hrs

Nausea, vomiting, diaphoresis, pallor, lethargy and malaise

Stage II

24-72 hours

o Elevated LFTs, PT and total bilirubin 0 RUQ_pain and tenderness

Stage III

72-96 hours

° Peak LFT elevation ° Jaundice, hepatic encephalopathy, bleeding, +/- acute renal failure ° Possible multisystem organ failure —* death

Stage IV

4 days —2 wks

Recovery

W hat are the symptoms of anticholinergic toxicity? W hat is the antidote? ° ° ° ° ° ° ° °

Hot as a hare (hyperpyrexia) Dry as a bone (decreased secretions) Red as a beet (cutaneous flushing) Blind as a bat (cycloplegia and mydriasis) Mad as a hatter (disorientation) Bloated as a toad (constipation and urinary retention) Tachycardia Decreased or absent bowel sounds

° Antidote -

W h a t are the signs and sym ptom s of cyanide ingestion? ° Tachycardia, hypertension, flushing, tachypnea —>obtundation —>coma —>death ° (discernible to 60% of the population) ° Late findings: bradycardia, hypotension, bradypnea, cyanosis, hepatic necrosis, renal failure ° Delayed-onset sensitive to cyanide)

in survivors of severe poisoning (basal ganglia is

W hat is the treatm ent for cyanide poisoning? ° Airway, Breathing, Circulation ° High-flow oxygen regardless of pulse-ox readings Mouth-to-mouth resuscitation is contraindicated here due to risk of provider exposure ° If oral ingestion —> 1 dose activated charcoal (50g in adults) ° Sodium thiosulfate Sulfur donor that facilitates the conversion of cyanide to thiocyanate which is renally excreted ° Hydroxocobalamin (a vitamin B12 precursor) Directly binds cyanide to form cyanocobalamin which is less toxic and excreted in the urine Causes a reddish discoloration of the skin, mucous membranes and urine Works well in conjunction with sodium thiosulfate Generally preferred over nitrate-induced methemoglobinemia which can be lethal ° Amyl nitrate and sodium nitrate Induce methemoglobinemia which binds cyanide to form cyanomethemoglobin Goal methemoglobin level is 20-30% (but this is lethal in children and anemic patients) Contraindicated in patients with carboxyhemoglobinemia (usually from smoke inhalation) Methemoglobinemia can be reversed with methylene blue, but this should be avoided in cyanide-poisoned patients because it will release free cyanide - W orks well in conjunction with sodium thiosulfate

End of Session Quiz 8.

W h a t cardiac medications are safe to treat the tachycardia and hypertension associated w ith cocaine overdose?

9.

W h a t is given to treat acetam inophen overdose? W h a t other uses does this m edication have?

10. W h a t medications are used in cases o f cyanide poisoning?

TOXICOLOGY PART 2

3 Question Warm-Up 1.

A patient on haloperidol develops fever, muscle rigidity, confusion and diaphoresis. W h a t is the drug o f choice in the treatment of this patient s condition?

2.

W h a t kind o f tum or is the most common brain tumor?

3.

Lung cancer accompanied by muscle weakness is indicative o f what?

W hat are the classic features of digoxin toxicity? ° Nonspecific symptoms: fatigue, blurred vision, change in color vision (e.g., “yellow vision”), anorexia, nausea, vomiting, diarrhea, abdominal pain, headache, dizziness, confusion, delirium ° Characteristic EKG changes: Prolonged PR interval, “scooping” of ST segments - seen at therapeutic levels _____________________ - frequent vital sign abnormality PVCs are most common findings on EKG Atrial tachycardia with AV block (e.g., 4:1 or 6:1) —less common ° ___________________indicates the severity of digoxin toxicity (which inhibits the Na+-K"-ATPase) ° Elevated serum digoxin levels

W hat is the treatment for digoxin toxicity? ° Activated charcoal in repeated doses ° _____ _____________ if one of the following is present: Hemodynamic instability Life-threatening arrhythmias or severe bradycardia (even if responsive to atropine) Plasma potassium level > 5 mEq/L in an acute overdose Plasma digoxin level > lOng/mL Ingestion of > 10 mg of digoxin in adults or >4 mg in children Presence of a digoxin-toxic rhythm in the setting of an elevated digoxin level ° Treat hyperkalemia only if it is causing EKG disturbances and avoid_______________ which can worsen intracellular hyperkalemia in these particular patients ° If bradycardia —> __________________ ° ACLS medications as needed (except calcium)

° Airway, Breathing, Circulation ° NG tube gastric aspiration if ingestion of a large amount within the last 60 minutes (rare) ° _____________________ —to correct acidosis and limit penetration of toxic metabolites into tissues (such as retina) ° Inhibit the alcohol dehydrogenase enzyme with fomepizole or ethanol: ____________________ —preferred drug ___________________ - if fomepizole is unavailable, dose to serum level of 100 mg/dL ° Dialysis if________________________________________________________________ (e.g., vision changes or renal failure) ° Folic acid, thiamine and pyridoxine supplementation to optimize elimination pathways

7. W hat are the characteristics of aspirin overdose? ° _____________________ —an important part of the patient history 0 Hyperthermia (as ASA uncouples mitochondrial oxidative phosphorylation) 0 _____________________ (from hyperventilation) —>then mixed respiratory alkalosis and metabolic acidosis with elevated anion gap - Tachypnea results from A SA stimulation of the medullary respiratory center - Acidosis results from accumulation of lactic acids and ketoacids ° Nausea and vomiting, dehydration ° Altered mental status

8. W hat is the treatm ent of a patient that has an elevated IN R from excessive warfarin ingestion? ° IN R monitor IN R q I-2 days and resume dosing when IN R at desired level + lower routine dose Skip next dose + administer vitamin K (l-2.5mg orally) + lower routine dose ° IN R >9 Hold dosing until IN R in therapeutic range + administer vitamin K5-I0m g PO once and as needed subsequently + lower routine dose ° IN R >20 —>choose to treat as if “serious bleeding” or as IN R >9 ° Any serious bleeding - Vitamin K lOmg slow IV + administer fresh frozen plasma (FFP*) + monitor IN R and repeat FFP* dosing as needed + stop warfarin (individual clinical judgment must be used in decision to restart) * Prothrombin complex concentrate or recombinant human factor Vila can be used in place of FFP

TOXICOLOGY PART 2

6. W hat is the treatm ent for methanol or ethylene glycol toxicity?

End of Session Quiz 9.

W h at are the antidotes to each o f the following toxins? Aspirin Acetaminophen Opioids Benzodiazepines Tricyclic antidepressants Atropine Propranolol Digoxin Cyanide Methemoglobin Methanol Isoniazid Heparin

10.

A t w hat point in a patient with an elevated IN R due to warfarin would you consider dosing vitam in K to reverse the warfarin?

TOXICOLOGY PART 3

3 Question Warm-Up 1.

W h a t is the m ost sensitive test for multiple sclerosis?

2.

W h a t type o f bite/sting can rarely cause acute pancreatitis?

3.

In which endocrine disorder m ight weight loss completely eliminate the need for medication?

O X

o*— o Cl -< -a > 7>

4. A schizophrenic patient comes to the E R for ingestion of alkali plumbing liquid. How is this patient managed? ° Airway, Breathing, Circulation - If respiratory distress —> laryngoscopy —> +/- tracheostomy ° Emergency surgery if signs of perforation, mediastinitis or peritonitis. ° D O N O T give an emetic such as_____________________ ,_a _____________________ (which will improve nothing, but may result in thermal injury), o r__________________ (which may lead to perforation or emesis of caustic material) ° If asymptomatic and reliable history of low volume, accidental ingestion —» outpatient follow-up only may be appropriate. Otherwise, manage as follows: - Endoscopy (EGD) to look for severity of injury - ICU care to manage life-threatening complications (mediastinitis, peritonitis, respiratory distress, shock) ° Esophageal dilations 3-6 weeks after injury if necessary for strictures ° Surveillance EG D beginning 15-20 years after the ingestion at an interval of ql-3 years to screen for esophageal squamous carcinoma

[ 105]

W hat are the signs and symptoms of organophosphate poisoning? DUMBBELSS: Diarrhea, Urination, Miosis, Bronchospasm, Bradycardia, Emesis and Excitation of skeletal muscle, Lacrimation, Sweating, Salivation and abdominal cramping. (Think of extreme “rest and digest,” parasympathetic/cholinergic activity and loss of fluids from multiple areas of the body.)

W hat sequelae would you expect to see in a toddler that ingested too much iron when he found his mother’s prenatal vitamins? ° Gastrointestinal phase (30 min - 6 hours after ingestion) Abdominal pain, vomiting; diarrhea (often btoody), hematemesis, melena, lethargy, shock In cases of mild iron toxicity, this is usually the only phase present If no Gl symptoms develop within____________ after ingestion, it is unlikely that iron toxicity will occur (sinless the patipnt consumed enteric-coated iron)

° Latent/stable phase (6 - 24 hours after ingestion) Observation is required when patients have gone through the Gl phase in orderto determine if the toxicity was mild then resotyed or if the patient simply entered into this Latent phase

° Shock and metabolic acidosis (6 —72 hours after ingestion)

*

Widespread cellular dysfunction Multisystem organ failure, Gl bleeding and/or perforation, pulmonary dysfunction, coagulopathy, renal dysfunction and neurologic dysfunction can all occur in this phase Outcomes are poor in this stage and therapies have little effect

° Hepatotoxicity/hepatic necrosis (12 - 96 hours after ingestion) ° __________________________ ( 2 - 8 weeks after ingestion) Occurs from G l scarring classically at the gastric outlet Vomiting is the pfesenting.symptom

W hat are the antidotes to the following toxins? ° Arsenic ° Copper ® t-PA, streptokinase

W hat problems can arise from theophylline overdose? ° Therapeutic range is 10-20mg/L. Every patient is unique and some may have mild SE at ISmg/L. Even a level above 25mg/L is worrisome. ° Main SE: seizures, hyperthermia, hypotension and__________________________ ° Seizure SE may occur at levels of 14-35mg/L. Seizure risk is more likely in older patients, prior brain injury patients, patients with severe pulmonary disease and patients with hypoalbuminemia.

End of Session Quiz 9.

W h a t is the treatment for lead poisoning in adults? W hat is the treatment in children?

o -

o CD

-< > 7>

10. W h a t is the next step in the management o f a patient that presents to the E R w ith organophosphate poisoning?

11. W h a t classic toxic ingestion management options should not be chosen in patients presenting w ith alkaline fluid ingestion?

12.

A C O P D patient comes to the E R w ith tachycardia and hypotension. D uring the evaluation he begins to have seizures. W h a t is the most likely etiology?

13. W h a t are the antidotes to each o f the following toxins? Arsenic m 73

Anticholinesterases, organophosphates Carbon monoxide

O

Copper

on

C C

Iron Mercury

73 O m

t-PA

3

I [ 107]

CARDIOVASCULAR EMERGENCIES

3 Question Warm-Up 1

A 52-year-old diabetic presents to the E R w ith classic anginal chest pain, and you suspect an M I. W hile waiting for the EKG, what treatments do you initiate immediately?

2.

W h a t are the antidotes for each o f the following types o f overdose? Opioids Heparin Benzodiazepines Barbiturates (phenobarbital) Carbon monoxide

3.

A patient presents w ith weakness, nausea, vomiting, weight loss and new skin pigmentation. Labs show hyponatremia and hyperkalemia. W h a t is the treatment?

W hat drugs are used in treating a stable, asymptomatic ventricular tachycardia?

W hat is the treatm ent for supraventricular tachycardia (SVT)? Attempt vagal maneuvers and carotid massage —>adenosine —» ventricular rate-control (calcium channel blocker or (3-blocker)

W hat is the initial treatm ent for new atrial fibrillation with rapid ventricular rate of unknown duration? ° Rate control with one or more of the following: ^-blockers, diltiazem or verapamil, or digoxin Anticoagulation with heparin (then later warfarin)

W hat is the treatm ent for pulseless electrical activity (P EA ) or asystole?

Evaluate and treat H causes ______ —» rapid volume resuscitation through multiple IV sites and/ or central line ______________________—> correct via intubation, chest tube or oxygen ______________________—> IV push 1-2 amps bicarb (commonly needed in a prolonged “code”) ______________________ (common in a prolonged “code" due to acidosis) —» CaCI2 IV push, bicarb, insulin/glucose ______________________ —» administer KCI ______________________ (always check a finger-sticki) —> I amp D50 IV push ______________________ —> warming Evaluate and treat T causes pericardiocentesis ■needle decompression then chest tube ■cardiac cath or thrombolytic •thrombolytic or thrombectomy ■follow ATLS protocols

End of Session Quiz 8.

W hat is the maximum number of epinephrine doses that can be given when treating cardiac arrest?

9.

In treating pulseless ventricular tachycardia, what drug could be given in place of the first or second dose o f epinephrine?

10. W hat piece o f medical history should be obtained in deciding how to treat atrial fibrillation?

11. W h a t is the ACLS protocol for ventricular fibrillation?

CRITICAL CARE

3 Question Warm-Up 1.

W h a t are the antidotes to the following toxins? Antimuscarinic, anticholinergic agents Benzodiazepines Tricyclic antidepressants W arfarin M ethanol, ethylene glycol (antifreeze) Arsenic

2.

A n 18-year-old m an is found to have a systolic heart m urm ur heard at the apex and left lower sternal border th at increases in intensity while standing after squatting. W h a t diagnosis do you suspect?

3.

W h a t is the clinical definition o f hypertension?

CRITICAL CARE

End of Session Quiz 4.

W hich blood product is most appropriate in each of the following scenarios? Severe anemia due to autoimmune hemolytic anemia Hemophilia DIC Shock due to trauma or postpartum hemorrhage To maintain blood pressure during large volume paracentesis Hemorrhage due to warfarin overdose Need for vWF-rich blood product Thrombocytopenia

5.

W hich vasopressor matches each o f the following statements? Theoretically causes renal vasodilation High doses optimize the a t vasoconstriction A D H analogue Best choice for anaphylactic shock Best choice for septic shock Best choice for cardiogenic shock Causes vasoconstriction but with bradycardia

6.

In a Swan-Ganz catheter (pulmonary artery catheter), o f what is the wedge pressure a good estimate?

TRAUMA PART I

H :30 > C 3 > 5 H

3 Question Warm-Up 1.

W h a t is the classic E C G appearance in atrial flutter?

2.

W h a t side effects can arise from theophylline overdose?

3.

W h a t is the treatm ent for Kawasaki disease in the acute phase?

Traum a Assessm ent 4. W hat is the basic sequence of assessing a traum a patient? ° Airway - Obtain airway (ET tube or cricothyroid) if patient not moving air (If patient is speaking then patient is moving air) ° Breathing - Start 0 2 - Pulse-ox Bag ventilation if airway good, but still not breathing. - Consider needle decompression and/or chest tube placement if auscultation is consistent with pneumothorax. ° Circulation - Check pulse (carotid, femoral or other) - Check vitals and rhythm on heart monitor —> A CLS per protocol Listen for heart sounds - Make sure 2 large bore IVs or a central line has been started - Start IVF Control active bleeding ° Disability - Glasgow coma scale - Neuro exam (limited) - Fingerstick blood glucose ° Exposure Remove all clothes DRE & Foley - Cover with blanket to avoid hypothermia

[ 113]

TRAUMA PART I

Traum a Assessm ent cont ° Secondary Survey Reassess ABCs DRE & Foley (no Foley if: mobile or high-riding prostate, or blood from urethral meatus) N G T placement (as needed) History (Allergies, Meds, Drugs, PMH/PSH, Last meal, Events of trauma) Head-to-toe exam Examine back for wounds or spinal injuries Remove from backboard X-rays - usually C-spine + C X R + AP pelvis FAST scan (Focused Abdominal Sonography for Trauma) C T scans (as needed) Labs (CBC, Chem 14, EtOH, UDS, UA) ° Other Management Serious fracture management 0 Pelvic fracture: wrap bedsheet tightly around pelvis to stabilize ° Femur fractures: closed reduction via manual traction, then stabilize with traction brace ° Open fractures: remove any gross debris —>NS pressure wash x5L —> cover with Kerlex Meds: antibiotics, analgesics, tetanus shot Consults Documentation and talk with family

Head Trauma S. W hat is Cushing’s triad?

6. W hat are the classic physical findings in a basilar skull fracture?

7. W hat interventions can be used to lower intracranial pressure in a head injury patient? ° If patient needs intubation, pretreat with lidocaine to minimize ICP elevations ° _______________ IV loadthenq6 hours. q6hr serum______________ and______________ (don’t rely on the Chem 7 calculated osmolarity) Hold mannitol if Na > 152 mEq/L or osmolarity > 305 mOsm/L ° Intubate and_____________until p C 0 2is 25-30 on ABG. Decreasing p C 0 2by 5-10 mmHg will lower ICP by 20-30 mmHg. This effect lasts less than a few hours and should only be used as a temporary measure. ° Other options: ventriculostomy, barbiturate coma, paralysis

Spinal Cord Traum a 8. W hat is the treatm ent for anterior spinal cord syndrome following a traumatic injury? ° Assess and manage Airway, Breathing and Circulation first. Keep spine stabilized. ° If hypotension, aggressively bolus fluids, but hmit fluids once normotensive to avoid cord swelling. ° Immediate high-dose IV steroids (ideally 8 hours) - methylprednisolone 30 mg/kg initial IV bolus then 5.4 mg/kg per hour for 23 hours Controversial but still considered the standard of care at most hospitals "A treatment option” rather than treatment standard according to American Academy of Neuro. Surgeons and American Academy of Emergency Medicine Only used in non-penetrating cervical trauma without multi-system trauma ° C T and M RI of the spinal cord ° Once stabilized, decompression via closed reduction (halo headpiece and 5-15 lbs of weight) or surgical intervention

End of Session Quiz 9.

A patient presents to the E R after being involved in a motor vehicle collision (MVC). H e does not open his eyes or make any sounds, but he does withdraw to painful stimuli. W h a t is his Glasgow coma score (GCS)? W h a t needs to be done?

10. A nother M V C victim arrives to the ER. H e is able to carry on a conversation and even comment on how nice one o f the nurses’ hair looks. H e is able to move all o f his extremities on command, and he has significant periorbital ecchymosis and surrounding superficial scrapes. W h a t is his GCS? W h a t radiological study does he definitely need?

11. W h a t interventions are effective in the management o f elevated intracranial pressure?

12. W hat are the symptoms of a basilar skull fracture?

TRAUMA PART 2

§ s?i

TRAUMA PART 2

3 Question Warm-Up 1

W h at heart disease fits each o f the following descriptions? ST segment elevation in leads corresponding to the perfusion of multiple arteries Hypotension + distant heart sounds + distended neck veins Cardiac cath shows equal pressures in all heart chambers Chest pain that lessens when the patient leans forward

2.

W h at is the next step in the evaluation o f a pulsatile abdominal mass and bruit?

3.

W h at is the treatment for ventricular fibrillation?

Neck Trauma 4. W hat are the different zones of the neck and structures contained in each zone?

Zone

Landmarks

Structures Contained

I

Clavicle —> cricoid cartilage

Great vessels, aortic arch, trachea, esophagus, lung apices, cervical spine, spinal cord and cervical nerve roots

II

Cricoid cartilage —» angle of the mandible

Carotid and vertebral arteries, jugular veins, pharynx, larynx, trachea, esophagus, cervical spine and spinal cord

III

Angle of the mandible —> base of the skull

Salivary and parotid glands, esophagus, trachea, cervical spine, carotid arteries, jugular veins, major cranial nerves

1

Chest Traum a 5. W hat C X R findings might indicate a ruptured thoracic aorta? loss of aortic knob, pleural cap, deviation of the trachea and esophagus to the right, and depression of the left main stem bronchus

6. W hat are the signs of tension pneumothorax? W hat is the treatment?

I > c > 5 7> H

I 10

° Signs: absent breath sounds and hyperresonance to percussion (hollow sound) on the affected side, distended neck veins, hypotension ° Treatment: - Immediate_____________________________ If delay in chest tube placement,_____________________________ on the affected side (2nd or 3rd 1C space at midclavicular line or 5th 1C space at midaxillary line)

7.

W hat is a flail chest, and what is the classic presentation? ° Flail chest is a free-floating portion of the chest wall that moves paradoxically to the rest of the chest wall (inward motion with inspiration) and results from 3 or more sequential rib fractures ° Patients may have muscle splinting due to pain which may conceal the paradoxical motion of the chest wall ° Mechanism of injury is usually direct impact of the steering wheel or crush injury

8. W hat is the treatm ent for a flail chest? ° Oxygen supplementation ° Close monitoring for early signs of respiratory compromise ° Placing an object (e.g., sandbag) to the affected region to stabilize the segment is no longer done due to the consequence of restricting chest expansion BiPAP by mask or endotracheal intubation with mechanical ventilation (otherwise the patient may become hypoxic from limiting breathing due to pain)

[117]

TRAUMA PART 2

Abdominal Trauma 9. W hat are the initial steps in the management of an abdominal stab wound presenting to the ER? ° Airway, Breathing, Circulation If hypotensive —>■_________________________________________

® Abdominal exam Signs of peritonitis (peritoneal irritation)

— *

____________________________________

Explore th e stab wound under local anesth< -.ia »

If wound penetrates an terio r fa sc ia —> O perate

» If no penetration or unable to assess, admit for serial 24-hour exams —> surgical exploration if peritonitis, hemodynamic instability develop, Or positive on additional testing (listed below)

° NG tube - to decompress stomach and rule out blood in the stomach ° Urinary catheter Additional possible testing in the stable patient not already needing surgical exploration: ° Diagnostic Peritoneal Lavage (DPL) if indicated ° Upright CXR - to identify hemo- or pneumothorax and/or intraperitoneal air ° Diagnostic ultrasound —to identify hemoperitoneum ° Abdominal CT with contrast ° Diagnostic laparoscopy

10. W hat are the next steps in the management of blunt abdominal traum a in a patient with stable vital signs? ° Airway, Breathing, Circulation (Primary Survey then Secondary Survey) ° Establish IV access at two sites with large bore IVs ° NG tube and Foley O ° Stat H&.H +/- Blood type and cross

11. W hat are the next steps in the management of a patient with blunt abdominal traum a and unstable vital signs? ° Primary and secondary survey ° Assess for and manage pelvic fracture ° FAST (Focused Assessment with Sonography for Trauma) If blood in pelvis

' ________________ _____________________

If no blood in pelvis —»■possible retroperitoneal hemorrhage If FAST inconclusive

— >

___________________

> _______________

° If no blood in pelvis and angiography is normal, then +/- admission

+ observation

12. W hat is the treatm ent for a retroperitoneal hematoma? ° If penetrating injury or exsanguination into abdomen (bloody peritoneal aspirate) ° If blunt trauma without blood in the abdomen —> Follow_______________ If hemodynamically unstable or falling H&H —> ________________________________

Genitourinary and Pelvic Traum a 13. W hat is the immediate treatm ent for a patient with a pelvic fracture? ° Primary and secondary survey including a thorough neurovascular examination ° IV F luid+/-blood Ultrasound (FAST) to assess for fluid in the pelvis —> if no fluid in pelvis and patient is hemodynamically unstable —» _____________________________________ to detect bleeding missed by FAST If blood detected —» _____________________________________ If hemodynamic instability but no blood detected —> consider retroperitoneal hemorrhage —> _____________________________________ ° _______________________ until the external fixator is placed (a bedsheet twisted and tied tighdy around the pelvis can be used if nothing else is available) Assess for bladder and urethral injury with retrograde cystourethrogram -

If extraperitoneal bladder rupture —^ _____________________________________

-

If intraperitoneal bladder rupture —>■______________________________________

End of Session Quiz 14. W h at is the next step in the evaluation o f the following patients? Pelvic fracture + DPL shows blood in the pelvis Pelvic fracture + DPL shows urine in the pelvis Pelvic fracture + DPL shows nothing + hemodynamic instability Blunt abdominal trauma + unstable vital signs + FAST shows fluid in pelvis Blunt abdominal trauma + unstable vital signs + FAST shows no fluid in pelvis Blunt abdominal trauma + unstable vital signs + FAST inconclusive Blunt abdominal trauma + stable vital signs Abdominal stab wound + hypotensive or signs of peritonitis 15. W h at additional studies can be performed in the case o f a stable patient w ith an abdominal stab wound that penetrated the peritoneum?

TRAUMA PART 3

3 Question Warm-Up 1.

W h a t type o f immunodeficiency increases the risk o f anaphylactic transfusion reaction?

2.

W h a t would you find on physical exam o f a patient w ith pericardial effusion?

3.

A patient is in the hospital and begins to have atrial fibrillation w ith RVR (rapid ventricular rate). This patient has had chronic atrial fib previously. W h a t study has to be performed before the patient can be cardioverted?

4. W hat antibiotic prophylaxis should be provided for rape victims? W hat other prophylactic measures should also be taken? ° _______________________ 125mg IM (gonorrhea) ° _______________________ lg PO o r________________________ lOOmg PO bid for 7 days (Chlamydia) ° _______________________ 2g PO (Trichomonas) ° Hepatitis B vaccine #1 of 3 (if not yet vaccinated) +/- Hep B immune globulin (not standard of care) ° _______________________ for 3-7 days with follow-up for further counseling. Common prophylactic regimens: ° Antiemetic (Promethazine) for nausea caused by H IV meds and pregnancy prophylaxis ° Levonorgestrel (Plan B) 0.75mg PO —» repeat dose in 12 hours (alternatively, both doses can be taken at once for improved compliance) (other options for emergency contraception discussed in Gyn lecture)

End of Session Quiz 5.

W h a t should be done to evaluate an extremity trauma?

6.

Since bleeding between the fetus and m other is a concern in traum a, w hat actions should be taken once the patient and the fetus are stabilized?

PRE-OP AND POST-OP ISSUES

3 Question Warm-Up 1.

W hich antidepressants are associated w ith hypertensive crisis? W h a t substance can exacerbate this effect?

2.

W h a t EK G finding is associated w ith hypothermia?

3.

W h a t are the classical physical findings in cases o f endocarditis?

End of Session Quiz 4.

W hen is the greatest risk for a post-operative M I? W h at is recommended perioperatively for patients with known CAD?

5.

W h at lab findings suggest hepatic disease during a pre-operative work-up?

6.

W hat interventions are helpful in optimizing lung function in the post-op period in patients with preexisting lung disease?

7.

W h a t studies are ordered to evaluate the cause o f a fever in a post-op patient?

SURGICAL EMERGENCIES AND TRANSPLANTATION 3 Question Warm-Up 1.

W h a t would you suspect in an E R patient with blood in the urethral meatus or a high-riding prostate?

2.

W h a t are the tw o measures o f how severe the hypotension is in shock?

3.

W h a t causes a continuous machine-like heart murmur?

4. W h at is the typical E R lab work-up for a patient with acute abdominal pain?

5. W hat are the signs and symptoms of acute mesenteric ischemia? ° ° ° ° °

Sudden onset of severe abdominal pain (periumbilical) that is_______________________ Vomiting and diarrhea Condition that could cause emboli formation (e.g., atrial fibrillation) Early exam: mosdy normal, abdominal distention, +/- occult blood on stool Late exam (bowel infarction): abdominal distention, absent bowel sounds, peritoneal signs, feculent odor to the breath

6. W hat is the treatm ent for acute mesenteric ischemia? ° General care Hemodynamic monitoring and support. Dobutamine or dopamine if necessary. Broad-spectrum antibiotics for diagnosis and treatment ° Specific care Heparin anticoagulation - Papaverine infusion to decrease arterial vasospasm If embolism —» embolectomy and resection of necrotic bowel —» second-look laparotomy in 24-48 hours to remove additional necrotic bowel If thrombus —> thrombectomy and revascularization (e.g., supraceliac aortic graft to the involved intestinal artery) —> resection of necrotic bowel and second-look laparotomy as needed —» aspirin daily after recovery - Resection of necrotic bowel and bypass

SURGICAL EMERGENCIES AND TRANSPLANTATIO N

7. W hat are the classic signs/symptoms of chronic mesenteric ischemia? ° AKA intestinal angina ° Dull, crampy, postprandial epigastric pain within the first hour after eating, then subsiding over 2 hours ° Weight loss (due to food aversion to avoid postprandial pain) ° Possibly nausea, vomiting and early satiety ° Abdominal bruit (50%)

8. W hat are the treatment options for chronic mesenteric ischemia?

End o f Session Quiz 9.

W h at is seen on abdominal x-ray or C T scan that indicates a ruptured viscus?

10.

W h a t is the classic finding in the abdominal exam o f a patient w ith mesenteric ischemia?

11.

W h a t is the usual lab panel ordered in a patient presenting to the E R with generalized abdominal pain?

12.

W h a t is the type o f rejection th at is treatable w ith immunosuppressive agents? W h a t is the m echanism o f this rejection? W ith in w hat tim e frame may it show up?

C ardiovascular 1

C ardiology Basics

2

A therosclerosis

3

H ypercholesterolem ia

4

Stable A n g in a

5

U nstable A n g in a

6

M yocardial In farctio n

7

A rrh y th m ias p a rt 1

8

A rrh y th m ias p a rt 2

9

H e a rt Failure

10 V alvular D iseases 11 C ardiom yopathies and Pericardial D isease 12 M yocardial Infections 13 H ypertension 14 A ntihypertensives 15 Shock 16 V ascular C onditions 17 Vasculitis 18 Pedi C ardiology

CARDIOLOGY BASICS

3 Question Warm-Up 1.

Categorize the following antipsychotics into the appropriate category as lowpotency neuroleptic, high-potency neuroleptic, or atypical antipsychotics. olanzapine, thioridazine, quetiapine, chlorpromazine, haloperidol,fluphenazine, loxapine, risperidone, thiothixene, trifluoperazine, dozapine, aripiprazole, paliperidone, droperidol Low-potency neuroleptics —' High-potency neuroleptics Atypical antipsychotics —

2.

W h a t overdose causes metabolic acidosis and retinal damage leading to blindness?

3.

W h a t are the electrolyte abnormalities found in hyperaldosteronism?

End of Session Quiz 4.

W h ich coronary artery is the most common site o f occlusion?

5.

W h ich E C G leads correspond to occlusion o f the LA D ?

6.

In which phase o f the cardiac cycle do coronary arteries fill w ith blood?

7.

A n IC U patient has a B P o f 120/78. Calculate her m ean arterial pressure.

8.

W h a t is the electrophysiological reason that a Q R S complex would become widened?

■1EROSCLEROSIS

ATHEROSCLEROSIS

3 Question Warm-Up 1.

Otoscopy in a child presenting with acute onset o f ear pain reveals large reddish vesicles on the T M . W h at is the diagnosis, typical organism and treatment?

2.

A burn patient presents with cherry-red flushed skin and coma. 0 2 saturation is normal, but carboxyhemoglobin is elevated. W h at is the treatment?

3.

W h at mineralocorticoid medication is used in the treatment o f aldosterone deficiencies such as adrenal insufficiency and 21-hydroxylase deficiency?

End of Session Quiz 4.

Your patient is undergoing a treadmill stress test. W h a t key EKG finding would suggest myocardial ischemia during exercise?

5.

Stress testing is done as a screening test. I f angina or ischemia occurs with these tests, what test should follow?

6.

Besides lowering LD L, what other effect do statins have that may contribute to their ability to reduce the incidence of myocardial infarction?

HYPERCHOLESTEROLEMIA

3 Question Warm-Up 1.

A bipolar patient w ho is treated w ith a m ood stabilizer has concurrent depression. W h a t should you use to treat his depression?

2.

W h a t are the differing presentations o f Alzheim er disease, Pick disease and Lewy body dementia?

3.

W h a t are two key differences between somatization disorder and conversion disorder?

End of Session Quiz 4.

W h ich lipid-lowering agent matches each o f the following descriptions? SE: Facial flushing SE: Elevated LFTs, myositis SE: GI discomfort, bad taste Best effect on H D L Best effect on Triglycerides Best effect on LDL/cholesterol Binds C. difficile toxin

5.

W h a t are the recommendations for L D L levels in patients based on their C A D risk?

6.

How can the flushing reaction o f niacin be prevented?

STABLE ANGINA

3 Question Warm-Up 1

Chvostek and Trousseau signs are associated w ith what metabolic abnormality?

2.

Cold water is flushed into a patient’s ear, and the fast phase o f the nystagmus is toward the opposite side. W here is the lesion?

3.

A lesion to which area o f the brain is responsible for the following clinical scenarios? 0 Contralateral hemiballismus I Ilieumi in il neglect syndrome ° Coma ° Poor repetition “ Poor comprehension 3 Poor vocal expression

4. W hat medications are used in the treatment of Prinzmetal angina?

5. W hat is the most likely cause of chest pain in each of the following scenarios? ° ° ° ° ° ° ° ° ° ° ° ° °

ST segment elevation only during brief episodes of chest pain Patient is able to point to localize the chest pain using one finger Chest wall tenderness on palpation Rapid onset sharp chest pain that radiates to the scapula Rapid onset sharp pain in a 20-year-old and associated with dyspnea Occurs after heavy meals and improved by antacids Sharp pain lasting hours-days and is somewhat relieved by sitting forward Pain made worse by deep breathing and/or motion Chest pain in a dermatomal distribution Most common cause of noncardiac chest pain Acute onset dyspnea, tachycardia and confusion in a hospitalized patient Pain began the day following an intensive new exercise program Widened mediastinum on CXR

End of Session Quiz 6.

W h ich patients are more likely to have atypical angina (or no angina) during an episode o f myocardial ischemia?

7.

H ow does nitroglycerin work acutely in a cardiac ischemic episode?

8.

W h y should relief o f chest pain with nitroglycerin administration not be used as a diagnostic test for whether or not the chest pain is cardiac in nature?

UNSTABLE ANGINA

3 Question Warm-Up 1. W h a t would you suspect to be the cause o f hyperthyroidism in a patient presenting with the symptoms o f hyperthyroidism in addition to the following findings? » Extremely tender thyroid gland ° Pretibial myxedema ° Pride in recent weight loss, medical professional 0 Palpation of single thyroid nodule ° Palpation of multiple thyroid nodules ° Recent study using IV contrast dye (iodine) ° Eye changes: proptosis, edema, injection ° History of thyroidectomy or radioablation of thyroid 2.

W h a t is the difference between conduct disorder and oppositional defiant disorder?

3.

W h a t is the difference between conduct disorder and antisocial personality disorder?

Initial management of unstable angina ° ABCs ° MONA: IV Morphine (decreases stress and cardiac oxygen demand), Supplemental 0 2 (only if hypoxemia present), Nitroglycerin (decreases preload and BP), Aspirin ° p-blocker (if no signs of heart failure) decreases blood pressure, contractility, heart rate and oxygen consumption. Caution in patients with COPD, asthma or diabetic patients. ° Statin (preferably before PCI) ° Antiplatelet therapy to all patients (clopidogrel or ticagrelor). For very high risk patients (recurrent ischemic discomfort, dynamic EKG changes or hemodynamic instability), a GP Ilb/IIIa inhibitor (eptifibatide or tirofiban) may be added ° Give anticoagulant therapy to all patients. Unfractionated heparin to all patients undergoing PCI. Enoxaparin for patients not managed with PCI. ° Potassium above 4 mEq/L and magnesium above 2 mg/dL

End of Session Quiz 5.

W h a t are the mechanisms o f action o f the following drugs? ° Streptokinase ° Aspirin 0 Clopidogrel « Abciximab 0 Tirofiban 0 Ticlopidine T“* < ° Lnoxapann ° Eptifibatide

6.

W h a t is the benefit o f 3-blockers in patients w ith unstable angina?

7.

W h en should oxygen be used in a patient w ith suspect cardiac ischemia?

is !

MYOCARDIAL IN FARCTIO N

MYOCARDIAL INFARCTION

3 Question Warm-Up 1.

W h at is the treatment for opioid overdose?

2.

W hat is the classic presentation o f a patient w ith aspirin overdose?

3.

W h at is the first-line treatment for a growth hormone-secreting pituitary adenoma?

End of Session Quiz 4.

W h a t medications should all post-M I patients receive as outpatients?

5.

W hich medication types have a proven reduction in mortality following M I?

6.

In what time frame do thrombolytics need to be given in an M I? H ow does this differ from the time frame in which they need to be given for a stroke?

7.

W h at are specific labs ordered in patients suspected o f having an M I?

8.

W h at is the most common cause o f death in patients w ith an acute myocardial infarction?

ARRHYTHMIAS PART I

3 Question Warm-Up 1.

W h a t is the antidote to each o f the fallowing toxins? Salicylates (3-blockers Digoxin Iron Copper t-PA, streptokinase

2.

A patient is found to have hypertension, m ild hypernatremia, hypokalemia and metabolic alkalosis. W h a t is the m ost likely diagnosis?

3.

W h a t is the treatm ent far an M I due to a cocaine overdose?

ARRHYTHMIAS PART I

End of Session Quiz 4.

W hat type o f heart block is described by the following statements? PR interval prolonged more than 0.2 sec (5 small boxes) No relationship between P waves and QRS PR interval becomes progressively longer until a beat is blocked PR interval fixed, but with occasional blocked beats

I 136]

5.

W hich heart blocks need a pacemaker?

6.

W hat is the drug o f choice in paroxysmal supraventricular tachycardia?

ARRHYTHMIAS PART 2

1> yO X

“< H X

3 Question Warm-Up 1.

H o w does adjustment disorder w ith depressed m ood differ from major depressive disorder?

2.

In acute labyrinthitis, patients will have symptoms o f vertigo, nystagmus and nausea/vomiting. W h a t other im portant symptom will they have?

3.

Does strabismus cause amblyopia, or does amblyopia cause strabismus?

3 > CO "D > H NJ

End of Session Quiz 4.

W h ich endocrine disorder can cause atrial fibrillation?

5.

W h ich antiarrhythm ic should be avoided in patients w ith preexisting lung disease?

6.

W h a t is the treatm ent for sustained ventricular tachycardia?

n

> 7=> a

in

C /l

c 73

HEART FAILURE

HEART FAILURE

3 Question Warm-Up 1

A lesion to which area o f the brain is responsible for each o f the following clinical scenarios? ■> Resting tremor ° Intention tremor » Hyperorality, hypersexuality, disinhibited behavior ° Personality changes “ Agraphia, acalculia, finger agnosia

2.

W h at organism is known for causing infections in bum victims?

3.

W h a t is the treatment o f atrial fibrillation o f unknown duration?

4. W hat are Kerley B lines? Thickening of the subplcural interstitium (interlobular septa) about 1cm in length and 1mm in thickness seen in the periphery of the lower lung zones on CXR resulting from one of the following diseases: ° Left ventricular failure, mitral valve disease ° Lymphatic obstruction, lymphangitis carcinomatosis ° Asbestosis, sarcoidosis

138 i j

End of Session Quiz 5.

W h a t is a normal range for the ejection fraction?

6.

W h a t E C G finding may indicate a very early stage o f heart failure?

7.

W h a t lab marker is used to help diagnosis acute exacerbations o f C H F ?

8.

W h a t medications are im portant in the outpatient treatm ent o f chronic C H F ?

9.

W h a t is the treatm ent for acute exacerbations o f C H F ?

VALVULAR DISEASES

3 Question Warm-Up 1.

In which patient populations are -trip tan drugs contraindicated?

2.

W hich drugs block transmission through the AY node?

3.

W h a t causes stones, bones, groans and psychiatric overtones?

W hich heart valves should blood be flowing through during systole?

W hat are the systolic heart murmurs?

W hat heart valves should blood be flowing through during diastole?

W h at are the diastolic heart m urm urs?

End of Session Quiz 8.

W h a t is the next step in the work-up o f a low-grade systolic m urm ur in an otherwise healthy, asymptomatic patient?

9.

W h a t is the next step in the work-up o f a diastolic m urm ur in an otherwise healthy, asymptomatic patient?

10. W h a t type o f heart m urm ur fits each o f the following descriptions? Diastolic murmur heard best at left lower sternum that increases w ith inspiration Late diastolic murmur w ith an opening snap (no change w ith inspiration) Systolic murmur heard best in the second right interspace, parasternal Systolic murmur best heard in the second left interspace, parasternal Late systolic murmur best heard at the apex Diastolic murmur with a widened pulse pressure Holosystolic m urm ur that is louder with inspiration at the left lower sternum Holosystolic murmur heard at the apex and radiates to the axilla

CARDIOMYOPATHIES AND PERICARDIAL DISEASE

CARDIOMYOPATHIES AND PERICARDIAL DISEASE

3 Question Warm-Up

4.

1

W hat is the most common cause o f chest pain in a patient w ith sudden tearing chest pain radiating to the back?

2.

Increased skin pigmentation is seen in patients with which kind o f adrenal insufficiency?

3.

W h at are the classic symptoms o f Parkinson disease?

Kussmaul Sign vs. Pulsus Paradoxus

Kussmaul Sign

Pulsus Paradoxus

Event

JVD with inspiration

Decreased SBP by more than 10 mmHg with inspiration

Mechanism

Decreased capacity of RV

Decreased capacity of LV

Disease

Constrictive pericarditis >> cardiac tamponade

Cardiac tamponade » pericarditis

End of Session Quiz 5.

W h a t is the classic appearance o f the heart on a C X R o f a patient w ith pericardial effusion?

6.

W h a t is the treatm ent for cardiac tamponade?

7.

W h a t disease has signs o f heart failure + diabetes mellitus + elevated LFTs?

8.

In w hat scenarios m ight you see Kussmaul sign (increased JV D w ith inspiration)?

MYOCARDIAL INFECTIONS

3 Question Warm-Up 1

W h at are the symptoms o f neuroleptic malignant syndrome?

2.

W h at is the treatment for neuroleptic malignant syndrome?

3.

A teenager whose father died at age 30 in a motor vehicle accident suddenly collapses and dies while exercising. W h a t is the cause o f death?

End o f Session Quiz 4.

W hat are the common viruses th at cause myocarditis?

5.

W h at are the major Jones criteria o f acute rheumatic fever?

6.

W h a t are the H A C E K bacteria?

7.

Describe the following peripheral signs o f endocarditis: ° Janeway lesions 0 Osier node ° Roth spots 0 Splinter hemorrhages

8.

W h a t is the best study to visualize a vegetation on a heart valve?

HYPERTENSION

3 Question Warm-Up 1.

W h a t causes hypocalcemia, high phosphorus and low P T H ?

2.

W h a t is the classic EK G finding in pericarditis?

3.

W h a t commonly causes heart failure in young patients?

4. W hat blood pressures define prehypertension? How should prehypertension be managed? ° Prehypertension = SBP_______________ or D B P _______________ ° If the patient’s SBP is >130 or DBP >80 and has diabetes, chronic kidney disease, end-organ damage or cardiovascular disease, then medical management with an antihypertensive is indicated ° If none of the above comorbidities, then non-pharmacologic management with weight reduction, sodium restriction, increased physical activity and avoidance of excess alcohol

5. W hat tests can be used to diagnose renal artery stenosis? ° ______________________________of renal arteries - most frequently used screening test ° ______________________________of renal arteries with IV contrast ° ______________________________- time-consuming (2 hours) and requires well-trained operator ° ______________________________- gold standard, but invasive

End of Session Quiz 6.

H ow many consecutive abnormally high readings are needed before prescribing BP medications?

7.

W h a t intervention is most effective at reducing blood pressure?

8.

W h a t is the most common cause o f secondary H T N ?

9.

W h at is the most likely cause o f secondary hypertension given the following findings? High BP in U E but low BP in LE Proteinuria Hypokalemia Tachycardia, diarrhea, heat intolerance Hyperkalemia Episodic sweating, tachycardia

ANTIHYPERTENSIVES

3 Question Warm-Up 1.

W h a t lab marker is used to help diagnose acute exacerbations o f C H F?

2.

W h a t is the work-up for the underlying cause o f DK A ?

3.

W h a t are the “H ” causes o f PEA ?

End of Session Quiz 4.

W h ich antihypertensive class is first line in patients w ith the following problems? Diabetes H eart failure (multiple) BPH Left ventricular hypertrophy Hyperthyroidism Osteoporosis Benign essential tremor Post-menopausal woman Migraines

5.

W h ich antihypertensive drugs fit the following side effects? First dose orthostatic hypotension Hypertrichosis D ry mouth, sedation, severe rebound H T N Bradycardia, impotence, asthma exacerbation Reflex tachycardia Cough Avoid in patients w ith sulfa allergy Angioedema Development of drug-induced lupus Cyanide toxicity

.

SH O C K

SHOCK

3 Question Warm-Up 1.

W h at is the most common cause o f Cushing syndrome?

2.

W h a t is the Parkland burn formula?

3.

W h at is Beck’s triad for cardiac tamponade?

Shock 4.

[ A8 ] |

Know the different types of shock and how they present: Cause of shock:

M AP

SV R

HR

PCW P (left atrial pressure)

PCW P after fluid challenge

Hypovolemic (often due to hemorrhage)

Low

High

High

Low

Unchanged or high

Fluids

Cardiogenic

Low

High

Variable

High

Very high

Dobutamine or dopamine

Extracardiac obstruction (tension PTX, massive hemothorax)

Low

High

High

Low or normal

Unchanged or increased

Chest tube

Extracardiac obstruction (cardiac tamponade)

Low

High

High

High

High or very high

Pericardiocentesis

Neurogenic

Low

Low

Low

Low or normal

High

IVF, pressors, Atropine for HR

Septic

Low

Low

High

Low or normal

High

Fluids, Abx, NE

Anaphylactic

Low

Low

High

Low

High

Epinephrine

Treatm ent

End of Session Quiz 5.

Hypoperfusion and resultant tissue ischemia are the concern in shock patients. W h a t is the chemical marker o f this?

6.

W h a t complications can arise from the use o f vasopressors such as norepinephrine in treating shock?

7.

W h a t is the underlying mechanism o f each o f the following types o f shock? Cardiogenic Extracardiogenic Hypovolemic Anaphylactic Neurogenic Septic

VASCULAR C O N D IT IO N S

VASCULAR CONDITIONS

3 Question Warm-Up 1.

W h at are the two most common adverse effects o f statin use, and what labs would reveal these effects?

2.

W h a t type o f shock causes a decreased cardiac output, decreased P C W P and decreased peripheral vascular resistance?

3.

W hich blood pressure medications should be avoided in patients with ischemic stroke or subarachnoid hemorrhage because o f the increase in intracranial pressure associated w ith them?

Aortic Conditions 4. W hat are the indications for the repair of an abdominal aortic aneurysm (A A A ) in order to prevent future rupture? ° Diameter greater than________ in men o r_________ in women ° Increase in diameter by more than in a _________ month interval (should be receiving abdominal ultrasound every 6 months) ° Symptomatic (e.g., tenderness, pain in abdomen or back)

Arterial Conditions 5. W hat are the components of conservative medical management of peripheral artery disease (PAD)? ° ° ° °

Smoking cessation Glucose and BP control Daily exercise to increase collateral flow Cilostazol (Pletal) Improve flow to LE and decrease claudication More effective than pentoxifylline (Trental) Contraindicated if any heart failure due to increased mortality

° Daily aspirin or clopidogrel to reduce cardiovascular events ° Statin therapy to reduce cardiovascular events and increase pain-free walking distance

I 150]

|

Venous Conditions 6. W hat options are available to treat varicose veins? ° Weight reduction, avoidance of prolonged standing, leg elevation ° Compression stockings ° Sclerotherapy (injection of a substance directly into the vein that causes injury and thrombosis) ° Thermal ablation (laser therapy) ° Surgery involving ligation of the long saphenous vein or short saphenous vein

End of Session Quiz 7.

A patient comes to the E R following a motor vehicle collision, and C X R reveals widening o f the mediastinum. W h a t imaging study would confirm your diagnosis?

8.

W h a t study should be ordered for a patient suspected o f having an A A A (abdominal aortic aneurysm)?

9.

W h o should be screened for an abdominal aortic aneurysm?

10.

W h a t is the next step in the m anagem ent o f a patient w ith a D V T that has a high likelihood o f falling?

VASCULITIS

3 Question Warm-Up 1.

W h a t are the preferred vessels in the placement o f a Swan-Ganz catheter?

2.

A n EK G shows complete independence o f P waves and QRS complexes. W hat is the next step in the management?

3.

W hat two cardiovascular diseases are the biggest risk factors for C H F?

Thromboangiitis Obliterans (Buerger Disease) ° Vasculitis of small- and medium-sized peripheral arteries and veins ° Affects young male smokers (30s-40s) ° Clinical presentation: Intermittent claudication Superficial nodular phlebitis Raynaud phenomenon Gangrene, atifoSfftputation of the digits

° Treatment: smoking cessation

W hat are the diagnostic criteria for Kawasaki disease (mucocutaneous lymph node syndrome)? ° Fever (>40C or 104F) lasting at least five days in addition to 4 of the following 5 symptoms (CRASH): - _______________________ (bilateral, non-exudative, painless) - _______________________ on the trunk - _______________________ of the cervical lymph nodes - _______________________ and diffuse erythema of mucous membranes - _______________________ have edema with induration, erythema or desquamation ° Coronary artery aneurysms can occur within weeks of the illness onset, but are not included in the diagnostic criteria.

How is Kawasaki disease treated? ° IVIG (ideally within the first 10 days of illness) 0 High-dose aspirin, continued until 48 hours after fever resolution; followed by low-dose aspirin until inflammatory markers (platelets, ESR) return to normal (usually 6 weeks) ° Steroids are not indicated and are of no proven benefit. ° Echocardiogram in the acute phase and 6-8 weeks later

End of Session Quiz 7.

W h ich type o f vasculitis fits each o f the following descriptions? « Weak pulses in upper extremities ° Necrotizing immune complex inflammation of visceral/renal vessels ° Young male smokers ° Young Asian women ? Young asthmatics ° Infants and young children; involves coronary arteries ° Most common vasculitis ° Associated with hepatitis B infection » Occlusion of ophthalmic artery can lead to blindness ° Unilateral headache, jaw claudication

PEDI CARD IO LO G Y

PEDI CARDIOLOGY

3 Question Warm-Up 1.

Compare P T H , alkaline phosphatase, serum calcium and serum phosphate levels in patients w ith the following diseases: Serum C a2+

Serum Phos

Aik Phos

PTH

Paget Disease Osteomalacia/rickets Chronic renal failure Osteoporosis Osteopetrosis Primary hyperparathyroidism Hypoparathyroidism Pseudohypoparathyroidism

2.

W h a t autoimmune complication occurs 2-4 weeks after an M I?

3.

W h a t type o f psychotherapy is used to treat phobias, obsessive-compulsive disorder and panic disorder?

W hat is an Ebstein anomaly? ° Associated with maternal______________________ use ° ________________ aredisplacedintorightventricle,hypoplasticright ventricle, tricuspid regurgitation or stenosis ° Patent foramen ovale with a right-to-left shunt (80%) ° Dilated right atrium leads to an increased risk of: - Supraventricular tachycardia (SVT) Wolff-Parkinson-White syndrome (W PW ) ° Physical exam: - W idely split S2 - Tricuspid regurgitation ° Treatment: - PGE Digoxin Diuresis Propranolol for SVT

A 6-week-old infant presents to the pediatric emergency room for irritability and is found to have signs of left-sided heart failure. An EK G is interpreted as left-sided myocardial infarction. W hat is the m ost likely diagnosis?

PEDI CA RD IO LO G Y

End of Session Quiz 6.

W hat are the unique structures o f the fetal circulation that close after birth?

7.

W h a t medication is used to close a PDA?

8.

W hat are the abnormalities associated w ith tetralogy o f Fallot?

Pulmonology 1

URI

2

Low er R espiratory Infections p a rt 1

3

L ow er R espiratory Infections p a rt 2

4

A R D S and A sth m a

5

COPD

6

N eoplasm s and In terstitial L u n g Disease

7

P u lm o n ary Vascular Diseases

8

P leural Diseases

9

Sleep A pnea and P u lm o n ary Surgical C oncerns

10 Pedi P ulm onology p a rt 1 11 Pedi P ulm onology p a rt 2

c

7?

PULMONOLOGY

URi

3 Question Warm-Up 1.

W h a t is the treatm ent for cluster headaches?

2.

W h a t is the treatm ent for iron overdose?

3.

W h a t does a fall in systolic BP o f > 10 m m H g w ith inspiration indicate?

4. W hat are the diagnostic features of a peritonsillar abscess? W hat is the treatment? ° Infection between the tonsil and pharyngeal constrictors caused by Strep, pyogenes, Staph. aureus and/or Bacteroides spp. ° Severe sore throat, fever, muffled “hot potato” voice ° Classic diagnostic feature is obvious abscess on the tonsil or deflection of the uvula to the opposite side ° Other signs and symptoms include trismus (“lockjaw”) and drooling

Treatm ent ° Needle aspiration or I& D ° Pain meds + antibiotics - Amoxicillin/clavulanate Clindamycin

[ 159]

URI

End of Session Quiz 5.

W h at is a normal A-a gradient? W h a t causes a high A-a gradient?

6.

W hat are the feared complications o f streptococcal pharyngitis?

7.

W hat are the signs o f peritonsillar abscess?

LOWER RESPIRATORY INFECTIONS PART I

3 Question Warm-Up 1.

Short systolic m urm ur at the cardiac apex that decreases w ith squatting, is sometimes associated w ith a benign chest pain, and lasts only a few seconds:

2.

In w hich etiology o f restrictive cardiomyopathy is the pathology reversible w ith phlebotomy?

3.

Com pare the vasodilating effects o f the following: Nitroglycerine Dihydropyridine CCBs Hydralazine Nitroprusside

4. W hat are the most common causes of pneumonia and empiric antibiotics used for children in the age ranges of newborn, I -4 months, 4 months-4 years, and 5-15 years? Age

Causes

Empiric Treatment

Newborn

° ° ° °

Group B streptococcus (GBS) Escherichia coli Listeria monocytogenes Chlamydia trachomatis

Ampicillin and gentamicin +/- vancomycin (if M RSA is a concern) Erythromycin (Chlamydia trachomatis)

lm —4m

° ° o ° ° °

Respiratory syncytial virus (RSV) Chlamydia trachomatis Parainfluenza Bordetella Streptococcus pneumoniae Staphylococcus aureus

4m —4y

° ° ° ° °

RSV or other viruses* Streptococcus pneumoniae Haemophilus influenzae Mycoplasma pneumoniae Staphylococcus aureus

5 y -1 5 y

° ° ° °

Streptococcus pneumoniae Mycoplasma pneumoniae Chlamydophila pneumoniae O ther viruses*

+/- cefotaxime

1. Amoxicillin + clarithromycin 2. Azithromycin 3. Amoxicillin + doxycycline

* O ther viruses include rhinovirus, influenza, parainfluenza, adenovirus, coronavirus

W hat are the indications for pneumococcal vaccination in adults? ° 65 years old or older ° Serious long-term health problem such as heart disease, sickle cell disease, alcoholism, leaks of cerebrospinal fluid, lung disease, diabetes or cirrhosis ° Resistance to infection is lowered due to Hodgkin disease; multiple myeloma; cancer treatment with x-rays or drugs; treatment with long-term steroids; bone marrow or organ transplant; kidney failure; HIV/AIDS; lymphoma, leukemia or other cancers; nephrotic syndrome; damaged spleen or no spleen Cigarette smokers between the ages of 19 and 54

W hat antibiotics can be used in the treatment of Pseudomonas pneumonia? An antipseudomonal 3-lactam (piperacillin-tazobactam, cefepime, imipenem, meropenem, aztreonam) must be added to one of the following for at least 2 weeks for adequate coverage: ° Antipseudomonal quinolone (ciprofloxacin or levofloxacin) ° Aminoglycoside (tobramycin, gentamicin or amikacin) + azithromycin ° Aminoglycoside (tobramycin, gentamicin or amikacin) + antipseudomonal quinolone

W hat infectious agent is the cause of pneumonia based on the following lab test: ° ° ° ° °

Gram (+) cocci in clusters Gram (+) cocci in pairs Gram (-) rods in 80-year-olds Gram (+) cocci in neonate Gram (-) rods in neonate

End o f Session Quiz 8.

W h at is the empiric treatment for pneumonia in a 2-month-old? In a 2-year-old?

9.

W hat organism is associated with the following: Associated with atypical pneumonia in young adults Associated with atypical pneumonia in the elderly and very young Most common pneumonia in children (infant to 5 years old) Most common cause of pneumonia in neonates

10. A hospital has multiple admissions o f elderly patients presenting with atypical pneumonia. All were present at the grand re-opening o f an old bingo establishment that has an indoor waterfall. W h at is the likely causative organism?

11. Cystic fibrosis patients are at risk for acquiring pneumonia from which organism?

3 Question Warm-Up 1.

W h a t is the reversal agent for heparin?

2.

W h ich class o f antihypertensives is contraindicated in the following patients? • COPD ° Bilateral renal artery stenosis 0 Pregnancy 0 Advanced renal failure ° Gout

3.

W h a t is the most common cause o f hypertension in young women?

LOWER RESPIRATORY INFECTIONS PART 2

LOWER RESPIRATORY INFECTIONS PART 2

4. W hat x-ray and lab findings would lead you to highly suspect P C P as a cause of respiratory distress? ° CXR: diffuse, bilateral, interstitial infiltrates (most common finding) ° CD4 usually 220 (and a rising LD H despite appropriate treatment portends a poor prognosis)

5. W hat is the treatm ent for PCP? ° 21 days of antibiotics. Choices include: - TM P-SM X (IV or PO) Pentamidine (IV) - Primaquine (PO) + clindamycin (IV or PO)

[ 163]

End of Session Quiz 6.

Fill in the table of criteria used to determine when a P P D is considered positive: Induration

5mm

C onsidered Positive in th ese Situations

A” :

o 10mm

15mm

7.

, 4

W hich infectious agents fit the following descriptions? « Common cause of pneumonia in immunocompromised patients ° Most common cause of atypical/walking pneumonia ° Pneumonia associated with “currant jelly” sputum ° Can cause an interstitial pneumonia in bird handlers 4 Often the cause of pneumonia in a patient with a history of exposure to bats and bat droppings ? Often the cause of pneumonia in a patient who has recendy visited southern California, New Mexico or West Texas ° « 4 ° o

Common causative agent for pneumonia in alcoholics Associated with pneumonia acquired from air conditioners Most common cause of pneumonia in children 1 year old or younger Most Common cause of pneumonia in the neonate (birth-28d) Most common cause of pneumonia in young adults (including college students, military recruits and prison inmates)

° ° » o

Common cause of pneumonia in patients with other health problems Most common cause of viral pneumonia Causes woolsorters’ disease (a life-threatening pneumonia) Common pneumonia in ventilator patients and those with cystic fibrosis

ARDS AND ASTHMA

3 Question Warm-Up 1. A South Am erican im m igrant has cardiomegaly and achalasia. W h a t is the organism likely responsible for this patient’s disease?

2. D uring a surgical procedure, the anesthesiologist notices that the patient’s temperature has climbed quickly and her muscles are rigid. Recognizing this feared complication, the doctor should administer which medicine?

3. W h a t is the next step in the evaluation o f penetrating injuries to the different zones o f the neck?

4. W hat treatm ents are available when managing a patient with Acute Respiratory Distress Syndrome (ARDS)? ° Treat the underlying disease and provide supportive care in the ICU ° Mechanical ventilation with low tidal volumes (to minimize injury) and adequate PEEP (to recruit collapsed alveoli) ° Conservative fluid management to reduce pulmonary edema. Goal CVP 4-6 H 20 . (Furosemide and albumin may also be helpful to prevent pulmonary edema) ° Prone positioning improves oxygenation but does not improve survival ° Minimize oxygen consumption by preventing fever (anti-pyretics), minimizing anxiety and pain (sedative and analgesics), and limiting respiratory muscle use (paralytics), particularly in cases of asynchrony of the ventilator and patient’s efforts ° Transfuse blood only if Hgb drops below 7 g/dL as transfusion may T risk of death in ARDS patients

5. W hat methods of providing oxygen to a patient can be used in order to deliver a specified percentage of F i0 2? ° Nasal canula ° Face mask ° Non-rebreather » CPAP ° Mechanical vent

—* 24-40% F i0 2 (F i0 2 increases 3% for every 1 L/min up to 6 L/min) —> 50-60% F i0 2 —* 60-95% F i0 2 -» 80% F i0 2 up to 100% F i0 2

ARDS AND ASTHMA

I 166 |

End of Session Quiz 6.

In a patient with pulmonary edema, how can pulmonary capillary wedge pressure (PC W P) distinguish A R D S from cardiogenic edema?

7.

W h at are the diagnostic characteristics o f Acute Respiratory Distress Syndrome (ARDS)?

8.

W h a t defines mild persistent asthma? W h a t is the outpatient management?

COPD

3 Question Warm-Up 1.

W h a t heart sounds are considered benign when there is no evidence o f disease?

2.

A young wom an presents w ith amenorrhea, bradycardia and abnormal body image. vVhat is the diagnosis?

3.

A patient presents w ith a history o f interm ittent tachycardia, w ild fluctuations in blood pressure, headache, diaphoresis and panic attacks. W h a t is the diagnosis?

4. C O P D Staging ° ° o °

G O LD G O LD G O LD G O LD

I234-

FEV, FEV, FEV, FEV,

> 80% predicted (mild) 50-80% (moderate) 30-50% (severe) < 30% (very severe)

5. C O P D Management °

Category A (G O LD I or 2 with mild or infrequent symptoms) - short-acting bronchodilator (albuterol, atrovent) ° Category B (G O LD I or 2 with moderate to severe symptoms) - above + long-acting bronchodilator (j32-agonist or anticholinergic (tiotropium, ipitropium)) • Category C (G O LD 3 or 4 with mild or infrequent symptoms) - above + inhaled steroids ° Category D (G O LD 3 and 4 with moderate to severe symptoms) - above +/theophylline + home 0 2 if pulse-ox < 88%, +/- phosphodiesterase-4 inhibitor (roflumilast)

Bronchiectasis 6. W hat are the possible etiologies for bronchiectasis? ° ° ° °

Unknown etiology in 50% of patients Cystic fibrosis Immunodeficiency Dyskinetic cilia: Kartagener syndrome (dextrocardia, sinusitis, bronchiectasis), autosomal dominant polycystic kidney disease (ADPKD) ° Pulmonary infections (TB, fungal or lung abscess) ° Obstruction (e.g., foreign body aspiration, tumor, lymph nodes) ° Other etiologies: Young syndrome, foreign body aspiration, rheumatoid arthritis, Sjogren syndrome, allergic bronchopulmonary aspergillosis, cigarette smoking

End of Session Quiz 7.

W h a t is the hallmark pulmonary function test finding in C O PD ?

8.

A patient has an FEVj at 55% o f predicted and has frequent episodes o f shortness o f breath when walking on level ground. W h a t medications would be appropriate for this patient?

9.

A t what point do patients with chronic C O P D qualify for home 0 2?

3 Question Warm-Up 1.

W h a t are the “H ” causes o f PEA?

2.

W h a t are the “T ” causes o f PEA ?

W ssm iS & m l& Q ti *

3.

4.

.

i

«t-

f

\

I

t’ V

,

V

i

NEOPLASMS AND INTERSTITIAL LUNG DISEASE

NEOPLASMS AND INTERSTITIAL LUNG DISEASE

■■

W h a t is the treatm ent for endocarditis?

How is idiopathic pulmonary fibrosis diagnosed? ° CXR possible findings —normal in 10%, reticular pattern, nodular pattern or honeycomb lung (poor prognosis) ° High-resolution C T scan —greater diagnostic accuracy than plain films ° Serum markers including SP-A, SP-B, MCP-1 and KL-6 ° Pulmonary function testing (PFTs) reveal restrictive lung disease with decreased TLC, FRC and RV ° Lung biopsy is required to make the definitive diagnosis and determine the stage of disease

[ 169]

NEOPLASMS AND INTERSTITIAL LUNG DISEASE

E n d o f Session Q uiz 5.

6.

W hat is the next step in the work-up o f a patient with a solitary pulmonary nodule?

W hich type of lung cancer is associated with each of the following paraneoplastic syndromes? s ? Elevated ACT 11 —> glucocorticoid excess —> Cushing syndrome ° Elevated PTH-related peptide —>hypercalcemia Elevated AD H —» SIADIT —» hyponatremia > Antibodies to presynaptic Ca2*channels —>Lambert-Eaton syndrome

7.

W h at is the initial treatment o f a localized non-small cell lung cancer?

8.

W h a t are the classic radiological findings in idiopathic pulm onary fibrosis (IPF)?

9.

W h a t is the treatment for idiopathic pulmonary fibrosis (IPF)?

10. W h ich type o f pneumoconiosis matches each o f the following descriptions? Progressive fibrosis Increased risk of TB Associated with working with electronics; causes increased cancer risk M alignant mesothelioma and bronchogenic carcinoma

PULMONARYVASCULAR DISEASES

3 Question Warm-Up 1.

W h a t heart defect is associated w ith each o f the following disorders? ° Chromosome 22qll deletion ° Down syndrome ° Congenital rubella ° Turner syndrome ° Marfan syndrome

2.

W h a t is the treatm ent o f cardiogenic shock?

3.

W h a t is the next step in the m anagement o f a positive PPD ?

"O c r~ 3 O z > -< JS oo n c r~ > 70 O CO m >

4. W hat studies are helpful in the diagnosis of pulmonary embolism? ° ABG —respiratory alkalosis due to hyperventilation but low P 0 2 due to hypoxia ° D-dimer —a normal d-dimer makes PE much less likely and an elevated d-dimer is nonspecific 0 U/S of LE veins to rule out DVT - presence of DVT makes PE more likely, absence of DVT doesn’t tell you anything ° CXR —usually normal but may have atelectasis, pleural effusion or “Hampton’s hump” (wedge-shaped opacification at the distal lung fields) ° EKG changes - most commonly sinus tachycardia +/- nonspecific ST and T wave changes, and very rarely the classic S1Q3T3 (wide S in lead I, large Q_and inverted T in lead III) ° V/Q_scan —useful and accurate only in confirming low or high clinical probability ° C T scan with contrast (AKA: C T PE protocol, or C T pulmonary angiography) - most commonly used method to diagnose or rule out a PE, but not 100% accurate ° Pulmonary angiogram —the gold standard, but more invasive and rarely performed

5. Vasodilators used in pulmonary hypertension ° ° ° °

Prostanoids —epoprostenol, treprostinil, iloprost Endothelin receptor antagonists - bosentan, ambrisentan Cyclic G M P phosphodiesterase inhibitors —sildenafil CCB —nifedipine

[ 171 ]

End of Session Quiz 6.

W h at is a classic sign o f P E on CXR?

7.

W h a t is the next step in the diagnosis o f a patient that you suspect has pulmonary H T N because o f his symptoms o f JV D , loud S2, dyspnea and fatigue?

8.

A former smoker with chronic C O P D and chronic C H F presents w ith acute shortness o f breath and hypoxemia. W h a t C X R findings m ight suggest pulmonary edema as the cause o f his dyspnea?

9.

W h a t imaging studies are most helpful in diagnosing a PE?

10.

W h a t is the primary drug category used to treat pulmonary H T N ?

PLEURAL DISEASES

3 Question Warm-Up 1.

W h a t are the indications for surgical repair o f an abdominal aortic aneurysm?

2.

W h a t is the next step in the management o f a child w ith severe asthma exacerbation and persistently low oxygen saturation despite medication?

3.

W h a t is the next step once a brain tum or has been identified on C T or M R I o f the head?

End of Session Quiz 4.

W h a t study can help you determine whether pleural effusion is loculated or freeflowing in the thorax?

5.

A patient presents to the E R in respiratory distress following a M V C . C X R shows pleural effusion. W h a t is the next step in the management o f this patient?

6.

W h a t might cause a transudative pleural effusion?

7.

W hat size pneumothorax requires a chest tube placement?

SLEEP APNEA AND PULMONARY SURGICAL CO N CERN S

SLEEP APNEA AND PULMONARY SURGICAL CONCERNS

3 Question Warm-Up 1.

W h a t are the W s o f post-op fever?

2.

W hich type o f lung cancer is associated w ith hypercalcemia?

3,

A patient presents to the E R after an M V C with hypertension, bradycardia and abnormal respirations. After ABCs, w hat is the next step in the management?

4. W hat is the treatm ent for obstructive sleep apnea (OSA)? ° _____________________ if overweight or obese ° Avoidance o f____________ and other CNS depressants or sedatives ° ____________________ at night —first-line but poor compliance ° Oral appliance to protrude the mandible forward or prevent posterior tongue displacement - Not as effective as CPAP ° If excessive daytime sleepiness despite therapies —> modafinil qAM ° Surgical options: Tonsillectomy & adenoidectomy - especially in children or if hypertrophied Us pa r pharyngopla ' (UPPP) - most common, surgery in adults for O SA Gemoglossus advancement Maxillaryernandibular advancement

[. I /d ] !

W hat are the components of rapid sequence intubation? Rapid sequence intubation requires an acutely unstable patient needing an airway rapidly + rapid-acting sedative and paralytic to create optimal intubating conditions enabling rapid control of the airway. ° Preparation ° Pre-oxygenation ° Pretreatment (toblunt ICP elevations) Fentanyl (to blunt the pain response and BP elevations) _____________________ (to decrease airway secretions and prevent bradycardia in children during intubation) ° Paralysis with induction Rapid induction agents: etomidate, ketamine, midazolam or propofol Rapid paralytics: succinylcholine, rocuronium ° Protection and Positioning (including______________ pressure to help prevent aspiration) ° Placement of E T tube ° Post-intubation management - secure E T tube, CXR, set ventilator

End of Session Quiz 6.

W h a t medication is used prior to intubation in head injury patients?

7.

W h a t substances should be avoided in patients w ith obstructive sleep apnea?

PEDI PULMONOLOGY PART I

3 Question Warm-Up 1.

W h at is the usual time frame for stopping warfarin prior to surgery?

2.

W h at are the indications for operating on an abdominal aortic aneurysm?

3.

W hen might subclinical mitral stenosis from rheumatic heart disease become clinically apparent?

Croup 4. W hat are the classic signs and symptoms of croup? ° Inflammation of larynx, trachea and/or bronchi —>____________________, respiratory distress, upper airway obstruction w ith____________________ 0 Symptoms are worse at night 0 75% caused by parainfluenza viruses 0 6% incidence annually in children under 6 years old ° Common cause of hospitalization in children younger than 4 years old (esp. in fall/winter months) ° Course: 12-72 hours of mild fever and coryza —> hoarseness and barking cough —» peak resp distress at 24-48 hours —* resolution in 1 week

5. W hat is the treatment for croup? W hat are the indications for hospitalization? ER Treatment ° Change ambient temperature ° Humidified oxygen mask ° Dexamethasone 0.6mg/kg IM or PO once. Prednisolone may be administered for several days because of shorter half-life. ° ____________________ ql-2 hours 0 Admission criteria: progressive stridor or stridor at rest, no improvement 5 hours after steroids, hypoxia/cyanosis, resp distress, unclear diagnosis ° Home treatment: cool mist humidifier (esp. at night) (commonly used, but no proven benefit)

6. W hat are the characteristic features of epiglottitis? ° Acute-onset fever, severe sore throat, dysphagia, drooling and “toxic” appearance ° Progression to airway obstruction with inspiratory distress, muffled speech (“hot potato” voice) and cyanosis ° “Tripoding” (arms back, trunk leaning forward, neck hyperextended and chin thrust forward) in an attempt to maximize airway diameter ° CBC: leukocytosis with bandemia ° Lateral neck X-ray: “thumb sign” (enlarged epiglottis) ° Visualization of the swollen epiglottis in young children simply by depressing the tongue with a tongue blade Anxiety and increased respiratory effort with this portion of the exam may cause cardiopulmonary arrest and should be performed cautiously and only if prepared to intubate ° Direct or indirect laryngoscopy may be needed in older children to confirm the diagnosis: cherry-red, swollen epiglottis

PEDI PULMONOLOGY PART I

Epiglottitis

7. W h at is the treatm ent for epiglottitis? ° Minimize the child’s anxiety during the interview ° Intubation or tracheostomy as soon as possible upon making the diagnosis Nasotracheal intubation with an ET tube 0.5-1 mm smaller than what would normally be used is ideal in these patients - Average duration is 2-3 days ° Culture and sensitivity from blood and epiglottis surface ° Empiric IV antibiotics (Oxacillin, nafcillin, cefazolin, clindamycin or vancomycin) + (ceftriaxone or cefotaxime) Duration 7-14 days

( 177 ]

Bronchiolitis 8. W hat is the classic presentation of RSV bronchiolitis? ° Starts with 1-3 days of mild URI symptoms: rhinorrhea, mild cough and mild fever +/otitis media ° Cough with prolonged expiration, expiratory wheezing, crackles and hyperresonance on percussion ° Possible respiratory distress with nasal flaring, tachypnea, retractions, grunting ° Possible apnea in infants (indication for mechanical ventilation) ° Complete resolution usually takes one month

9. W hat is the treatment for RSV bronchiolitis? ° Supportive care such as 0 2 and IVF as needed. ° Albuterol nebs o r____________________. If good results seen with either race Epi or albuterol, then continue, otherwise Stop treatment. Some studies showed a benefit of albuterol or epinephrine neb in infants under 12 months ° are N O T effective in bronchiolitis (NEJM 2007;357:331) ° ____________________ use in children is controversial, expensive, and generally is not used ° Hospitalize if: Toxic appearing, poor feeding, lethargy, dehydration Respiratory distress (nasal flaring, intercostal retractions or cyanosis) Hypoxemia (< 929 on room air) Apnea Parent is Unable to care for the child at home

End of Session Quiz 10.

W h at is the treatment for epiglottitis?

11.

W h at is the treatment for croup?

12. W h at is the treatment for RSV bronchiolitis?

PEDI PULMONOLOGY PART 2

! 1 TmJ i— ”U C

r~

3 Question Warm-Up 1.

W h a t medication combination is used in the treatment o f T B meningitis?

2.

W h a t is the preferred diagnostic test for a pulm onary embolism?

3.

W h a t are the indications for surgical parathyroidectomy for a parathyroid adenoma?

o z o 1 — o o -< TJ > 73 _1 N>

Respiratory Distress Syndrome of the Newborn 4.

How is the lecithin:sphingomyeiin (L:S) ratio used in determining fetal lung maturity? As the lungs mature a t __________ weeks, the amount of lecithin produced increases while sphingomyelin remains constant. ° A L:S ratio o f__________ obtained by amniocentesis indicates fetal lung maturity ° If the ratio is 1.5-1.9,50% of these infants will develop respiratory distress syndrome (RDS) ° A ratio o f__________predicts a 75% chance of RDS development (Samples containing blood or meconium should be discarded due to unpredictable variations. In these cases a phosphatidylglycerol (PG) measurement is most reliable.)

5. W h at chest X -ra y characteristics distinguish neonatal respiratory distress syndrome from transient tachypnea of the newborn? ° RDS —low lung volumes, diffuse_______________ appearance w ith_______________ ° T T N - increased lung volumes with flattening of the diaphragms, prominent vascular markings from the hilum (sunburst pattern), fluid streaking in interlobular fissures, +/pleural effusions

6. W hat is the treatm ent for neonatal respiratory distress syndrome? ° ________________administration (from porcine or bovine source) ° Continuous positive airway pressure (CPAP) or intubation and mechanical ventilation - CPAP is preferred in larger babies (> I500g), or if active, breathing spontaneously and not in respiratory failure - Mechanical ventilation if respiratory failure

[ 179]

"U c r~ 2 o z

o l~ o o

PEDI PULMONOLOGY PART 2

Cystic Fibrosis 7. How is cystic fibrosis diagnosed? ° Sweat chloride test: sweat chloride concentration >___________ on two or more occasions Performed by a lab familiar with the test Considered the gold standard to diagnosing CF ° Genetic testing for___________ gene mutations Identifies about 90% of CF cases ° Nasal transepithelial chloride secretion: measures abnormalities in ion transport across the nasal epithelium

8. W hat are the general strategies for treating the pulmonary component of cystic fibrosis? ° ° ° ° °

9.

p 2 agonist (albuterol, salmeterol, formoterol) DNase I (dornase alfa) to decrease sputum viscosity Hypertonic saline for chronic cough Physiotherapy (such as aerobic exercise) for increased mucus clearance Azithromycin used liberally prn when lung function decreases in order to slow the decline in lung function and treat any Pseudomonas aeruginosa

W hat are the general strategies for managing the GI component of cystic fibrosis?

End of Session Quiz 10. W h at is the typical C X R appearance o f newborn respiratory distress syndrome?

11. Shortly after birth, a child has stridor, wheezing, and shortness of breath despite medical therapies. W h a t is likely to be causing this patient s symptoms?

12. W h a t are the general strategies for treating the pulmonary component o f cystic fibrosis?

G astroenterology 1 G I Infections V iral H epatitis

3

O ral and Esophageal C onditions

4

G astric C onditions

5

Sm all In testin e p a rt 1

GASTROENTEROLOGY

2

6 Sm all In testin e p a rt 2 7 L arge In testin e p a rt 1 8 L arge In testin e p a rt 2 9

C olorectal C ancer and G I Bleeding

10 Pancreatic D iseases

■HRH

I life r ■ B ll

11 B iliary D iseases p a rt 1

A R M l■ 12 B iliary D iseases p a rt 2

8 1

■ M B B

13 A lcoholic Liver D isease and C irrhosis 1 i1 14 O th e r H epatic D iseases 15 P edi G I D isorders



i

p

i

W B ^ M ISiIlgipS S iS i si

Gl INFECTIONS ..............

I “n m immediate improvement in urine flow SE: dizziness, postural hypotension, fatigue, asthenia. To reduce SE, dose qHS and titrate dose upward slowly over time (weekly) ° Tamsulosin (Flomax) (selective a iA-blocker) - fewer SE than nonselectives, has no antihypertensive effects, costs a bit more ° 5Q-reductase inhibitors: finasteride (Proscar), dutasteride (Avodart) Slowly reduces dihydrotestosterone levels —> 20% decrease in prostate volume over 3-6 months May work best in those with a palpably enlarged prostate Best to use in combination with an a-blocker Decreases PSA levels by about 50% —» when using PSA to screen for prostate cancer, double the value to attain the actual number. SE: decreased libido, ejaculatory disorder; impotence Surgical Intervention ° Indications for surgery: failure of medical therapy, refractory urinary retention, inability to express urine without a catheter, recurrent infection, persistent hematuria, bladder stones or renal insufficiency ° TU RP —transurethral resection of the prostate. Most common surgery for BPH. Symptomatic improvement in 88%. SE include excessive bleeding (1%), retrograde ejaculation (70%), impotence (14%), partial incontinence (6%), total incontinence (1%) ° Open prostatectomy —symptomatic improvement in 98%, but is invasive and associated with more morbidity than TURP. Usually reserved for those with extremely large prostate or structural problems (protrusion into bladder, large bladder stone, urethral diverticulum). ° Other interventions: transurethral incision of the prostate (TUIP), transurethral microwave thermotherapy (TUMT), transurethral vaporization of the prostate (TUVP), transurethral electrovaporization of the prostate (TVP), transurethral needle ablation of the prostate (TUNA)

W hat is the next step in the management of a 65-year-old man that presents to the ER with inability to urinate and painful bladder distention? ° Decompression of bladder with 14-18 French Foley catheter (may downsize to 10-12 French if necessary) ° If h/o BPH, may require a cath with a firm Coude tip to “power through” the narrowed urethra ° If unable to pass urethral cath, then__________________ (usuallyunderUS guidance) ° If unable to pass urethral cath and no one trained in suprapubic cath placement will be available for hours, then____________________________________

Prostate Cancer 7.

A 60-year-old man presents to the clinic for a well-male exam. On digital rectal examination, a hard nodule is palpated on the prostate. Lab work-up shows an elevated PSA . W hat is the next step in the management of this patient?

End of Session Quiz 8.

W h a t is the treatm ent for urethritis in men?

9.

W h a t are classic symptoms o f benign prostatic hyperplasia (BPH)?

10.

11.

N am e the tw o classes o f prescription medications used to treat B PH .

In a patient suspected o f having prostate cancer, w hat m ight an elevated alkaline phosphatase indicate?

PART 2

MALE REPRODUCTION PART 2

MALE REPRODUCTION

3 Question Warm-Up 1.

W hat is the classic (but rare) EK G finding in pulmonary embolism?

2.

W h a t is the most common EK G finding in pulmonary embolism?

3.

I f pancreatitis is due to gallstone obstruction o f pancreatic duct, what should be done after the pancreatitis has passed?

Testicular Torsion 4.

How can testicular torsion be differentiated from epididymitis?

Torsion

Epididymitis

Onset

Acute, abrupt and often associated with a physical activity

Subacute and may be associated with STDs and/or anal intercourse

Infection

No signs of infection

Possible signs of STD (urethral discharge, fever, dysuria, erythema)

Visual changes

Testicle may be raised and horizontal

Testicle in normal position and lie

Cremasteric reflex

Absent

Present

Ultrasound

Compromised blood flow

Normal blood flow

Support

5. W hat is the difference between the treatment of testicular torsion vs. epididymitis? ° Torsion Surgical detorsion with bilateral orchiopexy within 6 hours ° Epididymitis Under 35yo = GC/Chlamydia —> ceftriaxone IM then doxycydine xIO days Over 35yo or h/o anal intercourse = Enterobacteriaceae —> fluoroquinolone x 10-14 days

[248]

Male Infertility 6.

W h a t are the characte ristic features of a varicocele? ° Dilation of pampiniform plexus in the scrotum (“testicular mass”) ° Dull, aching scrotal pain usually on the left (left-sided varicoceles are 10 times more common than right-sided) ° Right-sided varicocele may point to a _____________________ ° Testicular atrophy on the affected side ° Infertility is common —varicoceles are present in 25% of infertile men vs. only 11% of fertile men. ° Color Doppler ultrasound shows retrograde flow to the scrotum

Impotence 7. W h at are the proper steps in the evaluation of a patient presenting with erectile dysfunction? ° History: - Onset and duration Symptoms of depression (SIG E CAPS) Medication and drug use Psychological stressors and interpersonal conflict If dysfunction is situational (e.g., only with one particular partner) Presence of nocturnal or early-morning erections (absent if organic cause, present if psychogenic) ° Physical exam components: Anal tone (neuro dysfunction) - Lower extremity sensation (neuro dysfunction) Cremasteric reflex (neuro dysfunction) Femoral and peripheral pulses (vasculogenic cause) - Penis (Peyronie disease) - Testes (hypogonadism) - Secondary sexual characteristics (hypogonadism) Visual fields (pituitary tumor) - Gynecomastia (prolactinoma) ° Serum lab tests: total testosterone, prolactin, TSH, +/- PSA ° If vasculogenic —>cardiac stress test to assess for cardiac endothelial damage as well

8. W h at medications are known for causing erectile dysfunction? ° ° ° ° ° ° °

Most antidepressants, especially SSRIs Spironolactone Sympathetic blockers: clonidine, guanethidine, methyldopa Thiazide diuretics, ^-blockers Ketoconazole Cimetidine (but not ranitidine or famotidine) Antipsychotics

PART 2 MALE REPRODUCTION

9.

W h at are the available treatm ents for a patient with erectile dysfunction? ° First-line: Phosphodiesterase inhibitors: sildenafil (Viagra), vardenafil (Levitra), tadalafil (Cialis) ° Second-line: Penile self-injectable drugs: papaverine, phentolamine, alprostadil Vacuum and constriction devices ° Third-line: Penile prosthesis implantation Other: Androgen replacement if hypogonadal

End of Session Quiz

[ 250] I

10.

W h a t is the treatment for epididymitis?

11.

W h at is the most common germ cell tum or o f the testis?

12.

W h a t lab work is included in the work-up for erectile dysfunction?

PEDI UROLOGY

1 "O m -

C

50 o1— o o

3 Question Warm-Up 1.

W h ich type o f vasculitis fits the following descriptions? ° Weak pulses in upper extremities ° Necrotizing granulomas of lung and necrotizing glomerulonephritis ° Necrotizing immune complex inflammation of visceral/renal vessels ° Young male smokers ° Young Asian women ° Young asthmatics

2.

W h a t is the treatm ent for superior vena cava syndrome?

3.

W h a t tw o disorders should come to m ind when a neonate has meconium ileus?

4. W hat is the classic presentation of the most common renal tum or in children? W ilm s tumor: ° Most common age:__________________ ° Palpable flank mass (most common presenting symptom) ° Abdominal pain (30%) ° Hematuria (12-25%) Hypertension ( Possibly multiple other associated congenital anomalies, including WAGR syndrome - Wilms tumor Aniridia - GU abnormalities - Retardation (mental)

PEDI UROLOGY

5. A t what age should nocturnal enuresis be treated? W hat are the treatment options? ° Enuresis cannot be diagnosed until 5 years of age (chronological and developmental) ° Treatment is usually delayed until the child is at least 7 years of age ° First-line: behavioral interventions: Start toilet training if not yet attempted Motivational therapy (e.g., star charts) Restrict fluids before bed (with a compensatory increase in daytime fluids) Nighttime chaperone to the toilet or scheduled wakening to void using alarm clock Enuresis alarm (pad with alarm device) in bed for classic conditioning. This is most effective long-term therapy ° Second-line: pharmacologic interventions: High likelihood of recurrence upon discontinuation Imipramine for short term (up to 6 weeks) Desmopressin (DDAVP) orally Indomethacin suppository

End of Session Quiz

[252 ]

6.

W h a t is the most common physical finding/presenting symptom o f a child with a W ilm s tumor?

7.

A 4-year-old boy is diagnosed w ith a U T I. W h a t study should be performed next?

8.

W h a t are the recommended therapies for nocturnal enuresis?

9.

A male newborn has a distended palpable bladder and oliguria. W h a t is the most common cause o f congenital urethral obstruction?

H eme/O nc 1

A nem ia p a rt 1

2

A nem ia p a rt 2

3

A nem ia p a rt 3

4

G enetic D isorders o f H em oglobin

5

L eukocyte D isorders & H ypersensitivity

6

T hrom bocytopenia

7

C oagulopathies and H ypercoagulable States

8

H em atologic Infections

9

H IV

10 H IV T reatm en t 11 M yelom a and Lym phom a 12 L eukem ia 13 Pedi H em e /O n c

ANEMIA PART I

ANEMIA PART I

3 Question Warm-Up 1.

W h ich is more ethically unfavorable in a D N R patient: withdrawing lifesustaining care or withholding care?

2.

A 25-year-old m an is diagnosed w ith a solitary testicular mass by ultrasound. W h a t is the next step in the management?

3.

W h a t are the classic findings o f Henoch-Schonlein purpura (HSP)?

4. W hat are the characteristic findings of hereditary spherocytosis? ° Jaundice and gallstones o

M C H C (mean corpuscular ° Anemia with reticulocytosis and hemoglobin concentration) ° Higher incidence of pseudohyperkalemia as RBCs lyse after blood draw and intracellular potassium leaks ° Peripheral smear reveals _______________________ ° Positive osmotic fragility test

5. W h at is the treatm ent for hereditary spherocytosis? ° Folic acid lm g daily ° Red blood cell transfusions in cases of extreme anemia o ____________ in moderate to severe disease

[255]

End of Session Quiz 6.

Increased body temperature, acidosis and exercise shift the hemoglobin-oxygen dissociation curve in which direction? W h at effect does this have on oxygen delivery to the tissues?

7.

W h a t is the cause o f anem ia th at develops after taking a sulfa drug?

8. W h a t lab markers suggest anemia due to hemolysis?

9.

In hemolytic anemia, why is the serum haptoglobin level decreased? W h y is the serum lactate dehydrogenase (L D H ) increased?

ANEMIA PART 2

3 Question Warm-Up 1.

W h a t is the treatm ent for each o f the following diarrheal illnesses? 0 Entamoeba histolytica ° Giardia lamblia ° Salmonella ° Shigella ° Campylobacter

2.

W h a t is the treatm ent for m ild persistent asthma?

3.

W h a t is C harcots triad in cases o f cholangitis?

End of Session Quiz 4.

Com pare the serum iron, ferritin, and transferrin levels in iron deficiency anemia to those in anemia o f chronic disease.

5.

A n elderly m an is in the office for evaluation o f fatigue and exertional shortness o f breath. H e is fit for his age, but he says that he isn’t able to be as active recently. H e has no fever or symptoms suggesting infection. O n exam, his conjunctivae are pale and a stool guaiac is positive. U ntil proven otherwise, this patient should be considered as having w hat disease?

6.

W h a t would you see on a blood smear o f a patient w ith anemia due to lead poisoning?

ANEMIA PART 3

3 Question Warm-Up 1.

A patient presents w ith hypertension, depression and kidney stones. W h a t is the most likely underlying diagnosis?

2.

A n E G D with biopsy in a 65-year-old m an reveals gastric cancer. W h a t is the next step in the management?

3.

W h a t are the signs and symptoms o f pyelonephritis?

End of Session Quiz 4.

W hich RBC disorder is associated w ith each o f the following findings: ° Schistocytes (fragmented RBCs) ° Acanthocyte (spur cell) ° Bite cell ° Basophilic stippling of RBCs 0 Peripheral neuropathy + ringed sideroblasts in bone marrow ° Hypersegmented neutrophils ° Heinz bodies (denatured Hgb in RBC)

5.

Given each o f the following presentations, provide the type o f anemia and whether it is macro-, norm o- or microcytic:

Presentation

Anemia type

MCV

Mental status change, neuropathy, constipation Heavy menses, strict vegetarian, ice pica Dark urine, jaundice, hepatosplenomegaly Alcoholic, malnourished

6.

W h a t virus can cause aplastic anemia as well as erythem a infectiosum (“fifth disease”)?

GENETIC DISORDERS OF HEMOGLOBIN

3 Question Warm-Up 1.

W h a t test is used to rule out urethral injury?

2.

A 19-year-old m an presents w ith a palpable flank mass and hematuria, and renal ultrasound shows bilateral enlarged kidneys w ith cysts. W h a t brain anomaly is associated w ith this condition?

3.

W h ich lipid-lowering agent matches each o f the following descriptions? ° SE: Facial flushing ° SE: Elevated LFTs, myositis ° SE: G l discomfort, bad taste ° Best effect on H D L ° Best effect on triglycerides ° Best effect on LDL/cholesterol ° Binds C. diff. toxin

End of Session Quiz 4.

W h ich type o f thalassemia is m ost commonly associated w ith patients o f M editerranean descent? W ith patients o f African or Asian descent?

5.

W h a t complication occurs in 10% o f patients w ith sideroblastic anemia?

6.

W h ich organism may be responsible for osteomyelitis in a sickle cell patient?

7.

W h ich vaccines are particularly im portant in children w ith sickle cell disease?

8. W hat medication is used in the long-term management of sickle cell anemia?

LEUKOCYTE DISORDERS AND HYPERSENSITIVITY

3 Question Warm-Up 1.

W h at drugs are known for causing elevated prolactin levels?

2.

W hat substances are known to cause hemolysis in patients with G 6PD deficiency?

3.

W h at would you expect with eosinophilic casts found in the urine?

W hat is the differential diagnosis for serum eosinophilia? D NAAACP ° Drugs (NSAIDs, penicillins/cephalosporins) ° Neoplasm ° Allergies, asthma (Churg-Strauss), allergic bronchopulmonary aspergillosis ° Adrenal insufficiency (Addison disease) ° Acute interstitial nephritis ° Collagen vascular disease (PAN, dermatomyositis) ° Parasites (such as Strongyloides, and Ascaris —* Loeffler eosinophilic pneumonitis) (Other causes: HIV, hyper-IgE syndrome, hypereosinophilic syndrome, coccidioidomycosis and numerous other potential causes) W h a t is the treatm ent for anaphylaxis?

End of Session Quiz 6.

W h a t is the next step in the management o f a patient w ith febrile neutropenia due to chemotherapy?

7.

W h ich type o f infection will cause an eosinophilia?

8.

W h ich immunoglobulin class is associated w ith eosinophilia?

9.

Goodpasture glomerulonephritis results from autoantibodies targeting the glomerular basement membrane (GBM). W hich hypersensitivity reaction is this, and how can it be treated?

10.

A patient presents w ith red marks on her ear lobes and left wrist, and is ulti mately diagnosed w ith nickel allergy. W h a t type o f hypersensitivity reaction is this?

11.

W h a t is the m ost im portant medication in the treatm ent o f anaphylaxis?

THROMBOCYTOPENIA

3 Question Warm-Up 1.

A 45-year-old man presents with acute-onset flank pain and hematuria. W hat is the most likely etiology?

2.

W h a t is the most likely cause o f aortic stenosis in a 50-year-old patient?

3.

W hich type o f bias is introduced when screening detects a disease earlier and thus lengthens the time from diagnosis to death?

W hat drugs are known for causing thrombocytopenia? ° ° ° ° ° ° °

Heparin (H IT = heparin-induced thrombocytopenia), abtiximab (GP Ilb/IIIa inhibitor) Carbamazepine, phenytoin, valproate Cimetidine Acyclovir, rifampin Sulfonamides (e.g., sulfasalazine, TMP-SMX) Procainamide, quinidine Quinine, gold compounds

End of Session Quiz 5.

W h a t is the mechanism o f action o f each o f the following drugs? 0 Streptokinase ° Aspirin ° Clopidogrel ° Abciximab ° Tirofiban ® Ticlopidine ° Enoxaparin ° Eptifibitide

6.

W h a t is the classic pentad for throm botic thrombocytopenic purpura

7.

W h a t lab test is used to monitor warfarin? Heparin? L M W H ?

COAGULOPATHIES AND HYPERCOAGULABLE STATES

COAGULOPATHIES AND HYPERCOAGULABLE STATES

3 Question Warm-Up 1.

You suspect your patient has gastric cancer. D uring the physical exam you palpate in two places for enlarged lymph nodes associated with this disease. W here will you palpate, and what are the names o f these enlarged nodes?

2.

A 2-year-old boy presents with painless rectal bleeding. W h a t diagnosis do you suspect, and what study would you order to confirm the diagnosis?

3.

A patient tells you she does not want to know the result o f her recent lung biopsy, but the family is begging you to tell them. To whom do you tell the results?

4. W hat is the treatment for the most common inherited bleeding disorder? von Willebrand factor deficiency treatments: ° ________________________(which increases vW F secretion) is first-line for acute bleeding ° Cryoprecipitate or Factor VIII concentrates for severe or refractory bleeding 0 ________________________for menorrhagia ° Avoid________________________ and other platelet inhibitors

5. W hat are the most common causes of DIC?

W h a t are the inherited diseases of hypercoagulation? ° ° ° °

------------------------------Antithrombin deficiency Protein C deficiency Protein S deficiency

—most common (40-50%)

° Prothrombin gene mutation (prothrombin G20210A) ° Hyperhomocysteinemia (M TH FR gene mutation) ° Rarer disorders: dysfibrinogenemia, plasminogen deficiency

End of Session Quiz 7.

W h a t lab changes would you see in each o f the following diseases?

Disorder

Platelet Count

Bleeding Time

PT

PT T

H U S o rT T P Hemophilia A or B von W illebrand disease D IC W arfarin use End stage liver disease Aspirin use

8.

W h a t is the treatm ent for von W illebrand disease?

9.

W h a t are th e m ost com m on causes o f D IC?

10.

W h a t is th e m ost com m on m utation th a t predisposes to venous throm bosis in w hite patients?

HEMATOLOGIC INFECTIONS

HEMATOLOGIC INFECTIONS

3 Question Warm-Up 1.

A post-op patient has poor urine output, a BUN o f 85, creatinine o f 3, and clear lungs. W h a t is the next step in the management o f this patient?

2.

W h at infection causes aplastic crisis in sickle cell patients?

3.

A patient presents w ith glomerulonephritis plus bilateral sensorineural deafness. W h a t is the diagnosis?

4. W hat is the treatment for infectious mononucleosis? ° 0 0 °

There is no antiviral medication available for mononucleosis (acyclovir is of no benefit) NSAIDs or acetaminophen for fever, sore throat, malaise Encourage rest and plenty of fluids Return to sport (risk of splenic rupture):

May return gradually to noncontact sports 3 weeks after symptom onset May return gradually to contact sports 4 weeks after symptom onset ° Steroids only helpful if impending airway compromise due to enlarged tonsils or if life-threatening sequelae develop (e.g., fulminant liver failure, hemolytic anemia, thrombocytopenia)

2661

End of Session Quiz 5.

A 63-year-old obese wom an w ith uncontrolled diabetes undergoes an emergency laparotomy to repair a perforated duodenal ulcer. A fter her surgical procedure, she is taken to the IC U intubated and on a ventilator, w ith both an arterial line and a right subclavian central line. Post-op H gb is 12 g/dL. The surgical IC U resident is tasked w ith her continued care. As o f now, the patient s blood pressure is 106/58 m m H g w ith a heart rate in the 140s. H er temperature is 96.5°F. W h a t treatm ent is necessary for this patient at this point?

6.

W h a t are the criteria for the diagnosis o f SIRS?

7.

A m an returns from a safari in Africa and now has periodic fevers, chills, diaphoresis, muscle aches and fatigue. H ow could this m an have avoided this illness?

HIV

HIV

3 Question Warm-Up 1.

W hat is the classic (but rare) EK G finding in pulmonary embolism?

2.

A post-op patient w ith significant pain presents with hyponatremia and normal volume status. W h a t is the diagnosis?

3.

W h a t is the treatment for mild unconjugated hyperbilirubinemia in a neonate? Severe unconjugated hyperbilirubinemia in a neonate?

End of Session Quiz 4.

A n HIV-positive patient is seen in the E R w ith shortness o f breath. H e reports that over the past few weeks it has become increasingly difficult to breathe when exerting himself. H e has a cough, fever, and you notice white plaques on the tongue and visualized pharynx. H e also admits to having pain with swallowing. You fear this patient’s disease has advanced, so you order a C D 4 count. W h a t do you expect the C D 4 count to be?

5.

A 39-year-old male H IV patient receives a head C T for headache and new-onset confusion and slurring o f words. The imaging reveals ring-enhancing lesions. W h a t infection is suspected in this person? W h a t lab test would support the diagnosis?

6.

W h a t is the rate o f transmission o f H IV through a needle stick incident? W hat drugs should be given in case there is appreciable risk o f transmission o f H IV in this setting?

HIV TREATMENT

3 Question Warm-Up 1.

W h a t is the tum or marker for each o f the following? ° Hepatocellular carcinoma ° Colon cancer 0 Gastric cancer ° Pancreatic cancer 0 Ovarian cancer

2.

A patient w ith sickle cell disease presents w ith pain in the right tibia, and investigation reveals osteomyelitis. W h a t is the most likely causative organism?

3.

W h a t does an odds ratio estimate in the case o f a disease w ith low prevalence?

I

T)

P

O* P

p

n>

3’ p; g o

Crq o

pr g o

p

W p* <

£3 W T I) Cft) rq 5rT } p tr o cd

5'4 n3> 1 n> prf C^rq o S-g

cd

P

pr o o

O n;

c

0"Q

p

S’

p

p

S. 12 Cd

& H

crq

St

a1

?3WW ►-• Hh ^p*3< 22.

p.* o o "2!. p 3 p^ 3 p 2. < 3

CD

P 9 *

CD N

2.

H N r w> P p ig 3 ^

1, t §• 5 s

I2.1Ip-sa . -s: — 5 >p i cr

rt

5’

P

P

HH

3

cd

2>oco L £»JJ p g 5 q » n P. CD . § "2

p o 3-

< i— t eO »3.-t 6cd o' ~ P co 3a O o*-h*2PP op o’ p

O E

Z tr

z

M-j

> go*r

pp 3 ’

o crq

p r n>

p p pa £



£>Crq

ct*

p~ p p

(✓> 1 C& D 3

X

rt

X

p

crq £, JO § 2 o

§3 s, o

* 8 v >'

r r JS * p

2 ^CD x

n

pr^ o o CO

^

P

zCD

5 ’ 5 * r+ 13

n

12 o °*■+

r* rt O

P 0q CD P

< 13

o P 3

3

TD O P rt U> P

9-

X co P r* O

r5* ©.

o

oCD n P 9

13 9*

crq CD

3CD 9

M

CO

>

13 o

^ S

2 9

cr p

4* p ** cr

X O o crq

p

3 2 TJ °» 2h >n ft ■2q 3 9-. CD £ H5T m o

p OP CD P •-»

o 13 co XO pr

3 co

P

13 13

O

P P P

QP

P CD

3 p

270



CD n p) n

End of Session Quiz 5.

A 25-year-old m an is diagnosed with H IV and m ust begin a H A A R T regimen. W h a t classes o f drugs should his regimen include initially?

6.

W h ich antiretroviral class matches each o f the following statements? ° SE: lactic acidosis ° SE: GI intolerance ° SE: rash ° SE: hyperglycemia, diabetes mellitus, and lipid abnormalities

7.

W h ich antiretroviral matches each o f the following statements? ° SE: bone marrow suppression with megaloblastic anemia » SE: potentially fatal hypersensitivity reaction ° SE: neurophychiatric symptoms (depression and vivid nightmares) ° SE: hyberbilirubinemia, jaundice ° SE: teratogenic, should not be given to women of childbearing age off of contraceptives ® SE: inhibits cytochrome P450, used to “boost” other drugs

MYELOMA AND LYMPHOMA

MYELOMAAND LYMPHOMA

3 Question Warm-Up 1.

W hat is the most common cause o f aortic stenosis in a 70-year-old patient?

2.

W hich of the following are elevated in DIC: fibrin split products, D-dimer, fibrinogen, platelets, and hematocrit?

3.

W hich type of renal tubular acidosis (RTA) is associated with abnormal H + secretion and nephrolithiasis?

Polycythemia Vera 4. W hat is the classic presentation of polycythemia vera? Most signs and symptoms are related to hyperviscosity causing vascular sludging. ° Average age of onset 50-60 years (but children and young adults can be affected as well) ° Visual disturbances —blurred vision, amaurosis fugax, scintillating scotoma, ophthalmic migraine ° ____________________(15%) - stroke, M I or angina, claudication, DVT or PE, BuddChiari syndrome, superficial thrombophlebitis ° Erythromelalgia —_______________________________ with erythema, pallor or cyanosis ° ____________________(especially after a warm bath) O O ° Elevated H & H and red cell mass, basophilia, leukocytosis (40%), thrombocytosis (60%)

5. W hat is the treatment for polycythemia vera? ° Phlebotomy to keep hematocrit below 45% in men, 42% in women Induces a desirable iron deficiency anemia —> do not supplement iron! ° A dd___________________ if at high risk for thrombosis (over age 70, prior thrombosis, platelets >1,500,000, or presence of cardiovascular risk factors) ° ___________________ every day to help prevent thrombosis (MI, CVA, PE, DVT) ° If refractory pruritus or refractory erythrocytosis —>___________________ ° If symptomatic hyperuricemia —» allopurinol 300mg every day

[ 272 ]

End of Session Quiz 6.

A 66-year-old woman fractures her hip after falling from standing. Radiographs reveal “punched out” lesions in the vertebrae, hips and femurs. The patient says th at she has had increasing back pain, weakness and fatigue, but she has attributed all o f that to aging. Labs show anemia, hypercalcemia, and increased B U N and creatinine. W h a t studies would help to make the diagnosis, and what would you expect to see?

7.

A 21-year-old male patient presents w ith recent weight loss, pruritus and night sweats. Physical exam reveals hepatosplenomegaly and a nontender cervical lymphadenopathy. W h a t do you immediately suspect?

8.

W h ich blood cell pathology matches each o f the following high-yield descriptions? Associated w ith Epstein-Barr virus (in Africa) Reed-Sternberg cells, cervical lymphadenopathy, night sweats Bence-Jones proteins, osteolytic lesions, high calcium Translocation 14;18 M ost common lymphoma in the US Translocation 8;14 M ost common form of Hodgkin lymphoma “Starry-sky pattern” due to phagocytosis o f apoptotic tumor cells H igh hematocrit/hemoglobin, pruritus (especially after hot bath or shower), burning pain in hands or feet

LEUKEMIA

LEUKEMIA

3 Question Warm-Up

i 274 |

1.

A n elderly man is seen in the E R w ith a chief complaint o f headache. Further questioning reveals that since the weather has turned cold and he had to begin using his kerosine heater, he has felt fatigued and a little nauseated along with the headache. O n physical exam, the man’s lips seem remarkably red. W h at do you expect the pulse ox to show? H ow will you treat this man?

2.

A 3-year-old girl presents w ith an abdom inal mass, hem aturia and hypertension. W h a t is the m ost likely diagnosis?

3.

A recent C uban im m igrant with symptoms o f malabsorption is found to also have megaloblastic anemia. W h a t is the disease and treatment?

End of Session Quiz 4.

W h a t type o f leukemia matches each o f the following descriptions? ° Most common neoplasm in children (peak age 3-4 years) ° Most common leukemia in adults (average age of onset SO years) ° Philadelphia chromosome is almost always seen ° Smudge cells on peripheral smear ° Peripheral blasts are PAS (+) and TdT (+) ° Peripheral blasts are PAS (-), myeloperoxidase (+) and have Auer rods ° Pancytopenia in a Down syndrome patient ° Associated with translocation 9;22 ° W hite cells with hair-like projections

5.

W h a t medication is associated w ith remission in 95% o f patients w ith CM L?

6.

The peripheral smear o f an asymptomatic patient reveals macrocytosis and hypogranular granulocytes w ith bilobed nuclei. W h a t is the diagnosis?

PEDI HEME/ONC

PEDI HEME/ONC

3 Question Warm-Up 1.

W hat is the definition of maternal mortality?

2.

W hich type of lung cancer is associated with each of the following paraneoplastic syndromes? ° ° ° °

3.

[ 276 ]

Elevated ACTH —» glucocorticoid excess —> Cushing syndrome Elevated PTH-related peptide —>hypercalcemia Elevated ADH —* SIADH —> hyponatremia Antibodies to presynaptic Ca2* channels —> Lambert-Eaton syndrome

W hat disease causes glomerulonephritis with deafness?

4.

A 5-year-old boy is brought in w ith a “swollen leg.” The child has not had any recent traum a according to the parents, and the physical exam confirms that. However, the swollen area is actually a mass that seems embedded within the vastus medialis o f the right thigh. W h a t is the next step in the management?

5.

W h a t is the m ost common adrenal tum or in children? W h a t lab studies can be used to diagnose this?

’'.'.5a

A- ,

f



K jg

/

t j .

;

",



’A 1’

V '.

6.

A 4-year-old girl is brought into the county clinic w ith an upper respiratory infection (U R I). The m other reports th at this child seems to get sick more often th an her friends’ kids o f the same age. Physical exam is remarkable for dangling thum bs, short stature, and hypopigmentation o f some skin areas. Labs reveal a pancytopenia. W h a t is the likely diagnosis?

7.

A 3-m onth-old child is brought to the office w ith difficulty breathing, fatigue and pallor. H e has a heart m urm ur and abnormal thumbs. Labs are as follows: Hgb

4 g/dL

H ct

12%

W BC

8,000 per pL

Platelets

300,000 per pL

M CV

99 fL

PEDI HEM E/ONC

End of Session Quiz

W h a t is the diagnosis? W h a t would you expect the level o f erythropoietin to be?

[ 277]

Musculoskeletal 1

O rthop ed ics p a rt 1

2

O rthop ed ics p a rt 2

3

M etabolic Bone Diseases

4

Infections, O A and Neoplasm s

5

R A and Lupus

6

O th e r R heum atologic Diseases

7

Pedi O rth o

ORTHOPEDICS PART I

3 Question Warm-Up 1.

A n elderly woman w ith a history o f cholelithiasis presents w ith a 5-day history o f vague intermittent abdominal pain and vomiting. W h a t diagnosis do you immediately suspect?

2.

A 60-year-old w om an leaks urine w hen laughing or coughing. W h a t are her nonsurgical options?

3.

W h a t is the late, life-threatening complication o f chronic myelogenous leukemia (CM L)?

Dislocations 4.

How does an anterior shoulder dislocation present differently than a posterior shoulder dislocation?

A rm position

Neurovascular compromise

Anterior Shoulder Dislocation

Posterior Shoulder Dislocation

External rotation and slight abduction

° Internal rotation and adduction ° Unable to externally rotate

artery and nerve

Unusual

at risk

Classic scenario

Blow to abducted, externally rotated, extended arm (blocking a basketball shot)

° Blow to anterior shoulder and electrocution

Physical exam

Prominent acromion (if thin patient) and loss of shoulder roundness

Posterior prominence and anterior shoulder is flat

ORTHOPEDICS PART

Fractures 5. W hat nerve is damaged when a patient presents with each of the following symptoms? ° ° ° ° °

Claw hand Ape hand Wrist drop Scapular winging Unable to wipe bottom

0 ° ° ° °

Loss of forearm pronation Cannot abduct or adduct fingers Loss of shoulder abduction Weak external rotation of arm Loss of elbow flexion and forearm supination

° 0 ° °

Loss of wrist extension Trouble initiating shoulder abduction Unable to abduct arm beyond 10 degrees Unable to raise arm above horizontal

6. W hat nerve is most at risk of injury with the following types of injury? ° ° ° ° ° °

Fracture of the shaft of the humerus Fracture of the surgical neck of the humerus Supracondylar humerus fracture Fracture of the medial epicondyle Anterior shoulder dislocation Injury to the carpal tunnel

End of Session Quiz 7.

8.

W h a t are the classic symptoms o f carpal tunnel syndrome?

1 o3 I 17 H X

2

TJ

C

O " U > 7 —31

c 1 — o CO

O m

CO

n

m r ~ m

W h a t is the difference between a M onteggia fracture and a Galeazzi fracture?

1

r~

9.

W h a t complications should you look for w ith the following types o f fracture/injury? Fall on outstretched arm —> snuffbox tenderness A nterior shoulder dislocation Fracture of the 5th metacarpal neck Humerus fracture H ip fracture Femur fracture Tibial fracture Pelvic fracture

I [ 283 ]

ORTHOPEDICS PART 2

3 Question Warm-Up 1.

W hat infections are classically associated with cold agglutinins?

2.

W hat is the most likely cause o f secondary hypertension given the following findings? ° Hypertension measured in the arms but low BP in the LE ° Proteinuria ° Hypokalemia ° Tachycardia, diarrhea, heat intolerance ° Hyperkalemia ° Episodic sweating, tachycardia

3.

W hat is the cause o f erythroblastosis fetalis?

W hat type of knee injury matches each of the following statements? ° ° ° °

Most commonly injured knee ligament Positive Lachman test Positive McMurray test aids in diagnosis Common dashboard knee injury in an MVA

W hat are the characteristic features of com partm ent syndrome? ° ° ° ° °

Earliest sign: pain in excess of what is expected and occurs even with passive motion 6 Ps:________________________________________________________ Measured compartment pressures > _____________ mmHg Most common compartments: volar compartment of forearm, anterior compartment of leg Most commonly due to fractures (supracondylar humerus, both-bone forearm, proximal tibia)

End of Session Quiz 6.

H ow is com partm ent syndrome diagnosed?

7.

W h a t is the treatm ent for com partment syndrome?

0 TO H 1 o~o m u n CO "0 > TO

W h a t is the cause o f low back pain given the following hints at presentation? ° Pain increases with passive straight leg raise ° Pain lessens with flexion at the hips (e.g., bending over shopping cart) ° Elderly, weight loss, pain constant but worse when supine ° Acute urinary retention ° Pain made worse by walking and standing (AKA pseudoclaudication) o Loss of foot dorsiflexion and pain on crossed straight leg raise ° Pain limited to the paraspinal region

[ 285 ]

METABOLIC BONE DISEASES

[286 ] I

METABOLIC BONE DISEASES

3 Question Warm-Up 1.

A 4-week-old child presents with non-bilious vomiting despite changing from milk-based formula to soy-based formula. W h a t is the most likely etiology?

2.

W hich type o f vasculitis fits the following descriptions? ° Infants and young children; involved coronary arteries ° Most common vasculitis ° Associated with hepatitis B infection ° Occlusion of ophthalmic artery can lead to blindness ° Perforation of nasal septum ° Unilateral headache, jaw claudication

3.

W h a t are the four main causes o f microcytic anemia?

f ill

End of Session Quiz 4.

W h a t medications are used in the treatment o f acute gout? Pseudogout?

5.

Com pare P T H , alkaline phosphatase, serum calcium and serum phosphate levels in patients w ith the following diseases:

Serum C a 2+

Serum Phos

A ik Phos

PTH

Paget Disease Osteomalacia/rickets Chronic renal failure Osteoporosis Osteopetrosis Primary hyperparathyroidism Hypoparathyroidism Pseudohypoparathyroidism

6.

W h ich disease matches each o f the following descriptions? ° Knee x-ray reveals calcification of the menisci ° Needle-shaped, negatively birefringent crystals ° Child with low-trauma fractures ° Narrowing of the marrow cavity results in low H & H ° 55-year-old woman that trips and sustains a distal radius fracture

7.

A 6-year-old girl is brought to the children’s E R for suspected broken bone in the forearm after the child fell while running around the backyard. The parents tell you th at this is the patient’s third fracture. The patient doesn’t seem to respond to questioning, to w hich the parents inform you that she is hard o f hearing. W ith this clue, you check the patient’s eyes to help confirm your diagnosis. W h a t are you looking for? I f this diagnosis is correct, how can it be treated?

8.

A 60-year-old m an is in the clinic for a checkup. H e is a new patient and you notice th at his legs are bowed out. H e also is bent forward w ith kyphosis and is hard o f hearing. H e has no complaints besides his favorite hat not seeming to fit anymore. H e claims that it feels smaller. W h a t imaging is most sensitive to diagnose this process?

INFECTIO NS, OA AND NEOPLASMS

INFECTIONS, OA AND NEOPLASMS

3 Question Warm-Up 1.

W h at are the signs suggesting radial nerve damage with a humeral fracture?

2.

A patient presents to the clinic for follow-up and is found to have a blood pressure o f 150/85. You note in the chart that during his last visit one month ago, his blood pressure was 145/90. W h at is the next step in the management o f this patient?

3. W h a t is the most common testicular cancer?

4. W hat is the classic presentation and clinical course of Lyme disease? ° Early localized disease —80% of patients, usually within 1 month Erythema chronicum migrans - bull’s eye rash with central clearing that expands over days to weeks +/- constitutional symptoms (fatigue, headache, myalgias, arthralgias,...) 0 Early disseminated disease - weeks to months after the tick bite, may include any of the following: Meningitis (lymphocytic) Unilateral or bilateral cranial nerve palsies (esp. of the facial nerve —> "bilateral Bell’s palsy”) Radiculopathy Peripheral neuropathy - Carditis (AV heart block, myopericarditis) ° Late Lyme disease - months to years after infection onset Arthritis (esp. knee) Subacute encephalitis

5. W hat is the treatment for Lyme disease? W hat is the treatment for Rocky Mountain Spotted Fever? ° Early Lyme disease treatment options (14-21 days): ___________________________________ lOOmg PO bid (preferred agent, can be dosed over only 14 days, avoid in pregnancy) Amoxicillin 500mg PO tid Cefuroxime 500mg PO bid ° Late Lyme disease (carditis, encephalitis, arthritis) —>usually Ceftriaxone 2g IV q24 hours x 14-28 days 0 Rocky Mountain Spotted Fever treatment options: Doxycycline lOOmg PO bid x7 days Chloramphenicol 50mg/kg daily divided in four doses in pregnant patients

End of Session Quiz 6.

In cases o f an unhelpful x-ray and unavailable M R I, w hat 3 studies can be used to make the diagnosis o f osteomyelitis?

m n H O

z o> >

7.

W h a t is the treatm ent for Lyme disease? For Rocky M ountain Spotted Fever?

8.

W h a t is the empiric treatm ent for septic arthritis?

9.

W h a t are the m ost common causes o f bony metastasis?

10.

W h a t is the classic radiological appearance o f osteosarcoma? O f Ewing sarcoma?

11.

A patient presents w ith bone tenderness, and is found to have elevated W B C count, CRP, and ESR. W h a t is the most likely diagnosis?

12.

W h a t is the m ost common organism in osteomyelitis overall? W h a t organism should also be kept in m ind for sickle cell patients? W h a t organism should also be kept in m ind for IV drug users?

z a z m o

[ 289 ]

RAAND LUPUS

3 Question Warm-Up 1.

W hich glomerular disease would you suspect most in a patient with the following findings? ° Most common nephrotic syndrome in children ° Most common nephrotic syndrome in adults ° Kimmelstiel-Wilson lesions (nodular glomerulosclerosis) ° LM: crescent formation in the glomeruli o LM: segmental sclerosis and hyalinosis ° Anti-GBM antibodies, hematuria, hemoptysis • Nephrotic syndrome associated with hepatitis B ° Nephrotic syndrome associated with HIV

2.

Positive p-A N C A is associated with what conditions?

3.

A husband asks that his wife (your patient) not be told about her recentlydiscovered lung cancer. W h a t should you do?

W hat are the diagnostic criteria for rheumatoid arthritis? (Arthritis Rheum 2010; 62:2569) Synovitis in at least 1 joint, not better explained by another disease, plus a total score of > 6/10 (add score of categories A-D below): A. Joint involement ° 2-10 large joints = I point ° 1-3 small joints (with or without large joints) = 2 points ° 4-10 small joints (with or without large joints) = 3 points ° >10 joints (at least I small joint) = 5 points B. Serology » Low positive RF or ACPA = 2 points ° High positive RF or ACPA = 3 points C. Abnormal CRP or ESR = 1 point D. Duration of symptoms > 6 weeks = 1 point

W hat medications are considered first-line treatments for rheumatoid arthritis? Disease-modifying anti-rheumatic drugs (DMARDs) O ° Sulfasalazine O ° _______________ inhibitors - Etanercept Adalimumab Golimumab Certolizumab Infliximab ° Leflunomide ° Anakinra Steroids or N SAID s (ibuprofen 800mg qid, naproxen 500mg bid, celecoxib lOOmg bid) are only used as adjuncts

End of Session Quiz 6.

W h a t disease should you include in your differential with arthropathy o f the following joints? • D IP and PIP ° PIP and M CP (but not DIP)

7.

W h a t factors tend to elicit joint pain and stiffness in RA? W h a t factors tend to relieve pain/stiffness? H ow does this differ from the pattern seen in OA?

8.

W h a t disease is associated w ith each o f the following serologic markers?

Marker

Disease

Anti-histone antibodies Rheumatoid factor A nti-dsD N A antibodies Anti-Sm antibodies HLA -D R4

9.

W hat are the 4 skin findings that are diagnostic criteria for SLE?

OTHER RHEUMATOLOGIC DISEASES

3 Question Warm-Up 1.

W h at lab changes would you see in the following diseases?

Disorder

Platelet Count

Bleeding Time

PT

PTT

HUS or T T P Hemophilia A or B von Willebrand disease DIC Warfarin use End stage liver disease Aspirin use

2.

W h a t type o f acute renal failure would you suspect in a patient w ith FE N a improvement in 25-45% of patients ° Tylenol 650mg + Ultram 75mg QID —> 50% pain reduction ° Pregabalin (Lyrica) (FDA-approved for fibromyalgia June 2007) ° Duloxetine (Cymbalta) (FDA-approved for fibromyalgia June 2008) ° Fluoxetine 20-80mg qAM (+/- Elavil qHS) ° Milnacipran (FDA-approved for fibromyalgia Jan 2009)

6.

O

73

I m C 3 $

O r~ o O n o on m >

How is the diagnosis of C R E S T scleroderm a (A K A limited cutaneous systemic sclerosis) made? The diagnosis is primarily clinical, but lab studies can support the clinical diagnosis. ° Calcinosis cutis - subQcalcifications often in the fingers, not always present ° Raynaud phenomenon - cyanotic vasoconstriction especially in the fingers ° Esophageal dysmotility —due to lower esophageal sphincter sclerosis —>reflux, dysphagia ° Sclerodactyly —skin fibrosis especially at the fingers, hands and face ° Telangiectasias —on the Up, hand or face; not always present ° Labs: anti-Scl-70, anti-RNA, anti-Ul RNP, anti-centromere (Lab work can support the diagnosis, but cannot rule out scleroderma.)

I [ 293 ]

OTHER RHEUMATOLOGIC DISEASES

End of Session Quiz 7.

A 65-year-old African American woman is evaluated for weakness. H ie patient has been an active person but now has weakness in her lower extremities. O n exam, both are equally weak. She also has a rash on the chest. W hat labs would be helpful to order initially?

8.

A 67-year-old woman complains of pain in her extremities, especially in the shoulders and hips. It is hard for her to even get out of bed. ESR is markedly elevated. W hat is the treatment?

9.

W hat serious disease must you look out for in patients with polymyalgia rheumatica?

10. W hich rheumatologic disease matches each o f the following descriptions? ° Proximal muscle weakness and facial rash ° Pain and stiffness in the hips and shoulders o Muscle pain and tenderness in multiple distinct locations ° Man in his 20s with low back pain that improves with exercise 0 Jaw claudication and pain with standing from a chair ° Pencil-in-cup deformities of the DIP and PIP joints ° Bamboo spine on x-ray ° Arthritis + oral ulcers + proteinuria 11. W h a t are the seronegative H LA -B27 spondyloarthropathies?

O 73

HI X

°

3 Question Warm-Up 1.

W h a t are the four potassium-sparing diuretics?

2.

Low urine specific gravity in the presence o f high serum osmolality. W h a t is the diagnosis?

3.

W h a t is the diagnostic test for hereditary spherocytosis?

4. W hich infants should be screened for developmental dysplasia of the hip? Obtain hip sonogram at 6 weeks if:

5. W hat is the treatm ent for slipped capital femoral epiphysis? ° Bedrest ° Avoid weight bearing, crutches and/or wheelchair until surgically repaired ° Prompt surgical pinning of the head of the femur If acute/unstable —> admit to hospital for surgical treatment If chronic/stable —» urgent outpatient evaluation ° Closed reduction of acute slips prior to pinning is controversial

6. W hat is the recommendation for the amount of vitamin D supplementation for infants? All children (including breastfed infants) should receive vitamin D supplementation of 400 IU daily starting th e __________________________________

[295]

MUSCULOSKELETAL

PEDI ORTHO

PEDI ORTHO

7.

W hat is the treatment for juvenile idiopathic arthritis? ° NSAIDs are the drugs of choice ° If unresponsive to a trial of two different NSAIDs over at least six weeks, then second-line is____________________ o r____________________

8. W hat are the characteristic features for Osgood-Schlatter disease? ° Most common symptom is_____________________________ that increases over time and is worsened by quadriceps contraction (running, jumping) ° Signs at th e_____________________________ may include soft tissue swelling, a palpable bony mass and/or pain upon quadriceps flexion

9. W hat is the treatment for Osgood-Schlatter disease? ° Can continue sports despite pain ° Rehabilitation: Stretching the hamstrings and quadriceps Strengthening the quadriceps ° Osgood-Schlatter pad - protective pad over the tibial tuberosity ° Ice to the affected area after activities ° NSAIDs for pain ° Knee immobilizers are contraindicated

10. W hat is the treatm ent for a clavicle fracture in a newborn? ° No treatment needed (not necessary to immobilize by pinning the shirt) ° Evaluate for brachial plexus injury

11. W hat is the treatm ent for a mid-third clavicle fracture? ° Figure-of-eight strap and/or arm sling until fracture site and range of motion are painless (usually 4-8 weeks) ° Follow-up in 1-2 weeks, then every 2-3 weeks until asymptomatic (adults: usually 6-12 weeks, children: 3-6 weeks) ° Repeat x-ray at 6 weeks and upon clinical healing ° Orthopedic consult if nonunion after 12 weeks ° Rehabilitation: Elbow range of motion (ROM) exercises starting day I Shoulder ROM exercises after immobilization

12* W hat is the treatment for nursemaid’s elbow? D O 73 H X

o

13. W hat is the treatm ent for Legg-Calve-Perthes disease? 0 Non-weight bearing on the affected side for an extended period of time 0 If limited femoral head involvement and full RO M : Observation

0 If extensive femoral head involvement or limited RO M : Bracing Hip abduction with a Petrie cast Osteotomy

14. W hat is the classic presentation of childhood spondylolisthesis? 0 Anterior slip of a vertebra resulting in a palpable step-off on physical exam (usually L5 over S I)

0 Subacute back pain exacerbated by hyperextension of the spine 0 Knee-flexed, hip-flexed gait in cases where the sacrum becomes relatively more vertical and hip extension is impaired

0 Possible neurological dysfunction including urinary incontinence (very rare)

[297]

MUSCULOSKELETAL

° Reduce by gently flexing and. the arm with one hand while supporting the elbow and applying gentle pressure to the radial head with the other hand Give the child a popsicle that they can eat only by using the recently reduced arm to encourage movement and confirm successful treatment No need to immobilize

PEDI ORTHO

End of Session Quiz 15.

16.

W hat is the treatment for developmental dysplasia o f the hip in children younger than 6 months o f age?

W hat is the treatment for slipped capital femoral epiphysis?

17. W hat is the treatment for Osgood-Schlatter disease?

18. W h at disease is responsible for a painful limp in a child in each of the following scenarios? Scenario/Finding X-ray reveals femoral head sclerosis X-ray reveals ice cream scoop (femoral head) falling off of cone (femur) Obese, male adolescent with dull hip pain and inability to bear weight Acute onset of tibial pain, fever, malaise, elevated ESR, no joint pain Acute onset of knee pain, fever, elevated ESR, leukocytosis 7-year-old with growth delay and inner thigh pain 13-year-old boy with pain and swelling at the tibial tuberosity

19. W h a t is the treatment o f nursemaid s elbow?

D isease

D ermatology 1

Infections p a rt 1

2

Infections p a rt 2

3

Infections p a rt 3

4

Inflam m atory Skin C onditions

5

Bullous D iseases and N eoplasm s

6

Plastics, P igm entation and H a ir Loss

INFECTIONS PART I

3 Question Warm-Up 1.

W h ich vaccines should not be given to an HIV-positive patient?

2.

W h a t is the treatm ent for T T P ?

3.

C om pare the serum iron, ferritin and transferrin levels in iron deficiency (IDA) to anemia o f chronic disease (AOCD).

Serum iron

Ferritin

Transferrin

ID A AOCD

4.

Com plete the table:

Definition

Examples

Flat spot less than 1 cm (non-palpable, just visible)

Freckles, tattoos

Flat spot > 1 cm

Port-wine stain

Solid, elevated lesion < 1 cm (palpable)

W art, acne, lichen planus

Same as papule but > 1 cm and flat-topped

Psoriasis

Palpable, solid lesion > 1 cm and not flat-topped

Small lipoma, erythema nodosum

Elevated, circumscribed lesion < 5 mm containing clear fluid (small blister)

Chickenpox, genital herpes

Same as vesicle but > 5 mm (large blister)

Contact dermatitis, pemphigus

Itchy, transiently edematous area

Allergic reaction

Term

n H O z (/> !>

INFECTIONS PART I

5.

Describe the proper treatment for skin abscesses. ° Incision and drainage (I&D) if overt clinical abscess or proven by sono or CT If at risk for endocarditis, then antimicrobial prophylaxis prior to l&D (e.g., vancomycin or TMP-SMX) ° Culture in aerobic and anaerobic tubes. Ideally anaerobic sample is obtained via needle aspiration to avoid air exposure ° Surgery consult for I&D in OR when abscess is particularly large or in a sensitive area ° No antibiotic therapy needed if < 5cm and low-risk patient ° Antibiotics for 10-14 days for presumed community-acquired MRSA Bactrim DS + rifampin Clindamycin (may have inducible resistance) + rifampin Minocycline (or doxycycline) + rifampin Linezolid ($$$)

6. W hat are some of the distinguishing characteristics of necrotizing fasciitis? ° 0 ° °

Unexplained, excruciating pain in the absence of or beyond areas of cellulitis Erythema with blister and bullae formation and possible crepitus Diabetes patient with foot cellulitis and signs of systemic toxicity Perineal cellulitis with abrupt onset and rapid spread (Fournier gangrene)

7. W hat is the general treatment for necrotizing fasciitis? ° Immediate, extensive surgical debridement ° Antibiotics: General empiric polymicrobial coverage - imipenem (or meropenem) +/- vancomycin If Streptococci —penicillin G +/- clindamycin If Clostridia - penicillin G + clindamycin ° Treatment for shock if it arises (IV fluids, dopamine)

8. W hat are the distinguishing characteristics of gangrene? ° Wet gangrene - bruised, swollen, blistered with pus ° Dry gangrene Early signs - ache, cold, pallor Late signs —bluish-black, dry, hard, shriveled tissue ° Gas gangrene Early signs - pain and swelling around an injury site (often surgical incision) Classic signs —initially pale, then dark purplish-red, tense, tender, with soft-tissue crepitus Signs of systemic toxicity - tachycardia, low-grade fever; diaphoresis +/- shock and multisystem organ failure

9.

W hat is the treatm ent for a limb with dry gangrene? ° Autoamputation over time ° Angiography to evaluate the extent and location of peripheral artery disease —> distal bypass of stenotic areas —>if circulation improves and healing is adequate, then amputation of the affected region

10. W hat is the treatm ent for a wet gangrene infection?

[ 302 ] |

° Emergency debridement or guillotine amputation of the infected portion of the foot, then revision to a below or above the knee amputation 72 hours later ° Antibiotics are indicated if cellulitis or gas gangrene is present

End of Session Quiz 11. A patient is admitted to the IC U for fluid resuscitation and monitoring following trauma with significant blood loss. A central line that was inserted into the right groin emergently in the E R has been in place for a few days, and now the surrounding skin is red and warm. Also, the patient’s temperature is rising, and the W B C count is elevated. W h at is the likely cause?

12. A 44-year-old African-American woman is in the office for evaluation o f an area o f inflammation in her left axilla. She said that initially the area was simply itchy, but has now become painful. O n exam, the area has about six papules and nodules that are erythematous, indurated and warm. The skin is fluctuant, and drainage is noted at some o f the lesions. H ow will this patient need to be treated?

13. W h a t are the characteristic features o f necrotizing fasciitis?

14. W h a t is the treatm ent for dry gangrene? W h a t is the treatment for wet gangrene?

15. A 66-year-old m an w ith longstanding, poorly-controlled diabetes arrives at the E R complaining o f a horrid smell coming from his left foot. H e denies pain, b u t admits to having lost the feeling in his feet a long tim e ago from the diabetes. O n exam, there is an open wound between the 1st and 2nd toe on the left foot. Pus drains from the w ound and some crepitus is felt in the area. The odor is atrocious. The patient is tachycardic and feverish. W h a t treatm ent does he need?

16. A 7-year-old boy is brought to the county clinic w ith a rash. The m other denies th at the child has acted ill. The exam is unremarkable besides perioral honeycrusted lesions and regional lymphadenopathy. W h a t can be used to treat this patient?

INFECTIONS PART 2

3 Question Warm-Up 1.

W hich glomerular disease would you suspect most in a patient w ith the following findings? ° IF: granular pattern of immune complex deposition; LM: hypercellular glomeruli ° IF: linear pattern of immune complex deposition ° EM: loss of epithelial foot processes ° EM: subendothelial humps and tram-track appearance o Nephritis, deafness, cataracts ° Purpura on the back of the arms and legs, abdominal pain, IgA nephropathy ° Anti-dsDNA antibodies

2.

A patient presents to the E R with a very painful, irreducible inguinal mass. W h a t is the next step in the management o f this patient?

3.

W h at cause o f aplastic anemia is associated with thum b abnormalities, diffuse hypo- or hyperpigmentation, cafe-au-lait spots and short stature?

Acne Vulgaris 4.

W hich acne medication is known for causing photosensitivity?

5. W hat should you know about oral isotretinoin (Accutane) in the treatment of acne? ° ° ° °

Usually try 2-3 other therapies prior to using this therapy Check |3-hCG, CBC, lipids and LFTs regularly 25% develop increase in triglycerides (> 800 —> risk of pancreatitis) For dry skin - moisturizing soap, lotions, Chapstick, Polysporin to nares PRN, eye drops PRN ° Screen for depression and suicidal ideation each visit ° Never use with tetracycline —>combined risk of pseudotumor cerebri ° OCPs should be prescribed to women patients due to high risk of teratogenic side effects

Rosacea 6- W hat is the classic presentation of rosacea? ° Middle-aged patient ° Facial erythema with telangiectasias starting at the nose and cheeks ° Recurrent facial flushing provoked by various stimuli including hot/spicy foods, alcohol, temperature extremes, emotional reactions Inflammatory papules, pustules, cysts and/or nodules similar in appearance to acne but without comedones ° Ocular blepharitis, conjunctivitis and/or keratitis ° Rhinophyma (sebaceous gland hyperplasia of the nose)

7.

W hat are the treatm ent options for rosacea? Topical Treatment: ° Sulfacetamide 10% + sulfur 5% lotion/gel ° Metronidazole gel or cream ° Rhinophyma may require laser therapy Systemic Treatment: ° Tetracycline ° Doxycycline or minocycline ° Isotretinoin (Accutane) for severe refractory cases

Varicella 8. W h at are the clinical features of varicella chicken pox? ° Prodrome of malaise, fever, pharyngitis, headache and myalgia for 24 hours prior to rash onset ° Pruritic evolving rash: red macules —>teardrop vesicles —> rupture and crusting over

-

Vesicular rash starts on the face and trunk then spreads to extremities Rash appears in successive crops of vesicles over 2-4 days Most all lesions are fully crusted by 6 days

° Skin bacterial superinfections may occur (S. pyogenes) ° Adults may also develop pneumonia and/or encephalitis

9.

W h at treatm ents are available for children with chicken pox (varicella)? ° Antihistamines for pruritus ° Cut fingernails closely to avoid excoriations leading to bacterial superinfections ° Acetaminophen for fever ° No need for acyclovir in otherwise healthy children younger than 12 (AAP recommendation) because although it has been shown to decrease duration by 1 day and decrease the number of lesions, it does not reduce complications ° Acyclovir for the following groups: older than 12 years of age, household contacts, history of chronic cutaneous or cardiopulmonary disorders, those taking intermittent oral or inhaled steroids, those taking chronic salicylates

[305]

INFECTIONS PART 2

10. W hat is the treatment for an uncomplicated varicella zoster outbreak in an elderly patient? ° Antiviral therapy if uncomplicated zoster presenting within 72 hours of clinical symptoms Valacyclovir Famciclovir Acyclovir - high dosing frequency but low cost ° Analgesia with opioids ° Corticosteroids (prednisone tapered over 7 days) only if severe symptoms and no contraindications. Usually the high risk of side effects outweighs the only modest benefits.

II. W hat medications are used in the treatment of postherpetic neuralgia?

[ 306 ]

End of Session Quiz 12. A 15-year-old girl is brought to the dermatologist for treatment o f her acne. W hat is the causative organism in acne?

13. W h a t medication options are available for the treatment o f acne vulgaris?

15. A 41-year-old wom an w ith rosacea is seen in the clinic. It is a mild case, for which avoidance o f the triggers o f facial flushing would be an initial therapy. W h at are some examples o f things to avoid?

16. W h a t is the treatm ent for rosacea?

17. W h a t is the tim e frame in the treatm ent o f varicella?

18. A 25-year-old m an is in the office for cold sores. O n exam, he has a collection o f 3 small vesicles at the vermillion border. H e says that they are painful. This is the second tim e he has had these. H e says he knew he was about to get them because he could feel a tingly sensation at that location a few days before the lesions appeared. W h a t studies m ight be helpful to diagnose this disease?

19. A n elderly w om an presents w ith a rash on her right flank. She says that the rash appeared a few days ago and followed a recent cataract surgery. The rash is causing her terrible pain. O n exam, the rash extends from the spine past the m id-axillary line in a dermatomal distribution. The rash is composed of grouped erythematous vesicles. W h a t can be used to treat this woman?

DERMATOLOGY

14. W h a t side effects can arise from the use o f oral isotretinoin?

NFECTIONS PART 3

INFECTIONS PART 3

3 Question Warm-Up 1.

In what condition would you see Auer rods?

2.

In what circumstances should confidentiality not be protected?

3.

W hat x-ray finding is indicative o f croup? W h a t x-ray finding is indicative o f epiglottitis?

W arts 4. W hich H PV types cause skin warts? W hich cause genital warts?

5. W hat are the different treatment options for condyloma acuminata (genital warts)? ° Spontaneous regression of small asymptomatic warts within 3 months occurs about 25% of the time 0 Podophyllotoxin (self-administered) (for vulvar lesions in non-pregnant women) 0 Podophyllin (for vulvar lesions in non-pregnant women) ° Trichloroacetic acid (TCA) —often the first-line treatment 0 Imiquimod (self-administered) (for vulvar lesions in non-pregnant women) - induces interferon-mediated antiviral response ° Cryoablation with liquid nitrogen

Funga! Infections 6. W hat are the treatment options for tinea versicolor?

t 308 j

° Topical OTC antifungal for 2 weeks (terbinafine (Lamisil), clotrimazole (Lotrimin)) ° Selenium sulfide (foam, solution, shampoo) qd-bid daily to affected areas for 1 week —> then ql-3 weeks for prophylaxis ° Ketoconazole 2% shampoo daily for 3 days ° Oral antifungal for extensive disease: ketoconazole, fluconazole, itraconazole

7.

W h at is the treatm ent for onychomycosis? ° First confirm diagnosis by sending a nail clipping for pathologic diagnosis. Onychomycosis is only responsible for 50-60% of abnormal appearing nails ° Terbinafine (Lamisil) and itraconazole (Sporanox) have cure rates of only 60-70% ° Strongly consider pretreatment LFTs and mid-treatment LFTs ° Terbinafine - Fingernails (6 weeks) - Toenails (12 weeks) (Off-label dosing: 250mg qd x7d every 2-3m x lyr may have better efficacy) ° Itraconazole Fingernails 200mg PO qd x8 weeks, or 400mg qd for I week each month for 2 months Toenails 200mg PO qd xl 2 weeks, or 400mg qd for I week each month for 3 months ° Fluconazole (Diflucan) 150mg once weekly x24 weeks (consider for those with complicated med regimens) (efficacy not as good as Lamisil or Sporanox; cure only about 32%) ° Reassure patient that oral agents will continue to work after stopping use. It may take a few months to see complete resolution ° Ciclopirox Nail Lacquer (Penlac) x48 weeks has complete cure rate of only 7% which means 1 in 15 patients will have a favorable outcome

Lice and Crabs 8. W h at is the treatm ent for pediculosis capitis and pediculosis pubis? ° Pediculosis capitis (lice) —wash scalp normally then towel dry —>saturate scalp with permethrin cream (OTC Nix 1%) or pyrethrin (OTC Rid) for 10 minutes then rinse —> repeat in 1 week due to resistance (CDC rec.) ° Malathion lotion 0.5% (Rx) may be used instead of permethrin Lindane is not used due to potential neurotoxicity and widespread resistance Ivermectin can be used in resistant cases (not FDA approved). 200 mcg/kg PO x l, repeated in 2 weeks If younger than 2 years, then wet combing with conditioner or olive oil rather than insecticides performed q3-4 days for weeks - Children may return to school after the first treatment session (wet combing or insecticide) ° Pediculosis pubis (“crabs”) —permethrin 1% cream (OTC Nix 1%) or pyrethrin (OTC Rid) for 10 minutes then rinse —» repeat in 1 week - Malathion or ivermectin can be used as alternatives (see above) - Sexual partners need to be treated at the same time - Bedding and clothing should be machine washed and dried in a hot dryer, dry cleaned, or bagged for a min of 72 hours

INFECTIONS PART 3

End of Session Quiz 9.

W hat is the appearance o f molluscum contagiosum?

10. A n obese, 42-year-old diabetic woman complains o f a pruritic rash underneath her breasts. Exam reveals an erythematous, patchy rash underneath large, pendulous breasts. W h at diagnostic study would be helpful, and what would be seen?

11. W h a t medication is preferred in the treatment of scabies?

12. A 22-year-old Asian patient comes to your office. She is concerned because she notices small areas of hypopigmentation on her back. She says these areas are more noticeable in the summer. W h a t is the organism that is likely causing her skin condition?

[310] I

3 Question Warm-Up 1.

W h a t is the treatm ent for W hipple disease?

2.

W h a t complications can arise from electrical burns?

3.

W h a t are the differing presentations o f Alzheim er disease, Pick disease and Lew y body dementia?

INFLAMMATORY SKIN CO N D ITIO N S

INFLAMMATORY SKIN CONDITIONS

Erythem a Multiforme 4. W hat are the characteristic features of erythem a multiforme (EM)? ° Skin lesion with target appearance (dull red center, a pale zone and a darker outer ring) ° Lesions can take many different shapes (multiforme) ° Lesions develop over 10+ days: macule —>papule —>vesicles/bullae in the center of the papule ° Common sites: hands/forearms, soles/feet, face, elbows and knees, penis and vulva ° Severe form (EM major) always involves the mucus membranes —>can become SJS/TEN

5. W h at is the treatm ent for erythem a multiforme? ° ° ° °

Stop any inciting medication Symptomatic treatment with antipruritics If severe —> systemic glucocorticoids (although no proven effectiveness) I f patient also has history of HSY —> antiviral such as acyclovir or valacyclovir

Stevens-Johnson Syndrome and Toxic Epidermal Necrolysis 6. W h at is the distinction between Stevens-Johnson syndrome (SJS) and toxic epidermal necrolysis (TE N )? SJS is the less severe form of T E N . In SJS, skin sloughing (epidermal detachment) is limited to less than 10% body surface area. In T E N , at least 30% of the skin is detaching. There is overlap between the two at 10-30% skin involvement.

[ 311 ]

INFLAMMATORY SKIN CO N D ITIO N S

Seborrheic Dermatitis 7. W hat is the treatment for infantile seborrheic dermatitis (A K A cradle cap)? ° Selenium sulfide (Selsun Blue) shampoo twice a week until resolved ° Massaging olive oil into the scalp and leaving for 15 minutes can help remove scale when washing ° +/- Hydrocortisone 1% cream bid to affected area

8. How does seborrheic dermatitis manifest in adults? Erythema, scaling, and white flaking in areas of sebaceous glands including eyebrows, nasolabial folds, face, external ear, scalp, upper trunk, and body folds (axilla, groin)

9. W hat diseases are associated with an increased incidence of seborrheic dermatitis? ° Parkinson, HIV, psoriasis, immunocompromised patients (e.g., transplant patients) ° Exacerbations are common in emotional stress and hospitalizations 0 Severe intractable seborrheic dermatitis may point to H IV infection

Atopic Dermatitis 10. W hat are the available treatments for atopic dermatitis (A K A eczema)? ° Switching to a moisturizing soap (Dove, Aveeno) and adding an O TC emollient may be all that is needed for maintenance and mild cases ° Hydration/Emollients: Cetaphil, Eucerin, Lubriderm, Aveeno, Aquaphor (or generic equivalents) High-water/low-oil lotions will worsen xerosis and eczema, and high-oil creams and ointments will reduce xerosis ° Calcineurin inhibitors: tacrolimus (Protopic) or pimecrolimus (Elidel) ° Topical steroids ° Antibiotics for open lesions (cover Staph, aureus and Strep, spp.) ° Antihistamines ° Leukotriene inhibitors (Singulair) —theoretical efficacy supported by weak studies ° UV light therapy 0 Systemic steroids (l-2mg/kg/day in children, then taper) only in severe cases and only for short duration ° For very severe cases, consider methotrexate, cyclosporin, azathioprine (Imuran)

11. W hat are the possible side effects of the calcineurin inhibitors (Elidel cream or Protopic ointment) in the treatment of atopic dermatitis? ° Do not cause systemic side effects or skin atrophy like topical steroids. Safe on face/eyelids ° Try to avoid in children younger than 2 years due to higher rates of URIs. However, most dermatologists have no problems using these in those under 2 years, because the alternative of using topical steroids would probably have more side effects ° Preliminary studies suggest possible slight increase in risk of lymphoma. Therefore, keep duration as short as possible

12. W h at drugs are used in the treatm ent of psoriasis? ° ° ° ° ° ° ° ° ° ° °

Topical steroids (NEVER give oral steroids to someone with psoriasis) Calcipotriene (vitamin D3 analog that inhibits epidermal cell proliferation) Tazarotene (Tazorac) (topical retinoid —>normalizes keratinocyte proliferation) Coal tar (suppresses DNA synthesis) Anthralin SaUcyhc acid (keratolytic used to remove excess scale) UV therapy - for patients with more than 10% (refer to a Dermatologist) Soriatane (acitretin) Kenalog (triamcinolone) injections into dermis Enbrel (etanerept): Anti-TNF agent approved for use in mod-severe psoriasis Other agents: oral retinoids, methotrexate, cyclosporine

INFLAMMATORY SKIN C O N D IT IO N S

Psoriasis

Pityriasis Rosea 13. W hat is the treatm ent for pityriasis rosea? ° This is a self-limiting disease of 4-6 weeks, and no treatment is necessary. Sunlight is helpful. ° If significant itching, may use a moderate-potency topical steroid ° If extensive disease or severe itching, phototherapy ° Possibly beneficial, but unproven: Erythromycin 250mg four times daily x 14 days Acyclovir 800mg five times daily x 7 days

Lichen Planus 14. W h at are the characteristic features of lichen planus? ° Skin involvement - pruritic, purple, polygonal papules and plaques that are shiny and flat, and commonly occur on the flexor surface of the extremities (e.g., wrist) ° Wickham striae is a white, lace-like pattern on the surface of the papules/plaques ° Mucus membrane involvement - Wickham’s striae in the lateral buccal mucosa and possibly erosive lesions that may become infected with Candida ° Genital involvement —usually limited to violaceous papules on the glans penis in men and vulva of women

15. W h at infections are associated with an increased likelihood of lichen planus?

16. W h at is the treatm ent for lichen planus? ° Corticosteroids of medium to high potency - topical or intralesional (oral if topical unsuccessful) ° Acitretin (an oral retinoid)

I [313]

INFLAMMATORY SKIN CO N D ITIO N S

Decubitus Ulcers 17. W hat are the different stages of decubitus ulcers? Stage I

Pressure-related alteration in intact skin such as change in color, consistency, sensation or temperature

Stage II

Superficial ulcer, abrasion or shallow crater

Stage III

Full thickness skin loss with damage to the subQ_tissues; deep crater

Stage IV

Extensive destruction or necrosis; damage to muscle, bone or supporting structures

18. W hat are the treatments for the different stages of sacral decubitus ulcers? ° Address nutrition Insure adequate protein & calorie intake Possibly helpful: daily MVI, zinc sulfate, vitamin C, arginaid (with L-arginine) ° Relieve the pressure: Turn q2 hours (30° angle on side) or advise patient to have frequent small changes in position - Appropriate, soft mattress Elevate heels above bed surface with pillow placed lengthwise and curled at the end, or use heel protectors (which usually don’t work) ° Stage I —as above +/- application of a protective dressing such as Xenaderm ° Stage II —routine wound care and hydrocolloid dressing (avoid wet-to-dry dressings) ° Stage III or IV —wound care with debridement and hydrocolloid dressing

Stasis Dermatitis 19. W hat is the characteristic appearance of stasis dermatitis? Eczematous dermatitis with inflammatory papules, scaly and crusted erosions, increased pigmentation, stippling with recent and old hemorrhages, and possible ulceration

20. W hat is the treatment for stasis dermatitis? ° Compressive dressings or stockings with at least 20-30mmHg of pressure (usually 3040mmHg) ° Elevation of the legs above the heart whenever possible, but for at least 30 min 3-4 times a day ° Topical steroids ° Consider horse chestnut seed extract ° Aspirin 300-325mg/day (may accelerate the healing of venous ulcers)

Z

End of Session Quiz 21.

A patient presents w ith erythem a multiforme. W hich medications are the most common offenders?

“TI

r~

> Z

o7 3

-< C/l A

22.

W h a t are the distinctions between erythema multiforme, Stevens-Johnson syndrome, and toxic epidermal necrolysis (TEN )? icrolvsis (TI

23.

W h a t is the classic presentation o f pityriasis rosea? W h a t is the treatment?

24.

W h a t is the classic presentation o f erythem a nodosum?

25.

W h a t are the treatm ent options for psoriasis?

26.

W h a t is the classic presentation o f lichen planus?

z o o z g H o z

BULLOUS DISEASES AND NEOPLASMS

BULLOUS DISEASES AND NEOPLASMS

3 Question Warm-Up 1.

W h a t findings do the following signs describe, and with what diseases are they associated? ° Deep palpation ofRUQ_—> arrest of inspiration due to pain ° Charcot’s triad (fever, jaundice, RUQ_pain), hypotension, altered mental status ° R LQpain on passive extension of the hip ° RLQ_pain on passive internal rotation of the flexed hip ° LUQpain and referred left shoulder pain ° Ecchymosis of the skin overlying the flank ° Ecchymosis of the skin overlying the periumbilical area

2.

W h a t is the treatment for RSV bronchiolitis?

3.

W h a t is Beck’s triad?

Bullous Diseases 4. W hat are the distinctive features of pemphigus vulgaris and bullous pemphigoid?

Pemphigus vulgaris

Bullous pemphigoid

Bullae appearance

Flaccid; easy to rupture (positive Nikolsky)

Tense, hard; difficult to rupture

Presence of oral lesions

Almost always

Rare (10-35%)

Histologic location of antibodies

Epidermis

Dermal-epidermal junction (Basement membrane)

Type of antibody

Anti-desmosome

Anti-hemidesmosome

5. W hat is the treatm ent and prognosis for pemphigus vulgaris? ° ° ° ° ° °

Steroids - high-dose systemic Azathioprine or cyclophosphamide can be used as a steroid-reducing adjuvant Treat wounds as burns Antibiotics if infection is present Dermatologist referral and possible life-long suppressive therapy Prognosis —fatal if left untreated, mortality of 5% even with treatment

6. W hat is the treatm ent and prognosis for bullous pemphigoid? ° Topical steroids (e.g., clobetasol cream) have now been shown to be more effective (decreased mortality and less complications) than oral steroids for bullous pemphigoid ° If topical steroids are not possible (high cost, difficult to apply, mucus membranes involved) —> oral steroids (e.g., prednisolone)

Melanoma 7.

W hich type of melanoma matches each of the following descriptions? ° ° ° ° °

Most common type of melanoma Non-pigmented melanoma Dark papule on the legs or trunk that bleeds with minor trauma Occurs on palms, soles, or beneath nail plate in patients with dark skin Dark lesion larger than 6mm with irregular, asymmetric borders

End of Session Quiz 8.

W h a t is the classic appearance o f porphyria cutanea tarda?

9.

From the presentation, how m ight pemphigus vulgaris be distinguishable from bullous pemphigoid?

10.

W h a t do the antibodies target in bullous pemphigoid and pemphigus vulgaris?

11.

W h a t is the m ost im portant prognostic indicator in cases o f melanoma?

12.

W h a t is the classic appearance o f basal cell carcinoma?

13. W hat is the classic appearance of squamous cell cancer?

AND HAIR LOSS

PLASTICS, PIGMENTATION AND HAIR LOSS

PLASTICS, PIGMENTATION

3 Question Warm-Up 1.

W h at is the next step in the management o f testicular torsion confirmed with an ultrasound?

2.

W h a t are the causes o f hypovolemic hyponatremia?

3.

Fever + rash + elevated creatinine + eosinophilia. W h a t is the diagnosis?

Pigmentation Disorders 4. W hat are the treatment options for melasma? ° ° ° °

Hydroquinone Azelaic acid (cream) Flucinolone + hydroquinone + tretinoin Prevention by minimizing sunlight exposure and by using an opaque sunblock (titanium dioxide or zinc oxide)

5. W hat are the characteristic features of vitiligo? ° Sharply demarcated patches of complete depigmentation (due to loss of melanocytes) Borders are hyperpigmented More common at acral areas and around body orifices ° Skin is of normal texture (which excludes morphea and lichen sclerosis) ° Associated with thyroid disease in 30% of patients (especially women) ° Most common at ages 20-30

6. W hat comorbidities are associated with vitiligo? Autoimmune disorders: Graves disease, autoimmune thyroiditis, pernicious anemia, type 1 DM , primary adrenal insufficiency, hypopituitarism, alopecia areata, autoimmune hepatitis

7.

W hat is the treatm ent for vitiligo? ° Sunscreen to minimize tanning of normal skin which would increase the contrast ° Dyes and make-up to camouflage depigmented areas ° Corticosteroids if < 10% of skin affected If > 12 years, class III or IV —> e.g., fluticasone propionate ointment or mometasone cream q-day for 4-6 months If < 12 years, class V —> e.g., fluticasone propionate cream ordesonide 0.05% cream daily for 4 months ° Tacrolimus or pimecrolimus (calcineurin inhibitors) - Do not cause skin atrophy like steroids ° Psoralens (topical or oral) + UV light (PUVA or UV-B) by dermatologist for extensive disease ° Surgical mini-grafting an option when medical therapy fails ° Depigmentation of normal skin to match regions of vitiligo using hydroquinone is a last resort

8. W hat treatm ent options are available for treating acanthosis nigricans? ° Treat the underlying disorder which may require weight loss, discontinuation of an offending agent (e.g., glucocorticoids, OCPs), or identification and removal of a malignancy. ° Lightening agents may be used, which often include Retin-A (tretinoin) and topical steroids. ° Fish oil oral supplementation may also be used

Red Vascular Skin Lesions 9.

W hat type of hemangioma does each of the following statements describe? ° ° ° ° ° °

Purple-red lesion on face that does not regress with age Infant with bright-red lesion that regresses over months-years Benign, small red papule that appears on skin with age Bright red papule with radiating blanching vessels Blue, compressible mass that does not regress Red-pink nodule on a child that is often confused with melanoma

10. W hat is the treatm ent for an uncomplicated infantile hemangioma? W hen are infantile hemangiomas worrisome? ° Since most uncomplicated infantile hemangiomas (AKA strawberry hemangioma) gradually resolve within the first two years of life (or at least 10% resolution each year), observation is usually the best treatment ° They are worrisome and require additional treatment (such as systemic steroids) if periorbital, in an airway, or associated with high-output heart failure

PLASTICS, PIGMENTATION AND HAIR LOSS

Hair Loss 11. W hat are the clinical features of alopecia areata? ° ° ° ° °

Asymptomatic, inflammatory, non-scarring areas of complete hair loss May be precipitated by stress Regrowth after 1st attack in 30% by 6 months, in 50% by 1 year, in 80% by 5 years 10-30% will not re-grow hair, 5% progress to total hair loss Obtain syphilis screen, CBC, BMP, ESR, TSH, ANA (to rule out pernicious anemia, chronic active hepatitis, thyroid disease, SLE, Addison) ° Rule out trichotillomania (pulling out one’s hair) —look for broken hair shafts of different lengths, consider shaving a small patch and observe over a few weeks for growth

12. W hat is the treatm ent for alopecia areata? ° Fluocinolone oil and/or shampoo qd ° Intralesional steroid injection - most common therapy for limited involvement, triamcinolone injected into entire patch q4-8 weeks, can be used in girls past age 7-9 and boys past age 10-12 ° Minoxidil topical, works within 12 weeks, not best choice for children ° Anthralin cream - commonly used in children, growth within 2-3 months ° Squaric acid dibutylester (SADBE) or diphenylcyclopropenone (DPCP) rubbed into scalp creates an allergic response resulting in hair growth, both about 60% effective in children, may be best non-injection choice for children ° Topical steroids are ineffective due to poor scalp penetration. Oral steroids work well, but alopecia returns once stopped.

13. W hat is the name given to diffuse stress-related hair loss? W hat is the treatment?

14. W hat is the treatm ent for androgenic alopecia? ° If signs of androgen excess in women, check serum testosterone, DHEA and prolactin ° Men —> finasteride +/- minoxidil. Women —>minoxidil +/- spironolactone. ° Finasteride (Propecia) lm g qd Dose is lmg qd for hair loss (Proscar 5mg qd is used for BPH) Not used in women. Shouldn't even be touched by premenopausal women. Not effective in postmenopausal women. ° Minoxidil (Rogaine) 5% solution Can be used in both men and women. May use with finasteride or spironolactone. 5% works better than 2% ImL bid to dry/involved scalp bid for at least 4 months. Results will be seen by 12 months. Use indefinitely. Discontinuation will result in a return to baseline. SE: hypertrichosis (3%), pruritus, dermatitis, scaling ° Spironolactone 100-200mg PO qd

15. W h a t dermatologic condition matches each o f the following statements? ° Associated with obesity, diabetes or malignancy (especially if over 50) ° Pigmented plaques that appear to be stuck onto the skin ° Black, velvety plaques on flexor surfaces and intertriginous areas ° Rough lesions on sun-exposed skin that are easier to feel than see ° Circular rash with central clearing on the trunk or arms

AND HAIR LOSS

16. A 31-year-old wom an has patches o f hypopigmentation on her skin. Considering the associated comorbidities w ith this condition, what endocrine test m ight you order initially?

PLASTICS, PIGMENTATION

End of Session Quiz

17. W h a t type o f hemangioma does each o f the following statements describe? ° Purple-red lesion on face that does not regress with age ° Infant with bright-red lesion that regresses over months-years ° Benign, small red papule that appears on skin with age ° Bright red papule with radiating blanching vessels ° Blue, compressible mass that does not regress ° Red-pink nodule on a child that is often confused with melanoma 18. W h a t treatm ent is indicated for a newborn w ith an uncomplicated strawberry hemangiom a on the face?

[321 ]

G ynecology 1

M en stru al Physiology

2

M enopause

3

C ontraception

4

A m enorrhea

5

M e n stru al D isorders p a rt 1

6

M e n stru al D isorders p a rt 2

7

P C O S and Pelvic Prolapse

8

G ynecological Infections and S T D s

9

U terine and C ervical N eoplasm s

10 V aginal and O v arian N eoplasm s 11 B enign B reast D isorders 12 B reast C ancer

MENSTRUAL PHYSIOLOGY

3 Question Warm-Up 1.

Anticentromere antibodies are associated w ith w hat condition?

2.

W h a t blistering skin disease has a positive Nikolsky sign?

3.

W h ich blood cell pathology matches each o f the following high-yield descriptions? Associated w ith Epstein-Barr virus (in Africa) Reed-Sternberg cell, cervical lymphadenopathy, night sweats Bence-Jones proteins, osteolytic lesions, high calcium Translocation 14;18 M ost common lymphoma in the US Translocation 8;14 Translocation 9;22 M ost common form of Hodgkin lymphoma “Starry-sky” pattern due to phagocytosis of apoptotic tum or cells H igh hematocrit/hemoglobin, pruritus (especially after hot bath or shower), burning pain in hands or feet

End of Session Quiz 4.

W h at is the mean age o f menarche in the US?

S.

W h at is the definition o f precocious puberty?

6.

W h a t lab findings distinguish true precocious puberty from pseudoprecocious puberty?

7.

W h a t are some o f the causes o f pseudoprecocious puberty?

8.

W h a t is the treatment for central precocious puberty?

9.

W hich phase o f the menstrual cycle is fixed at 14 days, regardless o f cycle length?

10.

F SH triggers the release o f which hormone from the follicle?

MENOPAUSE

3 Question Warm-Up 1.

W h a t nam e is given to diffuse stress-related hair loss? W h at is the treatment?

2.

W h a t are the classic features that distinguish orbital cellulitis from periorbital cellulitis?

3.

W h a t is the classic (but rare) E K G finding in pulm onary embolism?

Menopause 4.

How is menopause diagnosed? ° 12 months of amenorrhea in a woman over 45 is diagnostic and requires no additional work-up ° A woman over age 45 with irregular menses (oligomenorrhea) and menopausal symptoms (hot flashes, mood changes, sleep disturbances) can be assumed to be going through perimenopause ° Seram FSH levels increase in the perimenopausal period and after menopause, but this is of little diagnostic value beyond obtaining a menstrual history and history of symptoms ° If younger than 45, other etiologies for oligo/amenorrhea must be excluded (TSH, serum hCG, prolactin, FSH)

MENOPAUSE

Hormone Replacement Therapy (H R T ) 5. W hat are the pros and cons of hormone replacement therapy for menopause? PROS ° Control of menopausal symptoms (hot flashes, vaginal dryness/atrophy, urinary incontinence, emotional lability) ° Reduced risk of_________________________________ ° Reduced risk of colorectal cancer

CO N S ° Not indicated for the prevention of chronic disease, stroke, heart disease and osteoporosis (USPSTF) ° HRT doubles risk of: Invasive breast cancer (+8 per 10,000) but not noninvasive breast cancer Endometrial cancer Venous thromboembolism (+8 PEs per 10,000) ° Increases risk of stroke by up to 32-41% (+8 per 10,000) ° Increases risk of heart disease by 29% (+7 per 10,000) ° However, if taken at ages 50-59, H R T results in less coronary calcification on C T scan (NEJM 2007;356:2591). This may or may not correlate with less risk of heart disease in women taking H R T during ages 50-59 ° Increases risk of biliary disease and need for biliary surgery

6. W hat non-hormonal options can be used in the treatment of menopausal hot flashes? ° Desvenlafaxine —only non-hormonal drug FDA-approved for hot flashes. Also works as an antidepressant ° Venlafaxine —good choice if any depression, anxiety, fatigue or isolation. Good first-line drug ° Clonidine - good choice if BP control is also needed. SE of dry mouth, constipation and drowsiness ° Gabapentin - about 50% reduction seen in a small trial. Good choice if insomnia, restless leg syndrome, seizure d/o, neuropathy, chronic pain ° Time —about 30-50% of women have symptom improvement within a few months, and most have resolution within 4-5 years ° Placebo - placebo effect is about 20-25% effective in reducing hot flashes

[ 328

7.

Prem ature menopause is defined as menopause before what age?

8.

W h a t is required for a diagnosis o f menopause?

9.

As periods become less frequent during perimenopause, what hormonal changes are occurring?

10.

MENOPAUSE

End of Session Quiz

W h a t are the side effects o f estrogen? Progesterone?

[ 329]

CONTRACEPTION

3 Question Warm-Up 1.

In which patients is bupropion contraindicated?

2.

W h at is the treatment for serotonin syndrome?

3.

A middle-aged man presents for knee pain, and x-ray reveals bilateral calcifications o f the articular cartilage. W h a t is the treatment?

W hat are the absolute contraindications to the use of oral contraceptive pills (OCPs)? ° Pregnancy (although accidental use in early pregnancy is not associated with congenital anomalies) ° History of thromboembolism (DVT, PE) or inherited thrombophilia ° History of estrogen-dependent tumor (endometrial or breast carcinoma) ° Cerebrovascular disease (history of stroke) or CAD ° Poorly-controlled hypertension O ° Hepatic disease/neoplasm (adenoma, cancer, hepatitis, cirrhosis) ° Abnormal vaginal bleeding of unknown etiology ° ___________________________________, neurologic symptoms or vascular involvement (increased risk of stroke)

W hat are the advantages and disadvantages of combination oral contraceptives? Advantages

D isadvantages

° Reliable (< 3% failure rate) ° Reduce risk of endometrial and ° Decreased incidence of ectopic pregnancy ° Menses more predictable, lighter, less painful

° ° ° 0

Daily dosing Do not protect against STDs Breakthrough bleeding Estrogen SE: bloating, breast tenderness, nausea, headaches ° Progesterone SE: depression, acne, hypertension ° Increased risk of DVT ° Elevated triglycerides

W hat type of liver pathology is associated with O C P use? ° Reversible cholestasis O Benign liver tumor which may undergo malignant transformation Incidence is 3-4 per I00K long-term users vs. 0.1 per I00K in the general population Development typically requires high-dose estrogen for > 5 years ° Budd-Chiari syndrome from hepatic vein thrombosis or IVC thrombosis ° Veno-occlusive disease of the terminal hepatic venules and hepatic sinusoids (similar to Budd-Chiari) ° Hepatocellular carcinoma ° Resulting cirrhosis, portal hypertension or liver failure from one of the above

W h at medications are well known for reducing the effectiveness of combination oral contraceptive pills through changes in liver metabolism? ° Antibiotics:_____________________________ (griseofulvin to a lesser degree) ° Antiepileptics (e.g., phenobarbital, phenytoin, carbamazepine, topiramate, oxcarbazepine, primidone) ° Other: St. John’s wort

W h at are the contraindications to IU D placement? ° Current vaginal or cervical infection ° Known pregnancy or desire for pregnancy in the near future ° Severe uterine distortion (bicornuate uterus, cervical stenosis, fibroids distorting the uterine cavity) ° Uterine bleeding that has not yet been worked up ° Copper allergy or Wilson disease —>avoid copper IUD ° Breast cancer —» avoid progesterone IUD

End of Session Quiz 9.

W h a t are 4 different options for emergency contraception?

10. W h a t are the mechanisms o f action o f OCPs?

11. O C P use decreases the incidence of what type of cancer?

AM ENORRHEA

AMENORRHEA

3 Question Warm-Up

[332

1.

A 25-year-old woman is seen in the clinic w ith complaints o f fatigue and chronic joint pain. Physical exam and routine labs reveal a malar rash, mildly elevated temperature and anemia. It is unclear whether this woman’s anemia is caused by autoimmune hemolytic anemia or from her recent menstruation. W h a t test can distinguish between possible autoimmune-mediated hemolytic anemia versus other causes o f anemia, including non-immune mediated hemolytic anemia?

2.

A study shows that taking 325mg o f aspirin a day has no effect on ischemic cardiac events. W h a t type o f error is this?

3.

W h a t labs should be ordered to identify the pathogen in a patient w ith an acute diarrheal illness?

W hat are the first steps in the work-up of a woman with prim ary amenorrhea? °

Thorough history and physical exam Congenital defects identified: imperforate hymen, transverse vaginal septum, vaginal agenesis If signs of hyperandrogenism —> serum testosterone and________________________ to assess for an androgen-secreting tumor If galactorrhea —> serum ___________________________ and thyrotropin to assess for prolactinoma

° °

°

Pelvic sonogram if uterus does not appear to be present or is difficult to assess If uterus absent —* and serum___________________ Androgen insensitivity syndrome (46,XY; elevated testosterone) Abnormal mullerian development (46,X X ; normal female testosterone levels) If uterus present —> and serum___________________ If p-hCG high —> pregnancy If FSH high —> karyotype for Turner syndrome (45,X O ) If FSH low —> cranial MRI for hypothalamic or pituitary disease If FSH normal —► serum prolactin and thyrotropin

A 15-year-old girl comes in for evaluation of prim ary amenorrhea. On physical exam , a bluish bulge is evident where the vaginal orifice should be. W hat is the diagnosis?

W h at are the first steps in the work-up of a woman with secondary amenorrhea? ° Serum [3-hCG to rule out pregnancy ° Thorough history and physical exam ° Serum prolactin (rule out hyperprolactinemia), serum TSH (rule out thyroid disease), serum FSH (rule out ovarian failure) ° If signs of hyperandrogenism —> serum DHEAS and total testosterone ° If all of the above are normal or history of dilation and curettage (D&.C) —> progestin withdrawal test (rule out Asherman syndrome)

End of Session Quiz 7.

A woman presents with prim ary amenorrhea, absent secondary sexual characteristics, and anosmia. W h at is the diagnosis?

8.

W h at is the definition o f premature ovarian failure?

9.

W h a t is the most common cause o f secondary amenorrhea?

10.

W h a t is the initial step in the management o f a woman presenting with secondary amenorrhea and new galactorrhea when the [3-hCG is negative?

11.

W h at are the basic components o f a work-up for secondary amenorrhea?

MENSTRUAL DISORDERS PART I

3 Question Warm-Up 1.

A young child presents w ith thigh muscle weakness, a waddling gait, and pronounced calf muscles. W h a t is the diagnosis?

2.

A female neonate who was born in breech position is found to have asymmetric inguinal and gluteal skin folds on her newborn exam. W h a t is the diagnosis and treatment?

3.

H ow is benign paroxysmal positional vertigo (BPPV) diagnosed? H ow is it treated?

4. W hat are the characteristic features of endometriosis? ° Pelvic pain (most severe during menses, 2-7 days prior to menses and possibly at ovulation) ° 3 Ds: dysmenorrhea, deep dyspareunia and dyschezia (painfiil defecation during menses) ° Difficulties with fertility Physical findings: ° Localized tenderness in the cul-de-sac or uterosacral ligaments (esp. at the time of menses) ° Palpable, tender nodules in the cul-de-sac, uterosacral ligaments or rectovaginal septum ° Pain with uterine movement ° Tender, enlarged adnexal masses ° Adhesions causing a fixed or retroverted uterus

5. W h at are the treatm ent options for treating endometriosis? ° Expectant management —if minimal symptoms or perimenopausal ° Pain control with NSAIDs ° Hormonal therapies: - Combined O CPs dosed continuously - common first-line option with prn NSAIDs GnRH agonist (nafarelin, leuprolide or goserelin) for 6-12 months - induces a medical menopause, used in patients with moderate-severe pain, side effects can be managed by "add-back” hormones - Progestin (PO, IM, IU) - usually 2nd- or 3rd-line because of breakthrough bleeding and other SE, including depression and weight gain Danazol for 6 months —induces a medical menopause but no “add-back" therapy available, usually a 3rd- or 4th-line option Aromatase inhibitors (anastrozole or letrozole) —used with GnRH agonist or OCP, otherwise follicular cysts develop ° Surgical intervention Laparoscopic surgery to confirm diagnosis and ablate ectopic endometrial tissue and lyse adhesions Hysterectomy with bilateral salpingo-oophorectomy (TAH/BSO), lysis of adhesions (LO A) and removal of endometrial implants

MENSTRUAL DISORDERS PART I

6. W hat is the first-line treatment for a young, infertile woman with obvious signs and symptoms of endometriosis?

End of Session Quiz 7.

W h a t is the treatment o f choice for primary dysmenorrhea?

8.

W h a t medications are effective in the treatment o f PM S and PM D D ?

9.

W h a t is the first-line treatment for endometriosis?

10.

W h a t is the most common cause o f female infertility?

11.

W h a t are the “3 D s” o f the presentation o f endometriosis?

MENSTRUAL DISORDERS PART 2

3 Question Warm-Up 1.

W h a t do an elevated ery th ro p o ietin level, elevated hem atocrit and norm al O a satu ratio n suggest?

2.

A n ew born is found to have a congenital h e a rt disease th a t causes early cyanosis. W h a t m edication does this new born need?

3.

W h a t is th e difference betw een a M onteggia fracture and a G aleazzi fracture?

4. W hat is the m ost likely cause of abnormal uterine bleeding (A U B ) in each of the following patients? M ost common cause of abnormal uterine bleeding Positive |3-hCG + intrauterine pregnancy + closed os Enlarged uterus + menometrorrhagia for months Bleeding associated w ith severe menstrual pelvic pain M enorrhagia + perimenopausal AUB started w ith menarche Positive |3~hCG + severe pain + no fetus in uterus on US M etrorrhagia especially after intercourse + no pain + normal-sized uterus Depression + constipation + AUB

MENSTRUAL DISORDERS PART 2

[ 338 ]

End of Session Quiz 5.

W h a t is the most common cause o f irregular, heavy uterine bleeding?

6.

W hen is an endometrial biopsy a necessary part o f the work-up for abnormal uterine bleeding?

7.

W h a t is the most common clotting disorder that can cause menorrhagia? W h a t lab values are abnormal?

PCOS AND PELVIC PROLAPSE

3 Question Warm-Up 1.

A hospitalized patient is suspected o f having an infection. W h a t should be done before empiric antibiotics are started?

2.

W h a t intracranial abnormality is associated w ith A D PK D ?

3.

Diming a routine exam, a 70-year-old m an is found to have new-onset iron deficiency anemia. W h a t should you suspect and investigate further?

4. W h at are the features of pelvic prolapse? ° Types - Cystocele - Prolapse of bladder into vagina - Rectocele - Prolapse of rectum into vagina Enterocele —Prolapse of small bowel into vagina (usually following hysterectomy) - Uterine prolapse —Prolapse of uterus into vagina ° Historical clues: Pelvic pressure or heaviness, obvious protrusion of tissue out of the vagina, “feels like I’m sitting on an egg” ° Treatment - Mild —» pelvic floor exercises and/or physical therapy with behavior modification (e.g., timed voiding) - Moderate —» pessary Severe —> surgical correction

PCOS AND PELVIC PROLAPSE

End of Session Quiz 5.

W h at is the most common cause o f hirsutism in the US? W hat lab findings are used to make the diagnosis?

6.

For which type o f cancer are women with P C O S at an increased risk and why?

7.

W h at medications are used in the treatment o f PCOS?

GYNECOLOGICAL INFECTIONS AND STDS

3 Question Warm-Up 1.

W h e n can lactational amenorrhea be relied upon as an effective method o f contraception?

2.

W h a t is the treatm ent for acute angle-closure glaucoma?

3.

W h a t rash is classically described as “dew drops on rose petals”?

4. W hat are the diagnostic features of pelvic inflammatory disease (PID)? ° ° ° ° ° ° °

or adnexal tenderness Leukocytosis on CBC New/unusual purulent cervical or vaginal discharge W BCs on wet prep of vaginal secretions Temp > 101° F Elevated ESR or CRP Imaging may reveal: Thickened or fluid-filled fallopian tubes - +/- fluid in the pelvic cul-de-sac

G YN ECO LO GICAL INFECTIONS AND STDS

[ 342 ] I

End of Session Quiz 5.

W h at are the distinguishing features o f bacterial vaginosis, Candida vaginitis and Trichomonas infection?

6.

W h a t is the treatment for gonorrhea? Chlamydia?

7.

A sexually-active woman presents with the classic symptoms o f cystitis. Gram stain of the urine shows no organisms. W h a t organism do you suspect is the cause o f this patient’s symptoms?

8.

W h a t medications can be used in the treatment o f syphilis?

UTERINE AND CERVICAL NEOPLASMS

c H m

70

zm >

Z

,? Question Warm-Up 1.

W h a t is the antidote to each o f the following toxins? Anticholinesterases, organophosphates M ercury Carbon monoxide

o n m 30 < n > o 03 z cn

Heparin Isoniazid

2.

W h a t is the laboratory work-up for abnormal uterine bleeding?

3.

A patient presents w ith a painless, pruritic papule w ith regional lymphadenopathy th at evolves over 7-10 days into a necrotic ulcer w ith a black eschar. W h a t is the diagnosis and treatment?

4. W h at are the current recommendations for Pap screening in patients with no history of abnormal Pap? 0 Initiate screening at age_________________ 0 Frequency of Pap smears: Every 3 years for ages 2 1-29 Patients 30+ may continue Pap smear every 3 years, or may opt for Pap smear + HPV testing every 5 years (preferred by some professional organizations) Women in higher risk groups may need more frequent testing 0 Screening may be stopped at age 65 if adequate normal Paps 0 If hysterectomy for benign disease, there is no need to screen for cervical cancer -

[343]

UTERINE AND CERVICAL NEOPLASMS

[ 344 ]

End of Session Quiz 5.

W h a t are the indications for an endometrial biopsy?

6.

W h a t is the next step in the management o f a C IN 2 cervical lesion identified on biopsy in a woman who has completed fertility?

7.

W h at is the next step in the management of an ASCUS Pap smear with positive H P V test?

8.

W h at is the next step in the management o f an AGUS Pap smear?

VAGINAL AND OVARIAN NEOPLASMS

3 Question Warm-Up 1.

W h a t is the differential diagnosis for eosinophilia?

2.

A 60 year old male smoker is found to have a varicocele that does not empty w hen the patient is recumbent. W h a t should you be suspicious o f in this patient?

3.

W h a t acid-base disorder would cause the following lab values? pH

hco

3

pco2

7.30

12

26

7.25

18

37

7.23

24

55

D iso rd e r

4. W h at are the general treatm ent strategies for squamous cell cancer of the vagina? Stage

T re atm en t

Stage I —less than 2cm Stage I —greater than 2cm Stage II, III, and IV

5.

Lichen Sclerosis ° Chronic inflammatory condition of the anogenital region, most commonly affecting postmenopausal women ° Classic late findings - ivory or porcelain-white macules and plaques with pruritus ° Treatment - Low threshold for punch biopsy to r/o SCC; steroids (clobetasol) or pimecrolimus

End of Session Quiz 6.

W h a t are the symptoms o f ovarian cancer?

7.

W h a t are the risk factors for ovarian cancer?

8.

W h a t serum marker may be elevated in cases o f ovarian cancer?

9.

W h a t ultrasound findings are consistent w ith benign ovarian tumors? W ith malignant ovarian tumors?

10.

W h a t type o f ovarian tum or is associated with psammoma bodies? Estrogen excess? Androgen secretion?

BENIGN BREAST DISORDERS

3 Question Warm-Up 1.

W h a t is the complication o f gonorrhea or chlamydia that infects the capsule of the liver?

2.

W h a t type o f heart m urm ur fits each o f the following descriptions? » Diastolic murmur heard best in left lower sternum that increases with inspiration ° Late diastolic murmur with an opening snap (no change with inspiration) ° Systolic murmur heard best in the second right interspace ° Systolic murmur heard best in the second left interspace ° Late systolic murmur heard best at the apex ° Diastolic murmur with a widened pulse pressure ° Holosystolic murmur that is louder with inspiration at the left lower sternum ° Holosystolic murmur heard at the apex and radiates to the axilla

3.

W h a t is the first-line treatm ent for pediculosis capitis and pediculosis pubis?

4. W h at is the differential diagnosis of gynecomastia? ° Puberty (resolves spontaneously in 6m to 2yrs) ° Drugs: spironolactone, marijuana (THC), chronic alcohol use, cimetidine, ketoconazole, estrogens, digoxin ° Herbal agents: tea tree oil, lavender oil ° Cirrhosis ° Hypogonadism (e.g., Klinefelter, hyperprolactinemia) ° Testicular germ cell tumor ° Hyperthyroidism ° Hemodialysis patients

BREAST DISORDERS BENIGN

End of Session Quiz 5.

W hich type o f breast disease matches each o f the following descriptions? ° Most common tumor in teen and young women ° Most common mass in patients 35-50 ° Often presents with serous or bloody nipple discharge

6.

W h at drugs are notable for causing gynecomastia?

BREAST CANCER 3 Question Warm-Up 1.

A n 8-year-old child was in a motor vehicle accident and now requires an emergency blood transfusion. H er parents are not present, but the child states she is a Jehovah’s Witness. W h a t do you do?

2.

A 50-year-old female smoker presents w ith hematuria. W h a t do you suspect?

3.

W h a t statistical calculation looks at true positives and divides them by the number o f patients w ith the disease?

End of Session Quiz 4.

W h a t is the m ost common breast cancer? W h a t is the most common site for breast cancer?

5.

W h a t findings are suspicious on a mammogram?

6.

W h a t is the treatm ent for ductal carcinoma in situ o f the breast?

7.

O nce you have ruled out invasive cancer, w hat is the management o f LCIS? W h y is drug therapy so effective?

O bstetrics 1

N orm al P regnancy Physiology

2

P renatal C are

3

M edical C om plications p a rt 1

4

M edical C om plications p a rt 2

5

M a tern al D ru g Use

6

C ongen ital Infections

7

O bstetric C om plications p a rt 1

8

O bstetric C om plications p art 2

9

O bstetric C om plications p a rt 3

10 O bstetric C om plications p a rt 4 11 L & D : A ssessm ent o f Fetus 12 L&JD: L abor 13 L & D : M alpresentation and C esarean Section 14 P o stp artu m C are

NORMAL PREGNANCY PHYSIOLOGY

3 Question Warm-Up 1.

W h a t is the difference between the following disorders? ° Schizotypal personality disorder ° Schizophrenia ° Schizoaffective ° Schizoid personality disorder ° Schizophreniform ° Brief psychotic disorder

2.

W h a t term describes heavy bleeding during and between menstrual periods?

3.

W h a t is the m ost common type o f tracheoesophageal fistula?

4. W hat change is responsible for physiologic anemia of pregnancy? A t what gestational age is it most apparent? W hen should anemia in pregnancy be treated with oral iron? ° This greater increase in plasma volume as compared to red cell mass is most apparent during the second trimester ° Treat anemia in pregnancy with iron replacement when hemoglobin falls below______ g/dL in the first or third trimesters, or when less than g/dL in the second trimester

NORMAL PREGNANCY PHYSIOLOGY

End of Session Quiz 5.

Between what weeks gestation is a fetus most susceptible to teratogens and why?

6.

W h at effect does pregnancy have on blood pH?

7.

H ow do the following parameters change during pregnancy? • ° • »

8.

TSH Blood pressure Cardiac output Ventilation

W h a t hormone contributes to glucose intolerance often seen in pregnancy?

PRENATAL CARE

3 Question Warm-Up 1.

W h a t is the most sensitive and specific lab test for the diagnosis o f chronic pancreatitis?

2.

W h a t is the classic presentation o f poststreptococcal glomerulonephritis?

3.

W h a t would be the concern in a sickle cell patient that contracts erythema infectiosum (“fifth disease”)?

4. W h at is Goodell’s sign? W hat is Chadw ick’s sign? W hat is Hegar’s sign? ° Goodell’s sign —softening and cyanosis of the

at 6 weeks gestation

° Chadwick’s sign - bluish discoloration of the _ congestion at 8-12 weeks gestation

. due to vascular

° Hegar’s sign - softening of th e ___

at 6 weeks gestation

5.

How do |3-hCG levels change during early pregnancy?

6.

How many additional calories are needed on a daily basis during pregnancy and breastfeeding? ° During pregnancy, an additional 340 kcal/day is needed in the second trimester and 452 kcal/day is needed in the third trimester ° During breastfeeding, an additional 500 kcal/day is required to breastfeed, but since fat stores developed during pregnancy begin to be mobilized, only an additional dietary 330 kcal/day is needed

7.

W h at dose of folic acid is recommended to mothers for the prevention of neural tube defects? o _____________________ folic acid daily is recommended to all women of childbearing years (CDC rec.) to reduce neural tube defects by 57% ° Higher doses of folic acid daily reduce the risk of neural tube defects even more ° If previous child with neural tube defect, recommended folic acid intake starting the month prior to pregnancy is 4 mg daily (AAP, ACOG)

PRENATAL CARE

8. W here would you expect to find the fundus of the uterus on physical exam throughout pregnancy? ° ° ° °

12 weeks 16 weeks — 20 weeks 20-36 weeks -

9. W hat are the indications for percutaneous umbilical blood sampling (PUBS)? In most cases fetal amniocentesis is sufficient and safer than PUBS. PUBS (AKA cordocentesis) is preferred only for confirmation o f_________________________ . with possible___________________________________ .

10. In pregnancy, which vaccines are currently indicated, and which are contraindicated? Vaccines indicated in pregnancy ° ________________ qlO years (OK in pregnancy. Dose TDaP in 2nd or 3rd trimester.) ° ________________ annually (OK in pregnancy.) ° ________________ , _________________ , and_________________ if indicated (OK in pregnancy) Vaccines contraindicated in pregnancy O o o o

Postpartum or post-abortion vaccines ° If rubella non-immune (titer < 10), then rubella vaccine prior to discharge ° If no h/o tetanus vaccine in last 10 years, then administer TD aP vaccine prior to discharge ° If no evidence of varicella immunity (h/o chickenpox/shingles, h/o vaccine, or serologic confirmation), then varicella vaccine prior to discharge. 2nd dose 4-8 weeks after the first

End of Session Quiz 11. W h a t additional supplements should be given to complete vegetarians during pregnancy?

12.

13.

W h a t supplements should be given to women on anticonvulsants during pregnancy?

W h a t disorders can increased nuchal translucency indicate?

14. Q uad screen shows decreased A FP, decreased unconjugated estriol, elevated inhibin A and elevated j3-hCG. W h a t diagnosis do you suspect?

MEDICAL COMPLICATIONS PART I

MEDICAL COMPLICATIONS PART I

3 Question Warm-Up 1.

W h at is the differential diagnosis for the dislocation of the lens o f the eye?

2. W h a t type o f infection causes honey-crusted lesions usually around the nose or mouth?

3. W h a t is the most common cause o f morbidity and mortality in patients w ith SLE?

4. A 30-year-old woman with hypertension is currently on an A C E inhibitor, but has just discovered that she is pregnant. Knowing that A C E inhibitors are teratogens, you decide to switch her to a different medication for her hypertension. W hat medications are commonly used in the management of chronic hypertension in pregnancy?

End of Session Quiz 5.

A pregnant woman at 24 weeks gestation has a random glucose o f 130. W h a t is the next step in her management?

6.

W h a t is the drug o f choice for gestational diabetes? W h a t other drugs are safe for use in pregnancy for gestational diabetes?

7.

W h a t fetal cardiac defect is most associated with pre-pregnancy diabetes?

8.

W h a t diagnosis would you suspect in a pregnant patient w ith hypertension prior to 20 weeks gestational age?

9.

For how long is magnesium sulfate continued after delivery in preeclampsia? In eclampsia?

10.

W h a t are the features o f H E L L P syndrome?

MEDICAL COMPLICATIONS PARI

MEDICAL COMPLICATIONS PART 2

3 Question Warm-Up 1.

W h a t is the next step in the evaluation o f each o f the following patients? Pelvic fracture + DPL shows blood in the pelvis Pelvic fracture + DPL shows urine in the pelvis Pelvic fracture + DPL shows nothing + hemodynamic instability Blunt abdominal trauma + unstable vital signs + FAST shows fluid in pelvis Blunt abdominal trauma + unstable vital signs + FAST shows no fluid in pelvis Blunt abdominal trauma + unstable vital signs + FAST inconclusive Blunt abdominal trauma + stable vital signs Abdominal stab wound + hypotensive or signs of peritonitis

4.

2.

W h a t antibiotic combinations are used in the outpatient treatment o f diverticulitis?

3.

W h a t is the next step in the management o f a woman w ith uncomplicated cystitis?

How is hyperemesis gravidarum distinguished from normal morning sickness? 0 Weight loss exceeding_______ %of pre-pregnancy body weight and detection of ketonuria due to starvation are usual markers used to distinguish hyperemesis gravidarum from morning sickness.

5. W hat work-up should be performed in a patient with hyperemesis gravidarum? ° Weight, orthostatic blood pressures ° Serum_________________________ , serum electrolytes and urine ketones ° (Expected, non-worrisome lab abnormalities associated with vomiting: elevated AST and ALT (but each < 1000), elevated amylase and lipase (but each < 5x normal), and elevated bilirubin (but < 4 mg/dL)) ° Ultrasound to detect gestational trophoblastic disease (molar pregnancy) and multiple gestations

W h a t are the treatm ent options for nausea in pregnancy? ° Conservative O TC nausea/vomiting control: Vitamin__________ 25mg tid, Ginger 250mg tid, Unisom (___________________ )I2.5mg bid(A),acupressure ° Prescription nausea/vomiting control: promethazine (C), ondansetron (B) or granisetron (B), metoclopramide (B) ° If dehydrated, IV fluids in ER or as inpatient (daily Chem 7, Mg, Phos) ° If vomiting for more than 3 weeks, multivitamins + thiamine lOOmg IV daily for 2-3 days ° If refractory to above and losing weight, then NG tube feeds (parenteral nutrition only as a last resort)

End of Session Quiz 7.

W h a t is the treatm ent for D V T in pregnancy and how long is it continued? W h a t drug do you N O T give during pregnancy?

8.

W h a t dipstick finding is considered diagnostic o f a U T I?

9.

W h a t is the first-line treatm ent for hyperemesis gravidarum?

MATERNAL DRUG USE

3 Question Warm-Up 1.

W h a t lab is often elevated in patients with an upper G I bleed?

2.

A 30-year-old female African immigrant presents with hematuria. W h a t do you suspect in this patient?

3.

A young woman thrown from a horse presents w ith low back pain, urinary retention, saddle anesthesia and decreased rectal tone. W h a t is the treatment for her condition?

W hat adverse effects can be seen with maternal marijuana use during pregnancy? ° At least six joints per week —>small head circumferences in children at all ages (Ottawa study) ° No association between prematurity or congenital anomalies ° Increases risk of using alcohol and cigarettes during pregnancy, which are both harmful to the fetus ° Small studies show problems later in life including increased incidence of psychiatric problems (ADHD, depression and substance abuse) and increased incidence of certain cancers (non-lymphoblastic leukemia, rhabdomyosarcoma and astrocytoma)

W hat adverse effects can be seen with maternal A C E inhibitor or A R B use? _________(+/- renal failure) —> oligohydramnios —» intrauterine growth restriction (IUGR), limb contractures,_______________________________, lack of skull ossification (—> craniofacial deformation), in utero death

W hat doses of radiation are considered safe in pregnancy? ° Less than 0.05 Gy (5 rads) over the pregnancy - no evidence of any harm to the fetus ° Risk of fetal malformations increases after 0.10 Gy (10 rads) ° Examples of fetal radiation exposure in maternal imaging: IVP (up to 0.9 rads), barium enema (up to 1.6 rads), abdominal C T (about 0.25 rads) ° (1 Gray = 100 rads)

7.

Identify the following teratogens based on the defects: Phocomelia

Deafness Spina bifida, hypospadias Cardiac (Ebstein) anomalies Craniofacial defects, IU G R, CNS malformation, stillbirth Fingernail hypoplasia, craniofacial defects Central nervous system, craniofacial, ear and cardiovascular defects Goiter, cretinism Cerebral infarcts, intellectual disability Clear cell vaginal cancer, adenosis, cervical incompetence 8.

W h a t are the m ain fetal complications o f tobacco use in pregnancy?

9.

H o w are migraines treated in pregnancy and why?

USE

Yellow or brown teeth

MATERNAL DRUG

End of Session Quiz

CONGENITAL INFECTIONS

3 Question Warm-Up 1.

W h a t is the definitive cure for preeclampsia?

2.

W h a t is the treatment for macular degeneration? W h a t is the treatment for retinal detachment?

3.

W h a t are the characteristic findings in tertiary syphilis?

W hat are the signs and symptoms seen in a newborn exposed to rubella virus in utero (congenital rubella)? ° Findings at birth: IUGR, radiolucent bone disease, hepatosplenomegaly, thrombocytopenia, purpuric skin lesions (blueberry muffin rash), hyperbilirubinemia ° Sensorineural deafness ° Cataracts, glaucoma ° Cardiac malformations:_________________________________ ° Neurologic sequelae: intellectual disability, meningoencephalitis, behavior disorders

W hat are the features of congenital syphilis? Early manifestations (first 5 weeks of life) ° Hepatosplenomegaly, elevated LFTs ° Hemolytic anemia, jaundice ° Rash followed by desquamation of hands and feet ° Snuffles (blood-tinged nasal secretions) ° Radiographic changes at birth: metaphyseal dystrophy and periosteitis Late manifestations (if left untreated in the first 3 months of life) ° Hutchinson teeth (notching or blunting of the upper incisors) ° Saddle nose deformity ° Frontal bossing ° Saber shins (anterior bowing of the tibia)

° 90% are asymptomatic at birth —>15% of these go on to develop progressive hearing loss (usually unilateral) ° Symptoms at birth: small for gestational age, hepatosplenomegaly, petechiae/purpura, jaundice ° Neuro: microcephaly, seizures, intracranial calcifications, feeding difficulties, hydrocephaly ° Ophtho: chorioretinitis, optic atrophy, central vision loss ° Thrombocytopenia, hemolytic anemia

7. W hat congenital defects are associated with new varicella infection during pregnancy? ° Skin dermatomal scarring ° ° ° ° °

Chorioretinitis, cataracts, microphthalmos, nystagmus, Horner syndrome Microcephaly, cortical atrophy, intellectual disability Hypoplasia of the hands and feet Low birth weight Early death

8. W hat congenital defects are associated with maternal zoster reactivation?

9.

W hen should you provide intrapartum antibiotic prophylaxis for Group B Strep (GBS)? ° ° ° °

GBS detected on vaginal-rectal screening culture a t_________________ weeks GBS bacteriuria during the current pregnancy History of early-onset GBS in a previous infant Intrapartum fever (> 38°C or 100.4°F), preterm labor (< 37 weeks gestation) or prolonged rupture of membranes (>18 hrs) if previous screening was not done

10. W hat antibiotics can be used for intrapartum prophylaxis for Group B Strep (GBS)? ° ° ° °

_________________ 5 million units IV, then 2.5 million units IV q4 hours or _________________ 2g IV, then lg IV q4 hours If penicillin allergic (h/o rash only), cefazolin 2g IV then lg q8 hours If penicillin allergic (h/o airway compromise), then GBS culture with antibiotic sensitivity testing + one of the following: vancomycin lg IV ql2 hours (or if sensitivity is known, use clindamycin 900mg IV q8 hours or erythromycin 500mg IV q6 hours instead of vancomycin)

CONGENITAL IN FE C T IO N S

W hat congenital defects are associated with in utero CM V infection?

CONGENITAL INFECTIONS

End of Session Quiz 11. W hat are the indications for Group B Strep prophylaxis?

12. W h a t congenital infection is a leading cause o f preventable blindness?

13. W hich congenital infection is associated with each o f the following defects? ? Initially asymptomatic, but later develops a unilateral hearing loss o Hydrocephalus, intracranial calcifications, chorioretinitis • Rash, deafness, cataracts ° Hearing loss, chorioretinitis, intracranial calcifications ° PDA or pulmonary artery stenosis oligohydramnios

W hat are the signs and symptoms of magnesium toxicity? W hat is the reversa agent? O o

° Reversal asrent is compromise.

(9.6-12 msr/dD (12-18 mg/dL) (24-30 mg/dL) la; IV over 5-10 min for situations of cardiorespiratory

End of Session Quiz 6. W hat are the risk factors for placenta previa?

7.

W h a t is the management o f a woman in labor who has a complete placenta previa?

8.

W h a t are the risk factors for placental abruption?

OBSTETRIC COMPLICATIONS PART 4

O CD

EE n n o

3 Question Warm-Up 1.

W h a t is the treatm ent for epididymitis?

2.

W h a t medications are used in the treatm ent o f W egener’s granulomatosis?

3.

A patient has signs o f peritonitis 6 hours after sustaining blunt traum a to a fully distended bladder, and you are suspicious o f a rupture o f the bladder. W h at portion o f the bladder m ust have been injured to allow for a chemical peritonitis to develop?

n > H

o z on 5 73

Gestational Trophoblastic Disease 4. W hat is the treatm ent for metastatic choriocarcinoma? 0 ______________________ to eradicate any drug-resistant local disease and shorten the course of chemotherapy Chemotherapy Single agent for stage I and II —. . or dactinomycin Combination for stage ll-IV - (EMA/CO) etoposide + methotrexate + dactinomycin, then cyclophosphamide + vincristine ° If future fertility is desired —» chemotherapy alone, then hysterectomy only if chemotherapy is ineffective

Infertility 5. W hat are the first steps in the work-up of an infertile couple? ° ____________ collected after 48-72 hours of abstinence (30-40% of infertility is due to the male’s sperm) ° Evaluation for anovulatory cycles (20%) Careful menstrual history Basal body temperature monitoring - rise in progesterone (2 days after LH surge) corresponding to I day after ovulation Home urinary ovulation test (detects LH surge in urine I day prior to ovulation), then postovulation serum progesterone level ( > 5 ng/mL indicates ovulation) +/- Endometrial biopsy on day 26 of cycle ° _______________ to r/o anatomic disorder (30%). Performed after menses cessation, but prior to ovulation. ° +/- Postcoital test: Performed 1-3 days prior to ovulation, 2-12 hours after intercourse.

[ 373]

W hat are some of the different anatomic causes of infertility in women? ° Scarring of fallopian tubes most commonly from prior STD O ° Adhesions from prior surgery or pelvic inflammation (STD, appendicitis, IBD) Tumor, fibroids (leiomyomata) ° Traumatic disruption of normal anatomy ° Congenital anomalies such as septate uterus

End of Session Quiz 7.

W h at is typically included in an infertility work-up?

8.

A 19-year-old G2P1 presents at 9 weeks gestation. She is vomiting all day every day and has lost 7% o f her body weight. O n ultrasound, no gestational sac is found, but rather, there is a “snow storm” appearance to the uterine contents. W h a t is the management o f this patient?

9.

This same patient is lost to follow-up, only to present back to clinic 8 months later complaining o f vaginal bleeding and hemoptysis. H er uterus is enlarged, but on ultrasound, there is no gestational sac. Rather, there is a uterine mass w ith a mix o f hemorrhagic and necrotic areas w ith parametrial invasion. W h a t is her prognosis?

L&D: ASSESSMENT OF FETUS

3 Question Warm-Up 1.

W h ich antibiotics should be avoided during pregnancy due to potential teratogenic effects?

2.

W h a t laparoscopic findings can be seen in endometriosis?

3.

W h y is thiam ine given in a glucose infusion to alcoholics w ith hypoglycemia?

4. Abnorm al fetal heart rates * Sensitivity is only about 85% and specificity is poor —>many infants with nonreassuring F H R are in good condition. ° Fetal tachycardia —FH R > 160bpm for > 10 minutes ° Fetal bradycardia - FH R < llObpm for > 10 minutes ° Sinusoidal —baseline 120-160 bpm with oscillating amplitude of 5-15 bpm, often due to fetal anemia ° Loss of variability —poor short-term or long-term variability, due to fetal sleep, CNS depression, or fetal acidosis; normal variability ranges from 6-25 bpm ° Early decels —begin with uterine contraction, due to pressure on the fetal head ° Variable decels - begin before, during, or after uterine contractions (variable onset), rapid fall in F H R often below lOObpm and rapid return to baseline, due to cord compression ° Late decels —begin after the uterine contraction, maximal after peak of contraction, and return to baseline after contraction complete; due to uteroplacental insufficiency / fetal hypoxia

5. W hat is the differential diagnosis for fetal tachycardia? o

o

0 Fetal anemia ° Fetal tachyarrhythmias (HR > 200)

° Maternal thyrotoxicosis ° Drugs or medications (_ ° Fetal hypoxia

° Fetal immaturity

., atropine)

6. W h at type of fetal surveillance strategy is typical for high-risk pregnancies?

7.

W h a t is considered a norm al, reactive non-stress test?

L&D: ASSESSMENT OF FETUS

8. What are the first steps in the management of non-reassuring fetal heart tones during labor? ® ° ° ° °

Place maternal 0 2 + t urn off Pitocin (remove cervidil) + turn mom to left side Correct hyperstimulation if needed w ith 0.25mg SubQ_ Correct any maternal hypotension (often associated with epidural). IVF bolus if needed SVE and place FSE (check for cord prolapse) Consider need for intervention such as amnioinfusion or C-section

End o f Session Quiz 9.

M atch the following statements to the type o f deceleration with which they belong: A check mark-shaped fetal heart tracing Onset either before, during, or after uterine contraction Consistent dips in fetal heart tones when uterine contractions begin Occur after uterine contraction has begun Unpredictable changes in fetal heart tone tracing

10. W h a t is the usual physical cause o f each o f these types o f decelerations? ° Early ° Variable ° Late 11. W h a t would be some contraindications to fetal scalp electrode placement?

i 376 |

L&D: LABOR

3 Question Warm-Up 1.

A patient comes to the E R s/p M VA with a suspected tib/fib fracture. The patient s lower leg is pale, the dorsalis pedis pulse is absent, and the patient has pain w ith passive motion o f the leg. W h a t is the treatment?

2.

Undescended testes put a patient at higher risk for what condition?

3.

A n African-American teenager presents to the E R w ith right hip pain and a hematocrit o f 25%. W h a t is the most likely diagnosis?

Stages of Labor 4.

W hat defines prolonged active phase of labor? ° Nulliparous 2 hours in multips and > 3 hours in primips (with epidural) Management ° Reassess the 3 Ps ° Placement of IUPC to better assess “Power” (> 200 MVU/lOmin) ° Augmentation with oxytocin to augment “Power” ° C-section if power, passenger and passage are unable to be further augmented

Induction of Labor 8. W hat is the definition of uterine hyperstimuiation (tachysystole)? W hat adrenergic agonist is particularly helpful in reversing uterine hyperstimuiation? ° Uterine hyperstimuiation is defined by one of the following: > 5 contractions over 10 minutes, averaged over a 30-minute window A single contraction > 2 minutes in duration

° Terbutaline 0.25mg sub-q is often used as a tocolytic to stop uterine contractions

End o f Session Quiz 9.

W h a t is the definition o f arrest o f descent?

10.

One hour into the active stage o f labor, a fetus’ heart tones become nonreassuring. W h a t actions are taken immediately?

11.

In addition to the actions taken above, w hat else would you do to manage uterine hyperstimuiation?

12.

W h a t is the role o f fetal pulse oximetry in labor and delivery?

L&D: MALPRESENTATION AND CESAREAN SECTION

3 Question Warm-Up 1.

W h a t are the symptoms o f a lacunar stroke?

2.

W h a t are the classic symptoms o f placenta previa?

3.

W h a t is the next step in the evaluation o f a patient w ith two consecutive Pap smears w ith atypical squamous cells o f undeterm ined significance (ASCUS)?

4. W hat are the management options in the case of a breech presentation? ° Spontaneous version often occurs (25% of the time after 36 weeks) ° C-section at time of labor is currently the standard of care when cephalic version is unsuccessful, due to a decrease in neonatal morbidity and mortality °_____________________ should be offered to all women with breech pregnancy when near term (after__________ weeks) (ACOG). Success rate is 35-75% ° Vaginal delivery only if there is no time for a C-section Deliver the feet and legs - Deliver the body (no traction above the pelvis) and rotate - When scapula visible, sweep posterior arm Sweep anterior arm - Rotate head to O A position Flex head for delivery by applying fingers to nose/maxillary prominences and applying maternal suprapubic pressure.

5.

H ow should a breech presentation be managed after 36 weeks gestation?

6.

W h at are some indications for cesarean section?

7.

About what potential events must patients considering VBAC be counseled?

L&D: MALPRESENTATION

AND CESAREAN SECTIO N

End of Session Quiz

[ 380 ]

POSTPARTUM CARE 3 Question Warm-Up

I

~o

ooo H "o > H C 2

i > y

( " 'V '

1.

H ow old would a child have to be before diagnosis o f a nocturnal enuresis could be made?

2.

A 19-year-old college student comes to the student clinic for evaluation o f recent fatigue and sore throat. She says that she has never gotten so tired w ith sore throats in the past, but this one has “wiped her out.” She does not remember having any sick contacts. O n exam, she has posterior cervical lymphadenopathy, fever, and an easily palpable spleen. W h a t test might confirm the diagnosis?

m

W h a t do an elevated erythropoietin level, elevated hematocrit and normal 0 2 saturation suggest?

Breastfeeding 4. W hat are some of the contraindications to breastfeeding? Hepatitis B and C are present in breast milk, but no transmission has ever been reported. Educate patients of this, and let them make the choice. Need to use medications contraindicated in breastfeeding such as________________ , chloramphenicol, topiramate, antineoplastic agents o r________________ . Use of illicit drugs Infantile galactosemia OK in breastfeeding: - __________________ (must also monitor levels in the newborn) - __________________ (does not pass into breast milk) - __________________ (passes into breast milk but appears to be relatively safe) (passes into breast milk and generally recommended to limit consumption to I glass a day)

I [381 ]

POSTPARTUM CAR

5. W hat is the treatment for mastitis in a postpartum woman? ° Continue nursing and/or pumping breast milk ° Rest and ibuprofen ° Antibiotic choices for 10-14 days: dicloxacillin cephalexin, amoxicillin-clavulanate (if no response to other agents in 24-48 hours), TMP-SMX (for presumed MRSA), add metronidazole 500mg PO tid (if odor suspicious of anaerobes) ° ± Incision and drainage

6.

How is a clogged milk duct (galactocele) distinguished from mastitis?

7. W hat type of oral contraceptive can be given to lactating women?

8. W hat is the treatment for a woman who does not wish to breastfeed postpartum?

Postpartum Bleeding 9. W hat are the risk factors for uterine atony? ° Uterine overdistention: multiple gestations, polyhydramnios, macrosomia ° Exhausted myometrium: prolonged labor, oxytocin stimulation ° Decreased ability to generate contractions: chorioamnionitis, use of Mag sulfate (e.g., in preeclampsia), general anesthesia, uterine fibroids ° Prior history: multiparity, prior history of postpartum hemorrhage

10. W hat are the treatment options for uterine atony/postpartum hemorrhage? o

° ° ° °

__________________________ 10 units IM xl (or IV) Methergine (methylergonovine) 0.2mg IM (ergot agent that is contraindicated if HTN) Hemabate (PGF2 ) 0.25mg IM or intrauterine (contraindicated if asthma) Surgical options: uterine artery ligation, internal iliac artery ligation, selective arterial embolization or hysterectomy ° Dilation and curettage (D&.C) if there is retained placenta ° Tamponade

[ 382

Postpartum Endometritis II.

W hat are the characteristic features of postpartum endometritis? 0 Incidence < 3% after vaginal delivery, but increased risk with c/s, prolonged ROM, multiple cervical checks, manual placental removal, internal monitors (IUPC) ° Fever on postpartum day 1-7 (temp > 100.4°F twice or > 101°F) O ° Absence of other potential fever etiologies (especially UTI) ° May also have: foul lochia, chills, lower abdominal pain ° Leukocytosis with a left shift

"O O oo H ~T3 > 73 H C

n >

TO m

12. W hat is the treatm ent for postpartum endometritis? 0 Antibiotic options: _____________________________+/- ampicillin 2g IV q6 hour Ceftriaxone + clindamycin - Ampicillin-sulbactam (Unasyn) + doxycydine Cefoxitin + doxycydine ° Continue antibiotics until afebrile for 24-48 hours ° No need for PO antibiotics after IV antibiotics unless blood culture is positive, in which case 7 days of oral antibiotics are given (e.g. clindamycin)

End of Session Quiz 13. A postpartum wom an presents w ith pain and tenderness o f the breast that is limited to only one region. There is no redness or warm th. W h a t is the most likely diagnosis?

14. W h e n can O C P s be initiated in postpartum patients who do not intend to breastfeed?

15. W ith in the immediate postpartum period, a patient develops sudden-onset hypoxia, cardiogenic shock and D IC . W h a t etiology is at the top o f your differential?

16. A patient loses more than 500 cc o f blood postpartum and now has anemia. Attem pts at breastfeeding have been unsuccessful, as it appears she is unable to generate any milk. W h a t diagnosis do you suspect?

[ 383 ]

Pediatrics 1

D evelopm ent

2

Infancy to Adolescence

3

Infections and Im m une D isorders

4

G enetic D isorders

5

Preventive M edicine

DEVELOPMENT

3 Question Warm-Up 1.

W h a t is the next step in the evaluation o f penetrating injuries to the different zones o f the neck?

2.

W h a t are the cardinal movements o f labor?

3.

A n IV drug user has JV D and a holosystolic m urm ur at the left sternal border. W h a t is the diagnosis? O r w hat is the treatment?

4. C a r Seats ° 3 sec

Urine out

Normal

Decreased

Anuric

[391

INFANCY TO AD O LESCEN CE

Adolescence 15. W hat mnemonic can you use for the questions you should ask during an annual adolescent exam? SHADESSS:

End of Session Quiz 16.

H ow many kilocalories are in an ounce o f breast milk? Formula?

17. W h a t is the #1 killer o f adolescents and why?

18. W h at newborn skin finding matches each o f the following descriptions? 2-3mm yellow pustule with red base (similar appearance to white head) arising in first 24-72 hrs, microscopic examination of the pustular contents (not necessary for diagnosis) reveals numerous eosinophils, usually gone by 3 wks Spider-webbing/marbling of the skin

Intense reddening of gravity-dependent side and blanching of the nondependent side with a line of demarcation between the two, lasts a few sec-min Due to accumulation of sweat beneath eccrine sweat ducts that are obstructed by keratin at the stratum corneum

3 Question Warm-Up 1* W h ic h vasopressor m atches each o f the following statements? ° Theoretically causes renal vasodilation ° High doses optimize the CXl vasoconstriction ° A D H analogue ° Best choice for anaphylactic shock ° Best choice for septic shock 0 Best choice for cardiogenic shock ° Causes vasoconstriction, but with bradycardia 2.

W h a t m edication is given to accelerate fetal lung m aturity, for how long is it given, and at w h at gestation is it no longer necessary?

3.

A pediatric p atien t presents w ith red currant jelly stools. W h a t is the diagnosis?

INFECTIONS AND IMMUNE D ISO RDERS

INFECTIONS AND IMMUNE DISORDERS

Pediatric Rash 4. W h at are the clinical features of measles infection (rubeola)? ° Prodrome for 2-3 days: fever, malaise, anorexia and 3 Cs (cough, coryza, conjunctivitis) ° __________________ on buccal mucosa after 1-2 days (white-gray spots with a red base) —>occur 48 hours prior to rash. Pathognomonic for measles ° Rash five days after prodrome onset: erythematous, maculopapular starting at the head then spreading to the feet —>lasts 4-5 days —>resolves from head down

5. W hat is the treatm ent for measles? ° Supportive therapy (antipyretics, fluids) ° Monitoring and treating bacterial superinfections such as pneumonia or otitis media O 100,000 IU PO xl in 6 -to 12-month-olds, 200,000 IU PO xl if olderthan 12 months W H O recommends vitamin A to all children with measles in areas where vitamin A deficiency is prevalent and measles mortality exceeds \% - A A P recommends vitamin A given as above to children 6 months - 12 years hospitalized for measles or its complications, or if immunodeficient or high likelihood of vitamin A deficiency (ophthalmologic evidence, intestinal malabsorption, malnutrition or recent immigration from an area with high measles mortality) » is not yet well studied and not currently standard of care for measles, despite that it harms measles virus in vitro -

I [ 393 ]

INFECTIONS AND IMMUNE DISORDERS

6. W hat are the classic features of rubella virus (German measles)? ° ° ° °

Low grade fever, lymphadenopathy and rash Prodromal malaise, fever, anorexia for 1-5 days prior to rash Lymphadenopathy —__________________________________________________ Erythematous, tender, maculopapular rash that starts at the___________________ , then generalizes ° Rash persists 5 days and does not darken as does the rash of measles ° Fever is mild and generally only on day 1, in contrast to measles ° Polyarthritis may be seen for up to a month in women and adolescents

7. W hat are the characteristic features of Coxsackie hand, foot, and mouth disease? ° Constitutional fever and anorexia ° ________________________ on the buccal mucosa and tongue ° Small, tender, maculopapular/vesicular rash on th e_________________________and sometimes buttocks ° Duration is typically 3-5 days without complications

8. W hat are the signs and symptoms of scarlet fever caused by Strep, pyogenes? ° 0 ° ° ° 0

Rash that is coarse (“sandpaper-like”), erythematous and blanching (“sunburn-like”) Rash starts on the trunk then generalizes, but________________________ Rash is most prominent in skin creases of axilla and groin (Pastia’s lines/sign) ________________________ , beefy-red pharynx, cervical LAD Fever/chills Later desquamation of hands and feet (additional DDx Kawasaki disease, toxic shock syndrome, acrodynia of mercury poisoning) ° Positive throat culture or rapid strep test

9.

W hat are the characteristic symptoms of roseola infantum (HHV-6)? ° Sudden, high fever (exceeding 102° F) for 3-4 days ° Child has no other signs of infection and often acts/plays normally ° Rash that appears when fever dissipates and starts on the trunk then spreads over entire body and lasts 24 hours 0 Other common findings: erythematous papules on soft palate and uvula, mild cervical LAD, edematous eyelids, bulging anterior fontanelle in infants 0 Commonly misdiagnosed as acute otitis media and subsequent antibiotic allergy!

10. W hat is the treatment for roseola infantum?

11. W hat is the differential diagnosis for cervical lymphadenitis in a child? 0 If acute and bilateral —> usually viral URIs: Rhinovirus, Adenovirus, Influenza, group A Strep Mono: EBV, CMV, Mycoplasma - Other viruses: HIV, HSV ° If acute and unilateral —>usually bacterial {Staph, aureus, group A Strep > anaerobes, GBS) ° If chronic and unilateral: Bartonella henselae —> cat scratch fever Toxoplasmosis TB —» scrofula - Actinomyces israelii —» sinuses drain pus ° Noninfectious causes (much less common): Kawasaki disease, Hodgkin lymphoma

INFECTIONS AND IMMUNE DISO RDERS

O ther Pediatric Infections

12. W hat is P FA P A syndrome? ° Benign 4-5 day syndrome consisting of Periodic Fever, Aphthous ulcers, Pharyngitis, and Adenitis Occurs monthly (q28 days) Exclusion criteria include neutropenia, cough, coryza, diarrhea, severe abdominal pain, rash, arthritis and neuro defects ° Usually affects preschool-aged children (2- to 5-year-olds) 0 Benign, self-limiting disease ° Treatments Glucocorticoids relieve symptoms in a matter of hours Cimetidine may be used for prevention of episodes, but is of questionable efficacy ° Average duration of recurring symptoms is 4.5 years

13. W hat are the classic symptoms of pertussis? ° Incubation 7-10 days ° Catarrhal stage (7-10 days): mild URI symptoms ° Paroxysmal stage (_____________________): paroxysms of cough with inspiratory whoop that is worse at night and often with post-tussive emesis and exhaustion Often confused with acute bronchitis 0 Convalescent stage (2-3 weeks): waning of symptoms

14. W hat is the treatm ent for pertussis? ° Antibiotic choices:___________________ (5 days), erythromycin (14 days), clarithromycin (7 days), TM P-SM X (14 days) ° Prophylaxis for close contacts (full course of one of the above antibiotics) ° Isolation from school/day care until 5 days of antibiotics have been completed, or three weeks after the onset of symptoms in untreated patients ° Hospital admission (with isolation) indications for children with pertussis: Respiratory distress Pneumonia Inability to feed Cyanosis or apnea (with or without coughing) - Seizures

395

INFECTIONS AND IMMUNE DISORDERS

15. W hat additional work-up, if any, is needed in a child diagnosed with a UTI? Renal and Bladder Ultrasound (RBUS) ° Child < 2 years with febrile UTI ° Child any age with recurrent febrile UTIs ° Child any age with family history of urologic disease, poor growth or hypertension ° Child with UTI that does not respond to antibiotics Voiding Cystourethrogram (VCUG) ° Child any age with > 2 febrile UTIs ° Child any age with first febrile UTI and family history of urologic disease, poor growth or hypertension

Immunodeficiencies 16. Bruton Agammaglobulinemia ° ° 0 °

X-linked (Boys) B cell deficiency —>defective tyrosine kinase gene —>low levels of all immunoglobulins Recurrent bacterial infections after 6 months No B cells on peripheral smear

17. Thym ic Aplasia (DiGeorge Syndrome) ° 3rd and 4th pouches fail to develop —> No thymus —>no mature T cells No parathyroids —> no PTH —> low Ca2+ —» tetany ° Congenital defects in heart / great vessels ° Recurrent viral, fungal, protozoal infections 0 90% have a chrom 22qll deletion

18. Severe Combined Immunodeficiency (S C ID ) ° Defect in early stem cell differentiation ° Can be caused by at least 7 different gene defects (e.g., adenosine deaminase deficiency) Presentation Triad 1) Severe re c u rre n t infections

- Chronic mucocutaneous candidiasis - Fatal or recurrent RSV, VZV, HSV, Measles, Influenza, Parainfluenza - Pneumocysticjirovetii pneumonia (PCP) 2)

C hronic diarrhea

3)

Failure to thrive

° No thymic shadow on newborn CXR ° DO NO T GIVE LIVE VACCINES

19. Chronic Mucocutaneous Candidiasis ° T cell dysfunction v. C. albicans ° Rx: antifungals (ketoconazole, fluconazole)

96

° ° ° ° ° °

Wiskott Aldrich Immunodeficiency Thrombocytopenia and purpura Eczema Recurrent pyogenic infections

21. Ataxia-Telangiectasia ° IgA deficiency ° ° ° ° ° °

Cerebellar ataxia and poor smooth pursuit of moving target with eyes Telangiectasias of face > 5yo ”[ Cancer risk: lymphoma 8c acute leukemias Radiation sensitivity (try to avoid x-rays) +/- 'j'AFP in children > 8m Average age of death = 25 years

INFECTIONS AND IMMUNE D ISO RDERS

20. W iskott-Aldrich Syndrome

22. Selective Immunoglobulin Deficiencies ° IgA deficiency is most common - Most appear healthy - Sinus and lung infections 1/600 European descent - Associated with atopy, asthma Possible anaphylaxis to blood transfusions and blood products

23. Chronic Granulomatous Disease (C G D ) ° Lack of NA D PH oxidase activity —* impotent phagocytes ° Susceptible to organisms with catalase {S. aureus, E. coli, Klebsiella spp., Aspergillus spp., Candida spp.) ° Dx: - Classic test is a negative nitroblue tetrazolium test (no yellow to blue-black oxidation) More commonly diagnosed by flow cytometry ° Prophylactic TM P-SM X ° IFN-y also helpful

24. Chediak-Higashi Disease ° Defective LYST gene (lysosomal transport) ° Defective phagocyte lysosome —» giant cytoplasmic granules in PMNs are diagnostic Presentation Triad: 1) Partial albinism 2) Recurrent respiratory tract and skin infections 3) Neurologic disorders

[ 397]

INFECTIONS AND IMMUNE DISORDERS

25. Hyperimmunoglobulin E Syndrome (Job Syndrome) ° Deficient IFN-y * impaired neutrophil chemotaxis ° High levels oflgE and eosinophils Presentation Triad: 1) Eczema 2) Recurrent cold Staph, aureus abscesses (think of biblical job with boils) 3) Coarse facial features: broad nose, prominent forehead (“frontal bossing''), deep set eyes and "doughy” skin

° Also common to have retained primary teeth resulting in 2 rows of teeth

26. Leukocyte Adhesion Deficiency Syndrome ° Abnormal integrins —>inability of phagocytes to exit circulation ° Delayed separation of umbilical cord

End of Session Quiz 27.

W h a t are the 3 Cs o f the prodrome o f rubeola?

28.

W h a t are some causes o f desquamation o f the hands and the feet?

29.

W hich viral infection is characterized by sudden high fevers for 3-4 days but is otherwise asymptomatic? A rash will often appear when the fever dissipates.

30. W hich immunodeficiency matches each o f the following descriptions? Congenital heart defect + low calcium + recurrent infections Chronic mucocutaneous candidiasis + chronic diarrhea + failure to thrive Negative nitroblue tetrazolium test Poor smooth pursuit of eyes + elevated A FP after 8 months Partial albinism + recurrent URIs + neurological disorders 31. W hen do infections typically begin in children w ith immune disorders?

|398|

1.

W h a t is “cradle cap” and w hat is the treatm ent?

2.

W h a t is the cause o f bilious emesis in a new born w ith in hours after the first feeding?

3.

W h a t are the tw o m ost com m on prim ary brain tum ors in adults? W h a t are th e 3 m ost com m on prim ary brain tum ors in children?

DISO RDERS

3 Question Warm-Up

GENETIC

GENETIC DISORDERS

4. W hat is the most com m on malformation of the head and neck?

5. W hat features are characteristic of fetal alcohol syndrome? ° Facial features: short palpebral fissures, thin upper lip, smooth philtrum, flattened midface ° Deficient brain growth: structural brain abnormalities, < 10th percentile for head circumference, abnormal neuro exam, variable intellectual disability ° Growth retardation: < 10th percentile for height and weight, failure to thrive despite adequate intake, disproportional low weight to height

6. W h at congenital defect is associated with lithium use during pregnancy? Ebstein anomaly: ° Tricuspid leaflets are displaced into right ventricle, hypoplastic right ventricle, tricuspid regurg or stenosis ° 80% have a patent foramen ovale with a R —> L shunt ° Dilated right atrium —>increased risk of SVT and W P W ° Physical exam: widely split S2, tricuspid regurgitation

7.

W h at are the differences in presentation between a branchial cleft cyst and a thyroglossal duct cyst?

[ 399 ]

GENETIC DISORDERS

8. W hat are the common possible presenting features of tuberous sclerosis? ° ° ° ° ° ° °

9.

Distinctive brown, fibrous plaque on the forehead seen in infancy Ash leaf spots (hypopigmented macules) - most easily identified with Wood’s lamp Shagreen patch (leathery cutaneous thickening usually on the lower trunk) Facial angiofibromas (AKA adenoma sebaceum) Seizures Intellectual disability Subependymal nodules

W hich glycogen storage disease matches each of the following statements? Lactic acidosis, hyperlipidemia, hyperuricemia (gout) Diaphragm weakness —» respiratory failure Increased glycogen in liver, severe fasting hypoglycemia Hepatomegaly, hypoglycemia, hyperlipidemia (normal kidneys, lactate, and uric acid) Painful muscle cramps, myoglobinuria with strenuous exercise Severe hepatosplenomegaly, enlarged kidneys

•100]

End of Session Quiz 10. I f a female child is in the 80th percentile for height and the 25th percentile for head circum ference, w hat chrom osom al abnorm ality should you suspect?

11. W h a t genetic disorder matches each o f the following descriptions? Cleft lip/palate, life expectancy < 1 yr, polydactyly High-pitched, cat-like cry Elfin facial features, cardiac defects Tall, thin man w ith gynecomastia and testicular atrophy Obesity and overeating M icrognathia, life expectancy < 1 yr, rockerbottom feet Happy mood, inappropriate laughter, ataxic gait Large ears, ID , macroorchidism M R , simian crease, G I and cardiac defects Short stature, infertility, coarctation o f aorta

12. A child presents w ith short palpebral fissures, a thin upper lip, smooth philtrum and a flattened midface. H e is below the 10th percentile for height and weight, and his IQjis 65. W h a t is the cause o f his abnormalities? W h a t could have been done to prevent them?

13. W h ich glycogen storage disease matches each o f the following statements? Lactic acidosis, hyperlipidemia, hyperuricemia (gout) Diaphragm weakness —» respiratory failure Increased glycogen in liver, severe fasting hypoglycemia Hepatomegaly, hypoglycemia, hyperlipidemia (normal kidneys, lactate, and uric acid) Painful muscle cramps, myoglobinuria with strenuous exercise Severe hepatosplenomegaly, enlarged kidneys

PREVENTIVE MEDICINE

3 Question Warm-Up 1.

H ow can the flushing reaction o f niacin be prevented?

2.

W h a t supplements should be given to wom en on anticonvulsants during pregnancy?

3.

W h a t findings would you expect to see in a post-term pregnancy beyond 42 weeks gestation?

W hat is the U S P S T F recommendation for hypertension screening?

W hat is the U S P S T F recommendation for cholesterol screening in patients without C A D risk factors?

W hat specific interventions are helpful in smoking cessation? ° Counseling to quit at every encounter - Ask, assess, advise, assist, and arrange Set a precise quit date while in clinic —> instruct patient to throw away all smoking paraphernalia including ash trays and lighters Offer American Cancer Society Quitline: I-800-ACS-2345 Offer prescription therapies to help them quit, especially varenidine (see below) ° Nicotine replacement doubles the quit rate Much higher efficacy when used with bupropion (see below) Patch: start 21mg —> I4mg (start here in light smokers) —» 7mg. Titrate over 10 weeks. Gum: chew until soft, then park it between cheek and gum for nicotine absorption Others: inhaler, nasal spray, lozenge ° Prescription therapies to help them quit Varenidine (Chantix) 30% I year success rate (12 week therapy), 44% I year success rate (24 week therapy) (JAMA 296:64,2006) SE: nausea, suicidal thoughts, bizarre dreams Bupropion (Zyban or Wellbutrin) Instruct pts to quit after I week. More effective if used with nicotine replacement. SE: dry mouth, weight loss. Contraindications: seizure d/o, pregnancy, already on 3 or more psychiatric meds, bulimia

Preventive Medicine for Ages 50-65 History and Counseling: ° Ask and assist with smoking and alcohol use ° Remind about exercise, high fiber (30-35g/day), sunscreen (SPF 15+), seat belts and highrisk behaviors ° Ask about depression, assault/abuse, advance directive/will ° Dental exam/cleaning q6 months-1 year ° Donation of blood and bone marrow (< 60 years old) Physical: ° PE with BP check, BMI, skin cancer screen ql year ° Snellen eye exam ql year M eds/Interventions: ° Daily MVI, calcium 1200mg/d and vitamin D >600 IU/d ° Flu shot ql year ° Tetanus-diphtheria booster (Td or Tdap) qlO years. Use Tdap instead of Td if no h/o Tdap. ° Assess for varicella immunity by h/o chickenpox/shingles, h/o vaccine, or serology —> vaccinate if nonimmune ° Zostavax once after age 60 regardless of shingles history ° Consider ASA 81mg QD (to help prevent M I, stroke, colon CA) Proven benefit if 10 year Ml risk is > 10%, or annual Ml risk is > 1% - Women who take A SA had a 25% decrease in deaths from all causes (Arch Intern Med 2007; 167:562) Lab/Studies: ° FOBT q l year ° Colonoscopy qlO years or Flex sig/BE q3-5 years ° TSH q5 years ° Lipid Panel q5 years ° Consider glaucoma screening ql-5 years (esp. Blacks) ° Consider EKG q5 years ° Consider regular Chem 14, UA, and CBC (ql-5 years) ° Offer H IV test

PREVENTIVE M EDICINE

7.

Vitamin Supplementation 8. W hich vitamin deficiencies match each of the following descriptions? ° ° ° ° ° ° ° ° ° ° °

Increased RBC fragility Dermatitis, cheilosis, glossitis Peripheral neuropathy, angular cheilosis, glossitis Hemorrhagic disease Neural tube defects Dermatitis, diarrhea, dementia Megaloblastic anemia Pernicious anemia Bitot’s spots, keratomalacia, xerophthalmia Osteomalacia Rickets

I [403 ]

Which vitamins match each of the following statements? ° ° ° ° ° ° ° °

Can be used to treat acne and psoriasis Involved in the hydroxylation of prolyl residues Requires intrinsic factor for absorption Deficiency may result from kidney disease Given prophylactically to newborns Can be used to elevate H D L and lower LDL Deficiency can be caused by isoniazid use Cobalt is found within this vitamin

End of Session Quiz 10. H ow often should a normotensive patient get their blood pressure checked?

11. W h en should you start screening for high cholesterol in otherwise healthy patients?

12. W h ich medication for sm oking cessation carries a black box w arning about its side effect o f suicidality?

13. W h a t sm oking cessation m ethod doubles the quit rate?

14. W h a t symptoms o f vitam in deficiencies do you get if you haven’t had enough folate? A? D? E? B3?

15. W h a t vitam in can be used to treat psoriasis?

16. W h a t vitam in can be used to prevent deficiency in isoniazid?

17. W h a t vitam in can be used to elevate H D L and lower LD L?

View more...

Comments

Copyright ©2017 KUPDF Inc.
SUPPORT KUPDF